click below
click below
Normal Size Small Size show me how
NBME (all) Review
Question | Answer |
---|---|
Polio | CSF=Lymphocytic Pleocytosis |
MS | oligonal bands on electophoresis |
ALS (Lou Gherig's Disease) | defect of superoxide dismutase 1 |
EBV | china kissing monoclonal carcinoma of the nasopharynx |
Internal Iliac | branch ligation required to control heavy vaginal bleeding |
Streptococcus Pneumoniae | prophylaxis required if pt has a splenctomy |
Locked-in Syndrome | basilar artery |
Inhaled glue | "pt not themself" confusion, clumsiness, frequent falls ataxic gait |
Xanthoma/ Achielles Tendon | absence of functional LDL receptors in hepatocytes |
Middle Meningeal Artery | pupils 4 mm in diameter and not reactive to light; due to trauma in temporal area |
Activation of Adenylyl Cyclase Toxin | stool shows gram negative, comma-shaped bacteria (Vibrio Cholerae) |
Alveolar Macrophage HALLMARK | shipyard workers |
NSAIDs use in hypertensive pt with bilateral renal artery stenosis | vasodilating prostaglandins at the afferent arteriole |
Calcitonin | marker to monitor for thyroid neoplasm |
Aortic Stenosis | cardiac valve defect and concentric left ventricular hypertrophy (systolic murmur) |
MS | Charcot's Triad |
Aldoesterone-secreting Adrenal Adenoma | decreased plasma renin activity |
Malignant Hyperthermia | decreases release of Ca+ from the SR |
9mo. old Baby | pincer grasp finger feeding standing while holding onto a table playing peek-a-boo |
Leptin | suppresses appetite by its action in the CNS |
21-Hydroxylase | female; facial hair chest hair clitoromegaly normal uterus increased concentrations of 17-hydroxyprogesterone and androstenedione |
Trypanosoma Cruzi | brazil organism (think cruziero soccer) |
Resolution of Pneumococcal Pneumonia | metaplasia of mesenchymal cells to pneumocytes |
Interstitial Pulmonary Fibrosis | Increased radial traction on airways |
Early Septic Shock Rx | IV 1 liter of isotonic saline (clean the blood) |
Dissecting Aneurysm X-ray | widened aortic arch |
Ascites Rx in addition to loop diuretics | spironlactone |
Randomized clinical trial | subjects assigned by coin toss to one of two groups |
Tumor Necrosis Factor | antibodies directed against in monocolonal antibody preparation |
Ezetimibe MOA | cholesterol decreasing drug inhibiting the transport of cholesterol through the intestinal wall |
DVT Rx | potentiates the action of antithrombin III |
First Degree Burn | severe erythema of back and extremities NO BLISTERS |
Osteomyelitis | predominance of neutrophils persistent FOOT PAIN |
Alendronate MOA | inhibition of osteoclast-mediation bone resorption |
Vagus Nerve | protude the tongue and say "Ah" |
GI Blood Loss labs | Hemoglobin: 9.5g/dL Leukocyte: 5400 Platelets: 350,000 |
Left Achilles Tendon Reflex | strong with lower extremity is immobilized in a cast |
Loperamide | opioid antidiarrheal agent with little CNS effect |
IgM | antibody isotype most likely causing agglutination; the isotype in anemia |
Nephrolithiasis (Kidney Stones) | can occur due to gout |
Major Depressive Disorder | low energy; irritability; crying spells; difficulty falling asleep; waking up frequently at night |
Proliferative Endometrial Tissue seen in | biopsy of ovarian cyst and/or peritoneal cyst |
5a-reductase gene mutation | labia majora would develop into scrotum |
Parietal cell absence | after gastrectomy |
Rosacea | erythema over nose, cheeks, and scattered telangiectasia and a few papules |
Pelvic Splanchnic Nerve Dysfunction Sx | constipation abdominal distension |
Listeria MonOCYTOgenes | gram positive rod fever, chills, and muscle aches can occur in pregnant women |
Ultrasound shows bilateral hydronephrosis and dilated ureters | increased hydrostatic pressure in bowman space leading to renal failure |
AflAtoxin | china peanut farmer |
Delirium | pt is confused by everything and is known to not behave in such a manner at home |
Bronzing of Skin | increased intestinal iron absorption |
Inferior Rectal Artery | 15-mm, blue-tinged, rounded mass at the anal margin |
Newborn with absence of bowel gas in abdomen | incomplete formation of pleuroperitoneal membrane |
Dynein arms | absent in epithelium in pts. with Kartagener Syndrome (Cystic Fibrosis) |
Metastatic Tumor to Cerebellum | ataxia of the left upper and lower extremities |
Vitamin B12 Deficiency | spasticity in arm and legs impaired proprioception in feet increased muscle stretch reflexes in arms and knees absence of muscle stretch reflexes in ankles bilateral extensor plantar responses |
Farmer with 7-mm red scaly plaque on the helical rim of left ear | actinic keratosis |
Tick Bite in Farmer Rx | doxycycline |
Renal Cell Adenocarcinoma | hypervascular mass 12-cm solid mass on kidney Sx include flank pain gross hemateuria |
Physician response to pt.'s Rx noncompliance | "using something twice daily can be difficult. I assume you are like most patients who miss at least 10% of treatments" |
Sacroiliac Joint Inflammation sx | lower back pain with stiffness stiffness when sat for prolonged periods of time pain radiates down to buttocks |
Ringing of Ears sx | salicylate/ asprin poisioning |
Ulcers Rx | sulfasalazine |
Ubiquitin Ligase MOA | promotes cell growth and malignancy by causing cellular p53 protein degradation |
Trophoblastic tissue | necrotic intrauterine mass and metastatic nodule in the lungs |
Bilateral Bell's Palsy | adverse effect of: Lyme Disease and Guillan-Barre |
Nephroblastoma sx | congenitial urethral obstruction; marked dilation of ureters and renal pelvis minimal renal function |
Thyroidectomy Consequences | decreased PTH decreased Calcium parenthesis of hands and feet |
Pudendal Nerve Block | Ischial spine injection with lidocaine alternative to epidural in labor |
Increased JVP and mild ankle edema | increased capillary hydrostatic pressure |
Tricuspid Valve | 2/6 holosystolic murmur, left fifth intercoastal space ajacent to sternum and increases with inspiration |
Camping Trip results in itchy rash in arms and legs | activation of T lymphocytes |
CMV infevction sx | petechial RASH, microcephaly, and HEPATOSPLENOMEGALY |
Puberty begins when | breast buds develop |
Hyperthyroidism sx | thyroid antibodies |
HSV sx in Males | broken blisters and open sores on penis |
Beta-Thalassemia | disruption of normal splicing by creation of a new 3' splice site mutation from G --> A at position 355 |
Post Par-tum Depression Rx | SSRI |
Fibrinous Pericarditis sx | pericardial friction rub after acute myocardial infarction |
Cervical Biopsy in Microinvasive Cervical Carcinoma | neoplastic cells in sub-basement membrane connective tissue |
Increased amniotic fluid volume | tracheoesophageal atresia |
IL-8 responsibility | recruitment of neutrophils to inflammatory site |
Warfarin MOA | VII (proconvertin) clotting factor first to be decreased by 50% after initiation of therapy |
Ectopic Pregnancy | vaginal bleeding direct and rebound tenderness with guarding b-hCG elevated closed cervical os |
Overdose of Triiodothyronine in Primary Hypothyroidism | decreased TSH, Free Throyxine Increased Free Triiodothyronine |
Cocaine Users | predisposition to myocardial ischemia |
The binding site and action of hCG that causes gynecomastia | testicle; produces estradiol |
Strings of Beads HALLMARK | fibromuscular dysplasia |
Bacterial sx are a result of systemic release of | IL-1 and TNF-alpa |
Clostridium Difficle toxin | intestinal tract will contain pseudomembranes of fibrin and inflammatory debris |
Lost in Menopause | ovarian secretion of 17beta-estradiol |
PTT prolongation | defect in factor 8 (VIII) antihemophilic factor |
Organs glucose produced in | liver + kidney |
Elderly pulmonary function | increased residual volume and alveolar-arterial Po2 difference decreased arterial Po2 |
Methylamphetamine (street drug Meth) MOA | increased release of dopamine and norepinephrine |
Asthmatic Pt. with Pollen Allergy Rx | Albuterol (acute resolution) |
Ibutilide Adverse Effect | Torsades de Pointes |
Mallory Hyaline in biopsy seen in | alcoholic hepatitis |
The right eye does not adduct past the midline on horizontal gaze when looking to the left indicates a lesion involving | abducens nerve |
The persistence of leukocytosis in the absence of infection indicates impairment in | leukocyte adhesion and transmigration |
Urethral Prolapse | seen in elderly women with vulvar itching that has not resolved with treatment physical examination with show atrophy and thinning of labia minora |
Rx inhibiting 30s ribosome binding | gentamicin (class of aminoglycoside) |
H. Influenzae Type B Vaccine | T-independent antigens to T-dependent forms to enhance protection |
Condition causing ANEMIA and SPLEEN to increase 5x it's normal size | Hereditary Spherocytosis |
Antihypertensive causing descreased serum potassium concentration | hydrocholorthiazide |
1,25-Dihydroxycholecaciferol | active form of vitamin D |
Calcified 80% Stenois can occur in | pts who smoke, are hypertensive and experience continued extertional chest pain that is relieved by rest |
Sepsis Rx | 0.9% Saline |
Indication for a Pacemaker | third-degree atrioventricular block, hypotension and variable intensity S1 |
Erectile dysfunction Rx MOA | inhibition of phosphodiesterase (PDE5 inhibitor) |
Autoantibody with affinity for acetylcholine receptor seen in | myasthenia gravis |
Physician answer to pt inquiry on whether weight gain is hereditary | "Yes, your weight gain can be caused by genes and enviromental factors" |
Alcoholic Liver Disease adverse effect | gynecomastia |
Surgical intervention shows half of the small intestine is found to have a dark purple-red hemorrhagic appearance | Mesenteric venous thrombosis |
Epinephrine | stimulates hormone-sensitive lipase in adipocytes that resulted in the accumulation of metabolites |
Systemic Amyloid | renal biopsy will show beta-pleated sheet structure |
Defective Heme Synthesis | porphyria; ALA rate limiting enzyme |
Rx for Urinary Urgency | inhibit of muscarinic receptors |
Naloxone | antidote for NARCOTIC OVERDOSE in an emergency situation |
Tardive Dyskinesia | grimaces rigid jerking purposeless movements of fingers |
Osteogenesis Imperfecta | type 1 collagen |
Cyclosporine MOA in transplant patients | suppresses early response of T lymphocytes to activation |
Schizoid Disorder | condition in which pt. avoid interacting with others and social activities in addition to being cold and detached |
First Generalized Tonic-Clonic Seizure occurs from abnormality in | CalCium |
Loraradine Rx used to treat | allergies; runny nose, itchy and watery eyes |
Physiological changes when in water of 60F for 20 mins | increased ADH and ANP decreased central blood volume |
Sepsis after abortion causation | decreased plasma fibrinogen concentration |
Physician response when pt claims diagnosis is bad news | "Yes, it is" |
Diagnosing possible defect in fatty acid oxidation | measurement of serum amino acid concentrations |
Myocardinal infarction (CK-MB; troponin) markers result from | protease inactivation by cytoplasmic free calcium ions |
Neurofibromatosis Type 1 | autosomal dominant |
Chandelier Sign (cervical motion tenderness) | gonorrhea, PID, or ectopic pregnancy dx |
Physician response to pt in denial | "It must be difficult for you to ACCEPT this diagnosis when you feel healthy" |
Criteria for informed consent | family agreement, competence, and cost |
Renal stone composition when urine analysis pH is 8 | struvite |
Cellular production in second-degree burn precipitated by which component | C5a; produced increased fibroblast migration and proliferation, increased synthesis of collagen and fibronectin, and decreased degradation of extracellular matrix by metalloproteinases |
Damage to right subthalamic | Hemiballisum; uncontrollable irregular movements of the LEFT side of the body |
Damage to posterior pituitary gland | progressive thirst and urinary frequency |
Abnormality in 6 week old baby vomitting a small amount of milk 2 to 3 times a day | immature lower esophageal spinchter |
Poxvirus sx | firm, smooth, umbilicated papules 2 to 4 mm in diameter in clusters |
Bullous Pemphigoid results from | development of autoantibodies against desmosomal proteins |
Bulimic patients will have enlargement of | parotid gland |
HCO3- levels in advanced phase of COPD | increased urinary excretion |
Most common benign tumor of connective tissue | myoxma |
Dysphagia causing difficulty swallowing solids, dilated cardiomyopathy would be as a result of enlargement where | left atrium |
Agent used to slow the DNA replication process of neoplastic cells in cancer patients | cyclophosphamide |
Weakness of grasp is indicative of | rheumatoid arthritis |
This casual virus replicates its genome within the cell's nucleus | |
Caspofungin MOA | beta-glucan carbohydrates in the cell wall |
Stool analysis showing increased fat concentration indicative of deficiency in | Vitamin E |
Physician response when pt. complains about tardiness | "I'm sorry I got delayed. I hope I haven't made you late somewhere else" |
L3 to L4 herinations causes pain | down the distal anterior thigh knee medial leg and foot |
Africa trip + Wright Stain dx | Malaria |
Subclavian Central Catheter bacterial infection | Staph Aureus + Enterococcus Facalis |
E-Coli Virulence Factor | Mannose-binding (type 1) fimbria |
Pellagra | 3 D's: dermatitis, dementia, and diarrhea (confusion, rash, and diarrhea) lack of NICOTINIC ACID or its precursor, TRYPTOPHAN in their diet |
Hypertensive patient already on hydrocholorthiazide if not responding to Rx should be put on this ACE inhibitor | Lisinopril |
Pt. experiencing proteinuria and elevated hemoglobin is likely to have which elevated marker? | Creatine Kinase |
Vertebra region in the lower right quadrant about 5 cm superomedial to the anterosuperior iliac spine? | T10 |
Which area should be avoided during ablation to leave the sinoatrial (pacemaker) node intact in a pt.? | the junction of the superior vena cava and the right atrium |
Marked increase in dopamine concentration when rising from a supine position to standing marks a deficiency of? | dopamine beta-hydroxylase |
Most common bacterial infection from breast feeding | staphylcoccus aureus |
Embryology: Syncytiotrophoblast secretes which hormone | hCG |
Embryology: Trophoblast acquisitions energy from | mother |
Embryology: Inner Cell mass will | establish order |
Embryology: Progesterone maintains the endometrial lining to prevent | menstration |
Embryology: Neural Crest derived from ECTODERM, but which layers make this even happen | notocord + mesoderm |
Neural Crest derivatives | PNS, ear, eye adrenal gland, mouth, heart, digestive system, thyroid, and skin |
Embryology: Germ layers serving as secondary energy reservoire | endodermal yolk sac |
Embryology: Week 10 | sex of fetus |
Teratogenic Rx: Aminoglycosides | Ototoxicity |
Teratogenic Rx: ACE inhibitors | renal malformations |
Teratogenic Rx: Fluroquinolones | cartilage damage |
Teratogenic Rx: Tetracyclines | discolored Teeth |
Teratogenic Rx: Cyclophosphamide | facial anomalies, limb hypoplasia, absence of digits |
Teratogenic Rx: Methotrexate | abortion, NTD |
Teratogenic Rx: Carbamazipine | NTD |
Teratogenic Rx: Valporic Acid | NTD |
Teratogenic Rx: Phenytoin | fetal hydration syndrome |
Teratogenic Rx: Lithium | ebsteins anomaly |
Teratogenic Rx: Statins | cns and limb anomalies |
Teratogenic Rx: Wafarin | facial/ limb/ CNS anomalies, spontaneous abortion |
Teratogenic Rx: Diethylstilbestrol (DES) | clear cell vaginal adenocarcinoma |
Teratogenic Rx: Thalidomide | phocomelia; rare congenital deformity in which the hands or feet are attached close to the trunk, the limbs being grossly underdeveloped or absent. |
Teratogenic Rx: Isotretinoin | spontaneous abortion |
Holoprosencephaly Consequence from | ethanol |
Limb Hypoplasia Consequence from | thalidomide; cyclophosphamide |
Homebox (HOX) Genes | blueprint for skeletal morphology (where things are supposed to go); code for transcription regulator |
Mutation HOXD-13 genes will result in | synpolydactyly (fused 3rd and 4th digit) |
4 chamber heart begins to develop at week | 4 |
Most common cause of NTD | folate deficency |
Most common cause of congenital malformations in the US | alcohol use in pregnancy |
Most common cause of congenital intellectual disability in the US | fetal alcohol syndrome |
Vitamin A excess during pregnancy will lead to a | cleft palate |
COP-II functions in the cell cycle to help move products | from ER to cis-Golgi; anterograde trafficking |
This helps transport hydrolase enzymes from trans-Golgi to lysosome | clathrin |
Mannose-6-phosphate deficency results in | I-cell disease |
Mitchondria acts as buffer to | calcium |
Intermediate Filament Structures: Vimenten | Structural component of: connective tissue |
Progeria (Advanced Aging) | nuclear lamins mutation |
Cholesterol makes up | 50% of the plasma membrane |
Membrane Bound Receptors | Tyrosine Kinase G-Protein-coupled Steroid receptors |
PDGF and IGF-1 receptors | 2alpha bound by disulfide bonds bidning extracellular ligand |
Main Role of Plasma Membrane | keeps sodium out of the cell; potassium inside of the cell |
Corticosteriods inhibits | phopholipase A2 |
Zileuton inhibits | lipoxygenase |
Zafirlukast and Montelukast inhibits | leukotrienes |
Cyclooxygenase inhibited by | NSAIDS, Acetomenaphin, COX-2 |
Cell membrane lipid converted to arachidonic acid by phospholipase A2 | phosphatidylinositol |
Which cells are constantly regenerating (stays in G1 phase and no G0 phase) | skin, hair follicles, and bone marrow |
In apoptosis, the plasma membrane is | blebbing away via phagocytosis |
Intrinsic pathway: bcl-2 is | anti-apoptotic |
Intrinsic pathway: BAX is | pro-apoptotic; increase in mitochondrial permability |
Extrinsic pathway: Death Receptor | TNF + Fas |
Extrinsic pathway: Killer T Cells use | perforin |
Result of Granzyme B entering cells | activation of caspases |
Intrinsic pathway: Increased Mitochondrial permeability releases | Cytochrome C |
Irreversible Cell Injuries | nuclear pyknosis karyolysis karyorrhexis Ca2+ influx leading to caspase activation |
Neutrophils found in | acute inflammation |
Collagen primarily needs | vitamin C |
Leukocyte Adhesion Syndrome | delayed umbilicus seperate |
Leukocyte Adhesion Syndrome | abnormal integrin molecules |
Granulomas are found in | chronic inflammation |
Collection of macrophages ultimately become | granulomas |
Receptor for Rolling (inflammation) | E-Selection (endothelium) |
Receptor(s) for tight binding (inflammation) | LFA-1; ICAM-1 |
Metals known to facilitate production of oxygen free radicals | iron and copper |
Strong (I) Slippery (II) Bloody (III) BM (IV) | collagen |
Step 1 in the production of collagen | precollagen sythesis alpha chains |
Step 2 in the production of collagen | hydroxylation of lysine and proline |
Step 3 in the production of collagen | glycosylation of hydroxylated lysine essentially making procollagen |
Step 4 in the production of collagen | exocytosis |
Collagen construction taken place in | fibroblasts |
Step 5 in the production of collagen | cleavage of terminal collagen tropocollagen |
Step 6 in the production of collagen | crosslink tropocollagen molecules to make collagen fibrils |
Steroioids inhibit collagen | synthesis |
Osteogensis Imperfecta is an abnormal or defect in | Type I collagen |
Osteogensis Imperfecta | Autosomal Dominant |
Osteogensis Imperfecta Hallmark | blue sclera |
Ehlers Danlos Syndrome Hallmark | hyperjointibility |
Kidney disease + Deafness + Eye problems | Alport Syndrome |
Alport Syndrome defecent in type | IV |
Cant see, Cant pee, Cant hear high C | Alport Syndrome |
Marfan Syndrome | hyperELASTICITY |
Marfan Syndrome defect in | fibrillin, which makes elastin |
Marfan Syndrome Hallmark | elasticity of skin |
a-1 antitrypsin deficency | defect elastin in alveoli |
Hypothalamus: Nucleus Masterclock for Circadian Rhythm | suprachiasmatic nucleus |
Hypothalamus: Nucleus making oxytocin | paraventricular nucleus |
Hypothalamus: Nucleus secretes ADH | supraoptic nucleus |
Hypothalamus: Nucleus Dealing with Obesity + Savage behavior | dorsomedial nucleus |
Hypothalamus: Nucleus inhibited by leptin | lateral nucleus |
Hypothalamus: if Nucleus destructed leads to obesity | ventromedial nucleus |
Hypothalamus: Nucleus regulating dopamine and GHRH | arcuate nucleus |
Hypothalamus: Nucleus conserving heat producing shivering in cold enviroments | posterior nucleus |
Hypothalamus: destruction of this body will lead to | wernicke's encephalopathy |
Hypothalamus: Nucleus stimulating GI | dorsomedial nucleus |
Posterior Pituitary otherwise known as | neurohyphysis |
Posterior Pituitary is derived from which germ layer | neuroectoderm |
Hypothalamus: Nucleus receives input from retina | suprachiasmatic nucleus |
Hypothalamus: Savage behavior + obesity from stimulation | dorsomedial nucleus |
Hypothalamus: Savage behavior + obesity from destruction | ventromedial nucleus |
Hypothalamus: Stimulation leads to eating and destruction further leading to starvation | lateral nucleus |
Hypothalamus: Regulates release of LH and FSH | preoptic nucleus |
Hypothalamus: Destruction results in neurogenic diabetes inspidius | supraoptic nucleus |
Hypothalamus: releases hormones affecting the anterior pituitary | arcuate nucleus |
Hypothalamus: which nucleus regulates appetite | lateral nucleus |
Neuro: decreased GABA + serotonin BUT increased norepinephrine is seen in | anxiety disorders |
Sleep stages: Stage N1 | theta waves |
Sleep stages: Stage N2 | bruxism (teeth grinding) sleep spindles and K complexes |
Sleep stages: Stage N3 | delta waves sleep walking bed wetting |
Sleep stages: REM | beta waves lose muscle tone |
Sleep stages: Relaxation | alpha waves |
Sleep stages: Rx decreases bed wetting (nocturnal enuresis) | imipramine |
Sleep stages: Rx MOA of impramine | TCA |
Sleep stages: Desmopressin (DDAVP) can aslo be used as Rx for | decreasing urination |
Sleep stages: Rx indomethacin decreases | renal blood floow |
Sleep Rxs: this med gives vivid dreams and should not be used for more than 3 months | melatonin |
Sleep Rxs: herbal remedy found OTC | valerian |
Sleep Rxs: first-line treatment for difficulty sleeping | anti-histamines |
Sleep Rxs: this med increases risk of priaprism | Trazodone |
Sleep Rxs: this med increases REM sleep | Trazodone |
Sleep Rxs: TCAs such as amitriptyline, doxepin increases cardiac risk of | arrhythmais (obtain EKG prior to use) |
Sleep Rxs: ADDICTIVE med that should only be used short-term | benzodiazepines |
Sleep Rxs: Popular meds acting on benZo receptors | zolpidem + zalepon |
Sleep Rxs: only med per FDA approved for long term | eszopiclone |
Sleep Rxs: nonaddictive med because it works on melatonin receptors rather than GABA reeptors | ramelteon |
Narcolepsy Rxs: first-line treatment | modafinil |
Cataplexy Rx | vanlafaxine, fluoxetine, or atomoxetine |
Sleep Rxs: GHB can assist in sleep and reduce | cataplexy |
Brain embryology: Forebrain develops into | telencephalon -> cerebral hemispheres + diencephalon ->thalamus |
Brain embryology: Midbrain develops into | mesencephalon -> midbrain |
Brain embryology: Hindbrain develops into | metencephalon + myelencephalon -> cerebellum, pons, and medulla |
Brain embryology: NTD avoided with routine | sonogram + quadruple screening |
Brain embryology: NTD indicative when alpha fetoprotein levels are | elevated |
Neuro Embyology: NTD defect indicative with AFP levels are | increased |
Neuro Embyology: AFP levels when decreased are indicative of | down's syndrome |
Neuro Embyology: Meningcele is a herniation JUST of the | meninges |
Neuro Embyology: Myelomeningocele occurs with hernation of both | spinal cord + meninges |
Neuro Embyology: No brain tissue separating amniotic fluid in the brain is indicative of | anencephaly |
Neuro Embyology: Holoprosencephaly occurs when the hemispheres | of the brain fail to separate |
Neuro Embyology: Cleft palate together with cyclopia are indicative of | holoprosencephaly |
Neuro Embyology: Sonic Hedgehod Gene mutations will be seen in patients with | holoprosencephaly |
Neuro Embyology: Posterior fossa (hindbrain) abnormalities are seen in | Chiari Malformations |
Neuro Embyology: Chiari Malformations occurs when cerebellum herniates download through | forman magnum |
Neuro Embyology: Syringomyelia is an enlargement of the central | canal in spinal cord |
Neuro Embyology: Compression of the spinothalamic tract occurs and is referred to | syringomyelia |
Neuro Embyology: HALLMARK: Cape-like, bilateral loss of pain and temperature sensation are seen in | upper extremities |
Neuro Embyology: HALLMARK: Anterior horn damage produces | weakness and atrophy of hands |
Neuro Embyology: Spinal cord trauma over time can produce | syringomyelia |
Neuro Embyology: Herniation of cerebellar tonsils are seen in | Chiari I Malformation |
Neuro Embyology: BOTH herniation of cerebellar tonsils and vermis | Chiari II Malformation |
Neuro Embyology: Hydrocephalus can be seen in patients with | Chiari II Malformation |
Neuro Embyology: Stenosis of aqueduct caused by a Chiari II malformations along with which other abnormality | hydrocephalus |
Neuro Embyology: Thoracolumbar Myelomeningocele is associated with | Chiari II Malformation |
Neuro Embyology: Second Posterior Fossa malformation | Dandy-Walker Syndrome |
Neuro Embyology: enlarged posterior fossa is indicative of | Dandy-Walker syndrome |
Neuro Embyology: cerebellar vermis fails to develop is indicative of | Dandy-Walker syndrome |
Neuro Embyology: Dilation of the 4th ventricle is associated with hydrocephalus indicative of which diagnosis | Dandy-Walker syndrome |
Neuro Embyology: Brachial Apparatus is composed of how many tissues | six; 6 |
Neuro Embyology: Bracial Apparatus arch is derived from | Mesoderm |
Neuro Embyology: Bracial Apparatus clef is derived from the | Ectoderm |
Neuro Embyology: Bracial Apparatus pouch is derived from the | Endoderm |
Neuro Embyology: Bracial Apparatus Pouch #1 will give rise to | Middle ear cavity Eutachian tubes Mastoid air cells |
Neuro Embyology: Bracial Apparatus Pouch #2 will give rise to | epithelial lining of the tonsils |
Neuro Embyology: Bracial Apparatus Pouch #3 will give rise to | inferior parathyroid gland + thymus |
Neuro Embyology: Bracial Apparatus Pouch #4 will give rise to | superior parathyroid gland |
Neuro Embyology: DiGeorge Syndrome occurs when abnormal development | 3rd and 4th brachial pouches |
Neuro Embyology: Pt. with absent thymus and parathyroid glands | DiGeorge Syndrome |
Neuro Embyology: Hallmark Triad in DiGeorge Syndrome | Absent thymus + hypocalcemia + T-cell deficiency |
Neuro Embyology: Brachial Arches are | Mesoderm derivatives |
Neuro Embyology: Bracial Arch #1 produces CMN, which are | Cartilage + Muscle + Nerve |
Neuro Embyology: Treacher Collins Syndrome | Series of facial abnormalities |
Neuro Embryology: HALLMARK of Treacher Collins Syndrome includes | Abnormal mandible + malleus |
Neuro Embryology: Brachial Arch #2 produces S structures such as | Stapes + Styloid Process + Stylohyoid ligament + lesser horn of the hyoid |
Neuro Embryology: Hemispatial neglect indicates a lesion in the | Parietal lobe (non dominant) |
Neuro Embryology: Pt. neglects half of a side of their body as nonexistent | Hemispatial neglect |
Medical Term used for Crossed-Eyed pt. | Strabismus or tropia |
Golgi Apparatus modifies with amino acids | S.A.T. Serine, Asparagine, Theronine |
Astrocyte foot processes, basement membrane, capillary lumen are the components of the | basement membrane |
Midbrain is composed of | red nucleus, medial meniscus, superior colliculus, cerebral aqueduct, medical geniculate body, spinothalamic tract, corticobulbar tract, corticospinal tract, substantia nigra |
Inferior Colliculus is located in the | caudal midbrain |
Superior Colliculus is located in the | rostral midbrain |
CN located in the midbrain | Oculomotor and Trochlear Nerves |
Pons contains the | abducens (CNVI) nerve |
MLF is found in the | pons |
Medial Lemnicus travels done the | medial aspect of the pons |
Corticospinal tract is located in the | medial aspects of the pons |
Lateral part of pons is supplied by | AICA |
Cranial nerves will __________ as you go down the brainstem | increase in number |
Medulla medial aspect contains | hypoglossal nerve (CN XII), medial lemniscus, and meduallry pyramid |
Anterior Spinal Artery supplies the | medial aspect of the medulla |
PICA supplies the | lateral aspect of medulla |
Lateral aspect of medulla is composed of | vestibular (CN VIII) nuclei, inferior cerebellar peduncle, spinal trigeminal tract and nucleus and lateral spinothalamic tract |
Spinal Tracts: Dorsal Columns are composed of | fasiculus cutaneous and fasciculus gracilis |
Spinal Tracts: Dorsal Columns decussate in the medulla to then | ascend |
Spinal Tracts: Dorsal Columns become what | medial lemniscus |
Spinal Tracts: Medial Lemniscus goes into the | VPL of thalamus |
Spinal Tracts: VPL is located in the | thalamus |
Spinal Tracts: Dorsal Columns work on which side | contralateral |
VPL is the _____ port of the thalamus | sensory |
Spinothalamic tract decussates at the | anterior white commisure |
Spinothalmic tract ascends | contralateraly |
Which tract travels contralaterally in the brainstem | spinothalamic |
Information to the Corticospinal Tract comes from | internal capsule |
Spinothalamic tract ascends as Corticospinal Tract | desends |
Norepinephrine is increase in anxiety and | mania |
Norepinephrine is decreased in | depressionb |
Raphe Nucleus is the center of | serotonin |
In depression, serotonin is | decreased |
The levels of serotonin are decreased in | anxiety |
CPK, Adolase,and Myoglobin are | osteomarkers |
This condition prevents thymidine repair dimers from UV damages | xeroderma pigmentosa |
Dexamethasone Suppression Test | |
Dopamine levels in Schizophrenia | increased |
Dopamine levels in Depression | decreased |
Dopamine levels in Parkinson's | decreased |
Microglia HALLMARK seen in | HIV multi-nucleated giant cells |
Bitemporal Temporal Lobe location of | HSV encephalitis |
Decreased levels of Vitamin B6 lead to | seizures |
GABA is decreased n conditions such as anxiety and | huntington's |
The nucleus acubens houses neuroinhibitors such as | GABA |
Injury to axillary nerve results for damaged to the surgical | neck of the humerus |
P. Circumflex Humeral Artery supplies the | surgical neck of the humerus |
Perforin + Granzyme B is released as a result of | cytotoxic T cells and NK cells during bacterial infection |
Clathrin works in the (hint: starts with a C) | cytosol |
Process of receptor mediated endocytosis | clathrin is activated which activates adaptins |
Center of Huntington's disease | caudate + putamen |
Receptor locations: Tyrosine Kinase | plasma membrane |
Receptor locations: G-protein coupled | cytosol |
Receptor locations: steroids | cytoplasm |
Retro peritoneal Organs | S.A.D.P.U.C.K.E.R |
Destruction of Alpha 3 chain of Type IV Collage results in | Goodpasture Syndrome |
Hypotension, Tachycardia, and Cool Extremities are sx of | hypovolemic shock |
How to diagnose lactose intolerance | pH of stool |
MOA inhibit substance flow across cell membranes thus inhibiting primary active transport | PPI |
Zileuton, Montelukast, Zafirlukast are agents used to treat | asthma |
Transplant rejections primarily occur due to preformed | antibodies against antigens |
Langerhans Histocytosis HALLMARK | birbeck granules |
Tennis racket shaped cytoplasmic organelles | langerhans histocytosis |
Encapsulated bacteria | Even Some Pretty Nasty Killers Have Shiny Bodies |
The most injure organ in blunt trauma (not a foregut structure) | spleen |
Bilateral temporal visual deficit seen in | pituitary adenomas |
Payer Patches seen in | small intestine (ileum) |
Rx producing antibody against CD20 | rituximab |
Causative agent of barking cough | croup |
Meningiomas found in | parietal lobe |
Meningiomas produce lower limb sensory loss and | hemineglectp |
PICA Disorder | inability to consume food; pt consumes items that are non-food such as ice, hair, paper |
Rx for phobia | benzodiapine |
Agents in innate immunty | neutrophils, dendrites, macrophages, and complement |
Hot T-Bone Steak | IL-1 (fever), IL-2 (T-cells), IL-3 (bone marrow), IL-4 (IgE + IgG production), IL-5 (IgA + Eosinophils production) |
Active Immunity involves | b + t cells |
IFNs released by virus infected cells | IFN-alpha and IFN-beta |
Rx immunosuppreant inhibiting calcineurin, production of IL-22, and T-Cell | Cyclosporin |
HALLMARK in Crohn's Disease | transmural inflammation |
HALLMARK Ground Glass Apperance of lungs | NRDS (Neonatal Respiratory Distress Syndrome) |
HALLMARK Air Space and interstitial opacities in lungs | NRDS (Neonatal Respiratory Distress Syndrome) |
Antidote for acetominophen overdose | N-Acetylcystine |
N-Acetylcystine leaves disulfide bonds within | muscous glycoproteins |
Extension seen in obstructive lung disease | expiratory phase (FEV down) |
Asthmatic antigen cross linking IgE on | pre-sensitized mast cells |
Levels in COPD | FEV down; FVC same |
Restrictive Lung Disease leads to decreased | TLC; normal FEV1:FVC ratio |
Side effect of Tamoxifen | endometrial hyperplasia |
Disorder in which pt. complains of symptoms that are not even there | somatization |
Thrombocytopenia can be caused by this agent (hint: thrombolytic) | heparin |
Thrombocytopenia can be caused by this agent (hint: antibiotic) | beta lactam antibiotics |
Thrombocytopenia can be caused by this agent (hint: anti bacterial) | vancomycin |
Splenomegaly seen in this kissing disease | EBV |
Thrombocytopenia can be caused by this agent (hint: used to treat gram + bacteria) | linezolid |
Rx used in treatment of CMV | foscarnet |
Foscarnet does not require | intracelullar activation |
Arsenie Poisioning is treated with this agent | dimercaprol |
Poision Ivy is mostly encountered in | hikers, forests, and wooded areas |
Neisseria Gonnorhea will elevate | neutrophils (think N in neiserria) |
Rx used treat general protozas | metrodiazole |
Rx used treat trophozoites | metrodiazole + tinidazole |
Rx used treat cysts | iodoquinol + paromomycin |
Rx swish and swallow method us used with nyacin to treat | oral candidasis |
Erythrocytosis in men should be | <52% |
Erythrocytosis in women should be | <48% |
Activation of Leukotriene C4 is found in | asthma |
HALLMARK for asthma histology | charcot-leyden crystals |
Pharm: direct acting is an | agonist |
Pharm: inhibition of action is a | anatagonist |
Organophosphate Poisioning is seen primarily in | farmers |
Adenosine Deaminase Deficency results in this condition | SCID |
A-1 Adrenergic causing adverse reaction of orthostatic hypotension | terazosin + doxazosin |
Raymonds phenomenon leads to | cerebral edema |
Asprin given to children younger than the age of 12 will results in | raymond's phenomenon |
Disease arsing from deficiency in mannose-6-phosphate | i-cell |
Sx clouded cornea, restruct joint movement, and course facial features seen in | i-cell disease |
This bacteria's polysaccharies capsule gives it it's virulence | step. pneumo |
DNA mismatch repair will cause | hereditary nonpolyposis colorectal cancer |
Rx agent used for motion sickness | scopolamine |
HALLMARK fishy odor with thin discharge indicative of | bacterial vaginosis |
HALLMARK frothy odor with green/white discharge indicative of | trichamonasis |
HALLMARK cottage cheese discharge indicative of | candidasis vaginosis |
Water Deprivation Test is done to pt. who is suspected to have | diabetes insipidus |
Desmopressin (ADH) Challenge will indicate whether the dx is | central or nephrogenic |
This syndrome's onset may be due to an adverse reaction to steroids | cushings syndrome |
Amyloidosis stains congo | red |
HALLMARK apple green bigeringate | amyloidosis |
HALLMARK histology showing rouleaux (poker chips like) | multiple myeloma |
Adverse Reactions: Anti-hypertensive Nifedipine | reflex tachycardia; useful for pt. with bradycardia |
Adverse Reactions: Anti-hypertensive Beta Blocker | prolong PR interval |
Olgiodendrocyte destruction seen in | multiple sclerosis |
Retro peritoneal organ most injured in car accidents | pancreas |
Abnormalty seen in hereditary spherocytosis | RBC membrane cytoskeleton |
HALLMARK Interstitial Fibrosis | honeycomb lung on x-ray |
Honeycomb lung on x-ray is indicative of | intersistial fibrosis |
Rx acetylcysteine decreases levels of | mucous |
1st line indicator in diabetic nephropathy | albumin |
Generalized Anxiety Disorders are treated with | buspirone |
Crackles on lung auscultation mostly heard when pt has | pulmonary edema |
HALLMARK Asbestosis | dumb bells appearance |
Asbestosis histology will show | enlongated structures with clubbed ends |
Myocardial Infarction will produce | prostagladins |
Prostaglandins produced in MI will | inhibit platelet aggregation |
Cytochrome oxidase inducers include | |
Rapid correction of HYPONATUREMIA will lead to this condition | central pontine myelinolysis |
MRI shows increased signal intensity to the pons indicating | rapid correcton of hyponatermia |
Sx of central pontine myelinolysis | paralysis, dysarthria, dysphagia, diplopia, loss of consciousness |
Low sodium levels need to be correct very | slowly |
Retinoic acid (vitmain A derivative) alters | HOX gene expression |
Which antibiotics are potentially teratogenic | F.A.T.; Fluroquinolones, Aminoglycosides, and Tertatogens |
Rx teratogenic in pregnant women | MTX, Statins, Warfarin, Isorebinoin, DES, Thalidomide |
Limb buds begin to form at week | 21 |
Fetal movement begins at week | 8 |
By which week does the mother begin to experience fetal movement | 8 |
Which vitamin should not be supplemented in large amounts during pregnancy | vitamin A; alters HOX gene expression |
Nuclear Localization Signals: Amino acids | rich in L.A.P: lysine, arginine and proline |
Nuclear Localization Signals: Essential component of protein bound for or residing in the nucleus | histones |
Where are histones located | nucleus |
Nuclear Localization Signals: Nuclear pores recognize these signals and transport proteins into the | nucleus via ATPase |
ATPase is the signal of transportation of proteins and into the | nucleus |
A single AA mutation may prevent | nuclear transport |
A deficiency leading to I-Cell Disease | mannose phophorylation |
Mannose phosphorylation secretion out of | cell instead of into lysosomes for degradation |
I-Cell Disease leads to death by the childhood age of | 8 |
Chaperones assist in folding and transport of polypeptides in the | ER and Golgi, etc |
Stress Chaperones rescue shock-stressed proteins from | misfolding |
If folding is not successful, the chaperones facilitate | degradation of damaged protein |
Degradation of damaged proteins involves | ubiquitin |
This is also known as a heat shock protein | ubiquitin |
Peroxisomes provide beta oxidation of very long chain | fatty and branched chain fatty acids |
Which cyclin-CDK complexes assist in the progression from G1 phase to S phase | Cyclin D and Cycline E; CDK 4 + CDK 2, respectively |
Which cyclin-CDK complexes assist in the progression from G2 phase to M phase | Cycline A and Cycline B; CDK 2 + CDK 1, respectively |
What molecule does the Golgi add to proteins in order to direct the proteins to the lysosomes | |
What are the different methods that a cell uses to break down proteins | |
What is the most common cause of intellectual disability in infants | fetal alcohol syndrome |
Structural component of connective tissue L.E.F. (leukocytes, endothelium, fibroblasts) | Vimentin |
Desmin is the intermediate filament of structural component | muscle (smooth, skeletal, and heart) |
Cytokeratin is the intermediate filament of structural component | epithelial cells |
Glial fibrillary acid proteins is the intermediate filament of structural component | astrocytes, schwann cells, other neuroglia |
Peripherin is the intermediate filament of structural component | neurons |
Neurofilaments is the intermediate filament of structural component | axons within neurons |
Nuclear lamins is the intermediate filament of structural component | nuclear envelope and DNA within |
Which drugs act on microtubules (the microtuble growth voiding pure chemicals) | thiabendazole + mebendazole + griseofulvin + vincrestine + pacixtel + vinblastine |
What are the defects in Kartagner Syndrome aka immotile ciliary syndrome | primary ciliary dyskinesia |
During what week of fetal development does organogensis take place | |
What molecule provide the structural framework for DNA and nuclear envelope | nuclear lamins |
The first step in the signaling cascade that it initiated by tyrosine kinase receptors is | autophosphorylation |
What type of protein is PDGF | single-pass transmembrane protein |
Insulin and Insulin Growth Factor-1 receptors | 2 alpha subunits and 2b subunits |
Which arachiodonic acid product causes: increased bronchial tone | prostaglandins |
Which arachiodonic acid product causes: decreased bronchial tone | PGF-12 |
Which arachiodonic acid product causes: increased platelet aggregation | thromboxane |
Which arachiodonic acid product causes: decreased platelet aggregation | PG-12 |
Which arachiodonic acid product causes: increased uterine tone | prostaglandins |
Which arachiodonic acid product causes: decreased uterine tone | PG-12 |
Which arachiodonic acid product causes: increased vascular tone | thromboxane |
Which arachiodonic acid product causes: decreased vascular tone | PG-12 |
What are the two most abundant substances in the plasma membrane | phospholipids and cholesterol |
What drugs act on the arachidonic acid pathway | Cycloxygenase (NSAIDs) |
What are the stages that an embroy goes through between conception and the development of an inner cell mass | cleavage, blastula, and gastrulation stages |
The blastula eventually organizes itself into two layers | inner cell mass and outer trophoblast |
Trophoblast will become the | placenta |
Which cell types are constantly regenerating themselves due to an absence of the G0 phase and a short G1 phase | GI, bone marrow, hair follicles, nails |
BCL-2 is major anti-apoptotic regulator of | mitochondrial permeability |
Any DNA damage or apoptotic sign will activate | Bax (pro-apoptotic) |
Bax creates channels in | mitochondrial membrane |
Cytochrome C moves from the | mitochondria and into cytosol |
Cytochrome C activates | caspases |
The primary initator of apoptosis | caspases |
Name death receptors | TNF-a + Fas ligand |
Cytotoxic T-Cells recognize | foreign or infected cells |
Cytotoxic T-Cells release which agents | perforin and granenzyme B |
Mechanism of Cell Injury: influx of calcium will increase | mitochondrial permeability and activate phospholipases, proteases, endonucleases and ATPase |
Mechanism of Cell Injury: accumulation of oxygen-derived free radicals | cell damage through membrane lipid peroxidation, protein modification and DNA breakage |
Radiation Exposure may cause | cell injury; DNA breakage, protein modification |
What histology features may be seen in apoptotic liver cells | infiltrate only if cause is alcohol is steatohepatitis |
Apoptotic liver cells under the microscope will show | histologic changes including steatosis, inflammation, ballooned hepatocytes, Mallory–Denk bodies, apoptotic hepatocytes, and fibrosis or cirrhosis |
What damaging events can cause irreversible cell injury | mitochondria swell, lysosomes swell, damage to plasma membrane and lysosomal membranes leads to enzyme leakage |
What substances do cytotoxic T cells and NK cells use to induce apoptosis in the cells infected with a virus | perforin and granenzyme B |
Redox reaction may cause | cell injury; DNA breakage, protein modification |
What cellular enzymes are responsible for handling oxygen free radicals | superoxide dismutase |
Transition metals may cause | cell injury; DNA breakage, protein modification |
Nitric Oxide may cause | cell injury; DNA breakage, protein modification |
Leukocyte Oxidative Burst may cause | cell injury; DNA breakage, protein modification |
Reperfusion injury may cause | cell injury; DNA breakage, protein modification |
Acute inflammation: release of mediators | neutrophils, histamine, bradykinin, serotonin |
Acute inflammation: mediators in vasodilation | prostagladins and nitrix oxide |
Tissue remodeling by metalloproteinses contains | C5a |
Polymyalgia rhematica laboratory indication | elevated ESR |
C-Reactive Protein in the acute phase is synthesized by the | liver |
Oposonization of bacteria and activation of complement is done by | C-Reactive Protein |
Temporal arteritis laboratory indication | elevated ESR |
Acute inflammation: mediators increased vascular permeability fluid exudation | bradykinin, serotonin, histamin, C3a + C5a, Leukotrienes, PAF, oxygen free radicals |
Can C-Reative Protein be lowered by smoking cessation | yes |
Maliganancy laboratory indication | elevated ESR |
Elevations in this marker is a strong predictor or MI, Stoke, PAD, and sudden cardiac death | C-Reactive Protein |
True o False: C-Reactive Protein can be lowered by exercise/weight loss | true |
Which metals are known to faciltate the generation of oxygen free radicals | metallprotienases |
Which tumor suppressor proteins prevent the progression of the cell in S phase | p53 and Rb |
Cutaneous wound healing: week-months | collagen production (Type III and Type I) |
Cutaneous wound healing: 0-3 hours | hemorrhage and clotting |
Cutaneous wound healing: 1-3 days | macrophage infiltration, granulation tissue, epithelization |
Cutaneous wound healing: 12-24 hours | acute inflammation (PMNs) |
Infection, inflammation (e.g. osteomyelitis) laboratory indication | elevated ESR |
Ehlers-Danlos Syndrome sx (3-D pneumonic) | Type III collagen deficiency |
Diseases activity in RA and SLE | elevated ESR |
Which AA are found in large concentrations in collagen | proline + lysine |
What is the role of Vitamin C in collagen | hydroxylation of proline and lysine |
Sx hyperflexible koints, archnodactyly, aortic dissection, lens dislocation are all indicative of | ehlers danlos |
Sx hereditary nephritis, cataracts, sensorineural hearing loss | |
What is the underlying dysfunction in Chediak-Higashi syndrome | Defect in lysosomal trafficking regulator gene (LYST); Microtubule dysfunction in phagosome-lysosome fusion; |
Chediak-Higashi Syndrome is an | autosomal recessive |
How does having a high cholesterol content in the plasma membrane affect its function | separate the phospholipids so that the fatty acid chains can't come together and cyrstallize. |
A man working out at the gym building muscle would be considered | hypertrophy |
What can happen to the cells of the lower espohagus in response to chronic acid reflux | metaplasia of simple squamous to columnar epithelium leading to GERD or Barret's Esophagus |
What is lipofuscin granule | aging or "wear-and-tear" pigments, found in the liver, kidney, heart muscle, retina, adrenals, nerve cells, and ganglion cells |
Four major dopaminergic pathways | mesocortical pathway |
Four major dopaminergic pathways | mesolimbic pathway |
Four major dopaminergic pathways | nigrostriatial pathway |
Four major dopaminergic pathways | tuberoinfundibular pathway |
Which nervous system cell matches the follow description: fried egg under histology staining | olgiodendrites |
Which nervous system cell matches the follow description: form multinucleated giant cells in the CNS when infected with HIV | mircroglia |
Which nervous system cell matches the follow description: myelinates mutiple CNS axons | |
Which nervous system cell matches the follow description: myelinates one PNS axon | schwann cells |
Which nervous system cell matches the follow description: damaged in Guillain-Barre syndrome | schwann cells |
Which nervous system cell matches the follow description: damaged in multiple sclerosis | ogliodendrites |
Which nervous system cell matches the follow description: macrophase of the CNS | microglia |
Which nervous system cell matches the follow description: cells of the blood-brain barrier | |
What is the main inhibitory neurotransmitter of the CNS | GABA |
In which diseases is GABA altered | huntington's and anxiet |
What organelle becomes hypertrophied in hepatocytes with chronic phenobarbital use? | smooth endoplasmic reticulum (SER) |
What enzyme mitigates the aging effects of cellular division by maintaining chromosomal right | telomerase |
What is currently known as the most effective way of prolonging life span | |
Which area of the hypothalamus regulate the autonomic nervous system | paraventricular nucleus |
Which neurotransmitter have altered levels in anxiety disorder | GABA |
Which collagen is typically deficient in Ehlers-Danlos syndrome | Type III |
Which collagen is typically deficient in osteogensis imperfecta | Type I |
Which drug is used to shorten Stage N3 sleep | imipramine |
What is the sleep pattern in a patient with narcolepsy | |
In what stage of sleep are night terrors found | Stage N3 |
What arachidonic acid product has actions that oppose that of prostacyclin | COX-1 + COX-2 |
What brain structure is responsible for extraocular movements during REM sleep | paramedian pontine reticular formation/conjugate gaze center |
What two nerves are tested with the gag reflax | vagus + glossopharyngeal |
Unilateral facial drooping involving the forehead | bell's palsy; LMN facial nerve (CN VII) palsy; UMN lesions spare the forehead |
What organelle and cytochrome are particularly important in intrinsic apoptosis | mitochondria + cytochrome C |
During what sleep stage would a man have variable blood pressure, penile/clitoral tumescence, and variable EEG | REM |
A 19-year-old patient presents with a furnuncle on his philtrum, and the cavernous sinus becomes infected. What neurological deficits might you see in this patient? | |
Which areas of the hypothalamus regulate the autonomic nervous system? | Anterior hypothalamus : Parasympathetic Posterior: Sympathetic |
Cranial nerve 1 (Olfactory) Site of exit from the skull? Function? Lesions? How to test the function? | Cribiform Plate Smell Cribiform plate fracture or Kallmann syndrome Smell something |
Cranial nerve 2 (Optic) Site of exit from the skull? Function? How to test the function? | Optic canal; [Sight] [Smell] [Eye chart] |
Cranial nerve 3 (Occulomotor) Site of exit from the skull? Function? Lesions? How to test the function? | Superior orbital fissure Parasympathetic: Cillary and sphincter muscles Innv. MR, SR, IR, IO (Extraoccular m.) Transtentorial/Uncal Herniation, Diabetes, Weber syndrome Follow my finger, pupillary light reflex, and convergence |
Describe Weger Syndrome | Form of stroke that causes occlusion of PCA. Causes contralateral hemiparesis and CN 3 palsy |
Cranial nerve 4 (Trochlear) Site of exit from the skull? Function? Lesions? How to test the function? | Superior orbital fissure Innv. SO Head trauma Move eye by following finger |
Cranial nerve 5 (Trigeminal)-V1 Site of exit from the skull? Function? Lesions? How to test the function? | V1 (Opthalmic): Superior orbital fissure Sensory for Medial nose and forehead Path: Trigeminal neuralgia Test: Facial sensation |
Cranial nerve 5-V3 (Mandibular) Site of exit from the skull? Function? Lesions? How to test the function? | Foramen ovale Motor fxn. of muscle of mastication (open and close jaw) Sensory: lower lip, lateral face, lower border of mandible Lesion: Jaw will deviate towards the side of lesion when opening Test: facial sensation, open jaw |
Which muscle opens the jaw | Lateral Pterygoid |
Which muscle closes the jaw | Masseter, Temporalis, Medial pterygoi |
Cranial nerve 6 (Abducens) Site of exit from the skull? Function? Lesions? How to test the function? | Superior orbital fissure Innv. lateral rectus Medial inferior pontine syndrome (contralateral hemiparesis and loss of tactile and vibrations + lesion of CN 6) Test with follow my finger |
Cranial nerve 7 (Facial) Site of exit from the skull? Function? Lesions? How to test the function? | Internal acoustic meatus Parasymp: Lacrimal, submandibular, and sublingual glands Innv. Facial expression, stapedius, stylohyoid, posterior belly of digastric m. Taste anterior 2/3 of tongue Bells Palsy (includes muscle of forehead) Test:Wrink |
Name the 5 branches of the Facial Nerve (CN VII) | Temporal Zygomatic Buccal Marginal mandibular Cervical |
Cranial nerve 8 (Vesitbulacocchlear) Site of exit from the skull? Function? Lesions? How to test the function? | Internal acoustic meatus Equilibrium and Hearing Acoustic schwannoma (vertigo, nystagmus, nausea and vomiting) Hearing and nystagmus |
Cranial nerve 9 (Glossopharynageal) Site of exit from the skull? Function? Lesions? How to test the function? | Jugular foramen Parasym: Parotid gland, stylopharyngeus m Sensory: pharynx, middle ear, auditory tube, carotid body and sinus, external ear, posterior 1/3 of tongue Lesions: PICA infarct Test: Gag relfex |
Cranial nerve 10 (Vagus) Site of exit from the skull? Function? Lesions? How to test the function? | Jugular foramen Symp: body viscera, laryngeal & pharyngeal m. Sensory: Trachea, External ear, viscera of esophagus, epiglottis Lesion: Thyroidectomy, PICA infarct Test: Gag reflex and saying ahh (elevates palate) Uvula deviates away from lesion |
Cranial nerve 11 (Accessory) Site of exit from the skull? Function? Lesions? How to test the function? | Jugular foramen Innv. SCM and Trapezius M. Lesion: PICA infarct Test: Turn head and shrug shoulders |
Cranial nerve 12 (Hypoglossal) Site of exit from the skull? Function? Lesions? How to test the function? | Hypoglossal canal Intrinsic tongue muscles Lesions: Anterior spinal a. infarct Tongue protudes towards lesion Test: Lalalala |
Corneal Reflex How to test? Normal reflex Afferent: Efferent: | Touch cornea with cotton N: Blink Afferent: V1 (opthalmic) Efferent: CN 7 (temporal branch)-closes eye (orbicularis oculi m.) |
Lacrimal Reflex How to test? Normal reflex Afferent: Efferent: | Touch cornea with cotton N: Lacrimate Afferent: V1 (opthalmic) Efferent: CN 7 (temporal branch) |
Jaw Jerk Reflex How to test? Normal reflex Afferent: Efferent: | Tap on chin Normal (no response) Afferent: V3 (Mandibular) Efferent: V3 (Masseter) UMN lesion if different |
Pupillary Reflex How to test? Normal reflex Afferent: Efferent: | Shine light in eye Normal: Pupillary constriction Afferent: Optic nerve Efferent: Parasymp CN 3 |
Gag Reflex How to test? Normal reflex Afferent: Efferent: | Stick tongue depressor in mouth and say ahhh Normal: Gags Afferent: CN 9 Efferent: CN 10 |
Midbrain contains which CN nuclei? | CN 3 and 4 |
Pons contains which CN nuclei? | CN 5-8 |
Medulla contains which CN nuclei | CN 9, 10, 12 |
Spinal cord contains which CN nuclei? | CN 11 |
What does the Pineal body contain | melatonin secretion, circadian rhythm |
What does the Superior colliculus contain | conjugate vertical gaze center |
What does the Inferior colliculus contain | auditory information |
What nerves run through Cavernous sinus | CN 3, 4, 6, branches (V1 and V2) of CN 5 |
What are other structures that run through Cavernous Sinus | Internal Carotid a optic chiasm pituitary gland sphenoidal sinuses |
The reticular activating system include which areas | Reticular formation Mesencephalic nucleus Thalamic intralaminar nucleus Dorsal hypothalamus Tegmentum |
The reticular activating system function | arousal and sleep awakening |
The Vagal nuclei are | nucleus solitarius nucleus ambiguus dorsal motor nucleus |
Nucleus solitarius | Nucleus solitarius Visceral sensory info: Taste Baroreceptors gut distension (CN 7,9,10) |
Nucleus Ambiguus | Motor innv. Pharynx Larynx Upper Esophagus (CN 9,10,11) |
Dorsal Motor Nucleus | Autonomic (Parasymp) info: Heart Lungs Upper GI (CN 10) |
Which sympathetic chain starts from what spinal cord segment and ends at what segment | T1-L3 |
Horner Syndrome will cause | sympathetic denervation of the face |
Describe Horner syndrome symptoms | Ptosis (drooping of eyelid) Anhidrosis (absence of sweating) and flushing (rubor) of affected side Miosis (constriction of pupils) |
What is caused by Horner syndrome | Spinal cord lesion above T1 assc.(superior cervical ganglion). Pancoast tumor (pre-ganglionic) Brown-Sequard syndrome Late stage syringomyelia |
What are the muscles of mastication | Open Jaw: Temporalis, medial pterygoid cLose Jaw: Lateral pterygoid Masseter m. |
Unilateral facial drooping involving the forehead | Bell's Palsy |
Sx Ptosis, miosis and anhidrosis are indicative of | Horner Syndrome |
What G protein classes do alpha receptors stimulate? | • α1= Gq • α2 =Gi |
What G protein classes do beta receptors stimulate? | • β1 =Gs • β2 =Gs |
What G protein classes do muscarinic receptors stimulate? | • M1 =Gq • M2 =Gi • M3 =Gq |
What G protein classes do dopaminergic receptors stimulate? | D2= Gs |
What is the treatment for choriocarcinoma | methotrexate vincristine vinblastine |
What is the treatment for AML | cytarabine (arabinofuranosyl) |
What is the treatment for CML | imatinib |
Which Rx prevents breast cancer | tamoxifen |
What is the treatment for breast cancer | etopside, bleomycin, cisplatin, ifosfamide |
What is the rate-limiting enzyme in purine synthesis? | Purine: PPRP (phosphoribosyl pyrophosphate synthetase II) |
What is the rate-limiting enzyme in pyrimidine synthesis? | Pyrimidine: Carbamoyl phosphate synthetase II |
Case-Control study | Compares a group of people with the disease vs those without the disease Looks for prior exposure or risk factor Retrospective/Observational |
Cohort Study | Compares a group with a given exposure or risk factor to a group without such exposure Can be prospective/retrospective Relative risk |
Cross-sectional study | Collects data from a group of people to assess frequency of disease (and related risk factor) at a particular point in time Disease prevalence Can't estimate causality |
Twin Concordance Study | Compares the frequency with which both monozygotic twins or both dizygotic twins develop the same disease |
In which phase of meiosis is a primary oocyte arrested until just prior to ovulation? | Prior ovulation: Prophase of meiosis I |
In performing a lumbar puncture to obtain a sample of CSF, what structures are pierced, starting with the most exterior? | Skin Subcutaneous fat Supraspinal ligament Interspinal ligament Ligamentum flavum Epidural space Dura Matter Subdural space Arachnoid matter Subarachnoid space |
What medications are used in the treatment of pulmonary hypertension | Endothelin receptor antagonists (Bosetan) MOA: Competitive antagonize endothelin-1 receptor PDE-5 inhibitor (Sildenafil) MOA: Inhibit cGMP PDE5 and prolong vasodilatory effect of NO Prostacyclin analogs (epoprostenol & iloprost) MOA: Prostacyclin |
Does the notochord become the neural tube | No, Notochord is derived from mesoderm; Notochord becomes nucleus pulposus of IV disk |
From where does the amygdala receive inputs | Inputs: Limbic cortex Neocortex of parietal, temporal, and occipital lobe |
To where does the amygdala send output | Output: Hypothalamus, Thalamus, Septum, Hippocampus |
Narcoleptic sleep episodes begin with what stage of sleep | Narcolepsy sleep episodes start at REM sleep |
What are the different stages of normal sleep | Awake: Beta waves Awake but relaxed: alpha waves Stage 1: Light sleep (Theta waves) Stage 2: Deeper sleep, Bruxism (Sleep spindles and K complexes) Stage 3: Deepest non-REM sleep (delta-slow wave) |
Which T cell type regulates the humoral response | T-Helper Cells 2 -Augment humoral (Plasma cell) response Generate IL-4 and IL-5 |
What is a reason why a woman might have primary amenorrhea (hint: congential) | turner syndrome |
What is a syringomyelia | Cystic cavity within spinal cord (C8-T1) Crossing anterior spinal commissural fibers are damaged |
What symptoms are commonly seen in patients with syringomyelia | Cape-like bilateral loss of pain and temp sensation in upper extremities (fine touch preserved) |
What type of skin cancer is associated with arsenic exposure in coal miners | Squamous cell carcinoma |
Which one will be able to increase the blood pressure of a hypotensive patient | Phenylephrine |
What are the steps in the conversion of vitamin D to its active form in the body | D3 from sun exposure in skin. D2 ingested from plants. Both converted to 25-OH in liver and to 1,25-(OH)2 (active form) in kidney |
By what mechanism does vitamin D help prevent osteoporosis | Increases absorption of dietary Ca2+ and Phosphate Increases bone resorption -> Increased Ca2+ and Phosphate |
What landmarks are used when placing an internal jugular central venous catheter | Between the medial and lateral heads of the sternocleidomastoid muscle and lateral to the carotid artery in most cases |
What is a reason why a woman might have primary amenorrhea? (hint:hymen) | imperfortate hymen |
What is a reason why a woman might have primary amenorrhea? (hint: congential) | mullerian delayed agenesis |
What can occur if a MAO inhibitor (MAOI) is added to an SSRI | Serotonin syndrome |
Rx for Serotonin Syndrome | Treatment: Stop serotonergic drugs Benzodiazepine Supportive care Cyproheptadine (5-HT2 receptor antagonist) |
Clavulanic acid, sulbactam, and tazobactam aid penicillins in their activity against bacteria through what mechanism | Beta-lactamase inhibitor |
What is the most common benign tumor of the salivary gland? | pleomorphic adenoma |
What is a Warthin's tumor | Warthin tumor: (papillary cystadenoma lymphomatosum) is a benign cystic tumor with germinal centers |
What cranial nerve goes through the parotid gland | CN VII goes through parotid gland |
Membranous glomerular diseases involve thickening of what structure | Basement membrane Membranous nephropathy |
Which bacteria are most commonly responsible for sialadenitis | Bacteria: Staph a. and Viridans strep |
What condition most commonly predisposes a patient to sialadenitis | Predisposes: sialolithiasis (stone obstruction of salivary gland) |
What important secretory products are secreted from the following cells of the GI tract? (hint: G cells) | Gastrin |
What important secretory products are secreted from the following cells of the GI tract (hint: I cells) | Cholecystokinin (CCK) |
What important secretory products are secreted from the following cells of the GI tract? (hint: S cells) | Secretin |
What important secretory products are secreted from the following cells of the GI tract? (hint: D cells) | Somatostatin |
What important secretory products are secreted from the following cells of the GI tract (hint: pariteal cells) | Gatric acid and IF |
What important secretory products are secreted from the following cells of the GI tract (hint: chief cells) | Pepsinogen |
What is the antidote for warfarin anticoagulation or warfarin overdose | Warfarin overdose: Vitamin K, plasma (if active bleeding) |
What is the antidote for heparin overdose | Heparin overdose: Protamine sulfate |
Most common cause of neural tube defects | Folate Deficiency |
Most common cause of congenital malformations in the US | Alcohol |
Most common cause of congenital mental retardation in the US | Fetal Alcohol Syndrome |
Sx Hyperflexible joints, arachnodactyly, aortic dissection, lens dislocation | Marfan's Syndrome |
Sx Hereditary nephritis, cataracts, sensorineural hearing loss | Alport Syndrome |
Sx Ptosis, miosis, anhidrosis | Horner's Syndrome |
Amyloid deposits in gray matter of the brain | Senile plaques in Alzheimer's Disease |
HALLMARK Drooling farmer | Organophosphate poisoning |
Sx Inability to breastfeed, amenorrhea, cold intolerance | Sheehan's Syndrome |
Sx Infertility, galactorrhea, bitemporal hemianopsia. | Prolactinoma |
Most common causes of Cushing syndrome (4) | 1) Exogenous Steroids 2) Ectopic ACTH--small cell lung cancer 3) Cushing Disease--pituitary tumor 4) Adrenal Adenoma |
Most common tumor of the adrenal gland | Adrenal Adenoma |
Most common tumor of the adrenal medulla (in adults) | Pheochromocytoma |
Most common tumor of the adrenal medulla (in kids) | Neuroblastoma |
Most common cause of primary hyperaldosteronism | Adrenal Adenoma |
Medical treatment for hyperaldosteronism | Spironolactone or Eplerenone |
Pheochromocytoma, medullary thyroid cancer, and hyperparathyroidism | MEN 2A |
Pheochromocytoma, medullary thyroid cancer, and mucosal neuromas | MEN 2B |
Adrenal disease associated with skin hyperpigmentation | Addison's Disease |
HTN, hypokalemia, metabolic acidosis | Conn Syndrome |
Most common thyroid cancer | Papillary Carcinoma |
Cold intolerance | Hypothyroidism |
Enlarged thyroid cells with ground-glass nuclei | Papillary Carcinoma of the thyroid |
Most common infections seen in chronic granulomatous disease | Bugs producing catalase: Candida Aspergillus Staph aureus Klebsiella E. coli |
Eczema, recurrent URI, high serum IgE | Hyper-IgE Syndrome aka Job's Syndrome |
Large lysosomal vesicles in phagocytes | Chediak-Higashi Syndrome |
Dark purple nodules on the skin in an HIV infected patient | Kaposi's Sarcoma |
Large cells with owl's eye inclusions | CMW |
Treatment of CMV | Ganciclovir |
Most common opportunistic infection in HIV patients | PCP |
Drug used to prevent Pneumocystis pneumonia | TMP-SMX |
Dysphagia, glossitis, and Fe deficiency anemia | Plummer-Vinson Syndrome |
Hematemesis with retching | Mallory-Weiss Tear |
Specialized columnar epithelium seen in a biopsy from the distal esophagus | Barrett Esophagus |
Biopsy of a patient with esophagitis reveals large pink intranuclear inclusions and host cell chromatin that is pushed to the edge of the nucleus | HSV |
Biopsy of a patient with esophagitis reveals enlarged cells, intranuclear and cytoplasmy inclusions, and a clear perinuclear halo | CMV |
An esophageal biopsy reveals lack of ganglion cells between the inner and outer muscular layers | Achalasia |
Biopsy of a mass in the parotid gland reveals a double layer of columnar epithelial cells resting on a dense lymphoid stroma | Warthin's Tumor |
Protrusion of the mucosa in the upper esophagus | Esophageal web |
Outpouching of all layers of the esophagus found just above the LES | Epiphrenic Diverticulum |
Basal cell hyperplasia, eosinophilia, and elongation of the lamina propria papilla seen in biopsy of the esophagus | Chronic GERD |
Goblet cells seen in the distal esophagus | Barrett Esophagus |
A PAS stain on a biopsy obtained from a patient with esophagitis reveals hyphate organisms | Candida |
Esophageal pouch found in the upper esophagus | Zenker Diverticulum |
Stomach biopsy reveals neutrophils above the BM, loss of surface epithelium, and fibrin-containing purulent exudate | Acute Gastritis |
Stomach biopsy reveals lymphoid aggregates in the lamina propria, columnar absorptive cells, and atrophy of glandular structures | Chronic Gastritis |
Diffuse thickening of gastric folds, elevated serum gastrin levels, biopsy reveals glandular hyperplasia without foveolar hyperplasia | Zollinger-Ellison Syndrome |
Mucin-filled cell with a peripheral nucleus | Signet Ring cells |
Most common type of stomach cancer | Adenocarcinoma |
Ovarian metastases from gastric cancer | Krukenberg tumor |
Gastric ulcerations and high gastrin levels | Zollinger-Ellison Syndrome |
Acute gastric ulcer associated with elevated ICP or head trauma | Cushing Ulcer |
Acute gastritis ulcer associated with severe burns | Curling Ulcer |
Painless jaundice | Pancreatic Cancer |
Most common cause of acute pancreatitis | Gallstones & alcohol |
Most common cause of chronic pancreatitis | Alcohol |
Small intestinal mucosa laden with distended macrophages in the lamina propria that are filled with PAS+ granules and rod-shaped bacilli seen by electron microscopy | Whipple's Disease |
Gluten Sensitivity | Celiac Sprue |
Total or subtotal atrophy of the small bowel villi, plasma cells, and lymphocytic infiltration into the lamina propria and epithelium, and hyperplasea/elongation of the crypts | Celiac Sprue |
Weight loss, diarrhea, arthritis, fever, adenopathy and hyperpigmentation | Whipple's Disease |
Anti-transglutaminase/anti-gliadin/anti-endomysial antibodies | Celiac Disease |
Vitamin given to pregnant women to prevent NT defects | Folate |
Newborn with chronic diarrhea, failure to thrive, and chronic Candida | SCID |
TG accumulation in hepatocytes | Fatty Liver |
Eosinophillic inclusion in the cytoplasm of hepatocytes | Mallory Bodies from Alcoholic Hepatitis |
Cancer closely linked to cirrhosis | Hepatocellular Carcinoma |
Severe hyperbilirubinemia in a neonate | Crigler-Nagir, Type I |
Mild, benign hyperbilirubinemia | Gilbert's or Crigler-Nagir, Type 2 |
Hepatomegaly, abdo pain, ascites | Budd-Chiari Syndrome |
Green/yellow corneal deposits | Wilson's Disease |
Low serum ceruloplasmin | Wilson's Disease |
Cirrhosis, diabetes, and hyperpigmentation | Hemochromatosis |
Preferred anticoagulant for immediate anticoagulation | Heparin |
Preferred anticoagulant for long-term anticoagulation | Warfarin |
Preferred antocoagulant during pregnancy | Heparin |
A boy with self-mutilating behavior, mental retardation, and gout | Lesch-Nyan Syndrome |
Elevated Uric Acid Levels (4) | Gout Lesch-Nyhan Tumor Lysis Loop or Thiazide Diuretics |
Bluish-colored lines on the gingivae | Lead Poisoning: Berton's Lead Lines |
Causes of hypochromic, microcytic anemia | Fe Deficiency Thalassemia Lead Poisoning |
Hypersegmented neutrophils | Megaloblastic Anemia: B12 or Folate Deficiency |
Skull x-ray shows a "hair-on-end" appearance | Marrow hyperplasia: Sickle-Cell Anemia or B-Thalassemia |
Basophilic Stippling of RBCs | Lead Poisoning |
Painful cyanosis of the fingers and toes, with hemolytic anemia | Cold agglutinins |
Red urine in the morning, and fragile RBCs | Paroxysmal Nocturnal Hemoglobinuria |
Basophilic nuclear remnants in RBCs | Howell-Jowell Bodies |
Autosplenectomy | Sickle Cell Disease |
Drug used to treat sickle cell disease | Hydroxyurea (blocks ribonucleoside reductase stopping UDP->dUMP) |
Antiplatelet Antibodies | Immune Thrombocytopenic Purpura |
Bleeding disorder with GpIb deficiency | Bernard-Soulier |
Most common inherited bleeding disorder | von Willebrand Disease |
Cancer most commonly associated with noninfectious fever | Hodgkin Lymphoma-exhibits B symtpoms |
HALLMARK "Smudge" Cells | CLL |
HALLMARK "Punched out" lytic lesions | Multiple Myeloma |
HALLMARK Sheets of lymphoid cells with a "starry sky" appearance | Burkitt Lymphoma |
HALLMARK RBCs clumped together like a stack of coins | Rouleaux Formation-Multiple Myeloma |
HALLMARK Monoclonal AB spike | Multiple Myeloma MGUS Waldenstrom's Macroglobulinemia |
HALLMARK Reddish-pink rods in the cytoplasm of leukemic blasts | Auer Rods-AML |
HALLMARK Large B Cells with bilobed nuclei and prominent "owl eye" inclusions | Reed-Sternberg cells-Hodgkin Lymphoma |
Most common leukemia in children | ALL |
Most common leukemia in adults | CLL |
HALLMARK Characteristic Auer Rods | AML |
Myelodysplastic syndromes have a tendency to progress to this | AML |
Myeloproliferative disorders may progress to this | AML |
More than 20% blasts in the marrow | Acute Leukemia |
Leukemia with more mature cells and <5% blasts | Chronic Leukemia |
HALLMARK PAS (+) acute leukemia | ALL |
HALLMARK Commonly presents with bone pain | ALL |
HALLMARK Numerous basophils, splenomegaly, and negative for leukocyte alkaline phosphatase | CML |
HALLMARK Always positive for the Philadelphia Chromosome | CML |
Acute leukemia positive for peroxidase | AML |
Solid sheets of lymphoblasts in marrow | ALL |
PAS (-) acute leukemia | AML |
Rx Treatment choice for Rickets or Osteomalacia | Vit D |
Swollen gums, poor wound healing, bleeding mucous membranes, and spots on the skin | Scurvy- Vit C deficiency |
Most common cancer of the appendix | Carcinoid Tumor |
Most common surgical emergency | Appendicitis |
GI hamartomas, hyperpigmentation of the mouth, hands and genitalia | Peutz-Jegher's |
Multiple colon polyps, osteomas, soft tissue tumors | Gardner's Syndrome |
Severe RLQ pain with rebound tenderness | Appendicitis |
HALLMARK Apple core" lesion on barium enema | Colon Cancer |
Most common cause of RLQ pain | Appendicitis |
Most common cause of LLQ pain | Diverticulitis |
Most common cause of RUQ pain | Cholecystitis |
Colonoscopsy reveals very friable mucosa extending from the rectum to the distal transverse colon | Ulcerative Colitis |
Most common site of colonic diverticula | Sigmoid colon |
HALLMARK "String sign" on contrast x-ray | Crohn's Disease |
HALLMARK "Lead pipe" appearance of colon on contrast x-ray | Ulcerative Colitis |
Food poisoning due to exotoxin | B. cereus [rice]; S. aureus [all others] |
Osteomyelitis in sickle cells disease | Salmonella |
Food poisoning as a result of mayonnaise sitting out too long | S. aureus preformed toxin |
Diarrhea caused by gram (-) nonmotile organism that does not ferment lactose | Shigella |
Rice-water stool | Vibrio cholera |
Diarrhea caused by an S-shaped organism | Yersinea |
Food poisoning from reheated rice | B. cereus |
Diarrhea caused by gram (-) motile organism that doesn't ferment lactose | Salmonella |
Most common cause of "traveler's" diarrhea | ETEC |
Diarrhea after a course of antibiotics | C. difficile |
Diarrhea caused by gram (-) lactose fermenting bacteria, no fever | E. coli |
Diarrhea caused by gram (-) comma-shaped organism, no fever | Vibrio |
Diarrhea + recent ingestion of water from a stream | Giardia/Entamoeba histolytica |
Food poisoning from undercooked hamburger meat | O157:H7 |
Ring-enhancing brain lesion in an HIV-infected patient | Toxoplasma |
Treatment for Trichomonas vaginalis | Metronidazole for patient & sex partner |
Most common protozoal diarrhea | Giardia Lamblia |
Most common helminth infection in the US | Enterobius vermicularis |
Second most common helminth infection in the US | Ascaris lumbricoides |
Cause of malaria | Cause of malaria Plasmodium |
What enzyme catalyzes peptide bond formation during protein synthesis | rRNA (ribozyme) Peptidyl transferase |
What conditions are associated with target cells (HALT) | HbC Thalassemia Aplenia Liver disease |
What name is given to anemia resulting from mechanical destruction of erythrocytes due to aortic stenosis or prosthetic heart values | Macroangiopathic hemolytic anemia |
Where do platelets come from | megakaryocytes |
Primary hemostasis | platelet plug |
What is inside an endothelial cell | vWF Factor VIII Thromboplastin tPA PGI2 |
What is vWF made up of | Several subunits linked by disulfide bonds, synthesized by endothelial cells and megakaryocytes |
Where are vWF specifically stored in the endothelial cells | Weibel-palade bodies and alpha-granules of platelets |
vWF function | Complexes with factor VIII Platelet adhesion to vessel wall and other platelets (def > Increased bleeding time) |
A deficiency of vWF would look like what coagulation cascade def | Factor VIII (Increased PTT) Hemophilla A |
vWF uses what glycoprotein to bind to platelets | GpIb |
Platelet content (10) | vWF COX ADP, PDGF, serotonin, fibrinogen, lysosomal enzymes, thromboxane A2, calcium, thrombin |
What does ADP do | Helps platelets adhere to endothelium by realising GpIIb/IIIa |
GpIIb/IIIa function | Binds to fibrinogen and forms cross-links among platelets (aggregation) |
Aspirin | NSAID Inhibits COX 1 and 2 (permanently by covalent acetylation) |
Aspirin | Leads to inhibition of TXA2 (decreases platelet aggregation) and Prostaglandin |
Aspirin | Increases BT, no effect on PT or PTT |
Aspirin indications | Antipyretic (Reduce fever) Analgesic (reduce pain) Anti-inflammatory Antiplatelet Acute MI Acute coronary syndromes Acute thrombotic stroke MI prevention |
Aspirin side effects | Gastric ulceration Tinnitus Hyperventilation Respiratory alkalosis Chronic use: Acute renal failure, interstitial nephritis, Upper GI bleeding |
Why should one not give children with a viral sickness aspirin | Reye syndrome (Hepatic encephalopathy Liver problems Hypoglycemia Mitochondrial abnormalities |
Ticlopidine, Clopidogrel, Ticagrelor, Prasugrel | Blocks ADP receptors -Prevents expression of GpIIb/IIIa - Inhibits platelet aggregation |
Why would you give an ADP receptor inhibitor instead of aspirin | You give them these drugs in case of allergy to aspirin |
ADP receptor inhibitor indications | Acute MI Coronary stenting Decreased incidence or recurrence of thrombotic stroke (Post-MI) Aspirin allergy |
ADP receptor inhibitor side effects | Neutropenia (ticlopidine) TTP (ticlopidine) |
Abciximab, eptifibatide, tirofiban Glycoprotein IIb/IIIa Inhibitors | Inhibits Gp IIb/IIIa (directly) - Inhibits platelet aggregation |
What is Abciximab made from | Monoclonal antibody Fab fragments |
What are Glycoprotein IIb/IIIa Inhibitors indications | Unstable angina (NSTEMI) Percutaneous transluminal coronary angioplasty |
Platelet disorder labs | PT, PTT, INR= Normal Bleeding time = increased |
Platelet disorder clinical presentation | Bleeding from mucous membranes Epistaxis Petechiae (microhemorrhages) Purpura |
Platelet disorders | Immune thrombocytopenia purpura (ITP) Thrombotic thrombocytopenia purpura (TTP) Bernard-Souiler syndrome Glanzmann thrombashenia |
ITP lab findings | Low platelet count Increased bleeding time Increased megakaryocytes (biopsy) |
ITP be either acute or chronic. Who is more prone to acute ITP and chronic ITP | Acute= children Chronic= women of childbearing age |
ITP can be associated with | Viral sickness or Lupus |
ITP treatment | Steroids IV Ig Splenectomy |
TTP lab findings | Decreased platelet count Increased bleeding time Schistocytes Increased LDH |
TTP is assc. with what other disorder | HUS (Hemolytic Uremic syndrome) |
What bacteria is HUS associated with; especially kids | E. coli O157:H7 |
Bernard-Soulier disorder labs | No change/decreased platelet count Increased Bleeding time Also no agglutination on ristocetin cofactor assay |
Glanzmann thrombasthenia | Defect in platelet plug formation Decreased GpIIb/IIIa (defect in platelet-platelet formation) Low aggregation |
Glanzmann thrombasthenia labs | Increased bleeding time Normal platelet count Agglutination with ristocetin cofactor assay |
Thrombocytopenia caused by other factors (Low platelet count) | Heparin induced Aplastic anemia |
von Willebrand disease (AD) | Defect in vWF > Increased PTT (similar to Factor 8 def) Defect in platelet plug formation: decreased vWF > defect platelet-vWF adhesion |
von Willebrand disease lab findings | No change in platelet count Increased bleeding time No change in PT Increased PTT |
Mixed platelet and coagulation disorders | von Willebrand disease DIC |
von Willebrand disease diagnosis | Ristocetin cofactor assay (Decreased agglutination= (+) |
von Willebrand disease treatment | (DDAVP) Desmopressin: releases vWF from Weibel-pallade bodies in endothelium |
DIC (Disseminated Intravascular Coagulation) | Widespread activation of clotting Consumes platelets and coagulations factors Hypercoagulable (Bleed time increased) |
DIC (Disseminated Intravascular Coagulation) lab findings | Low platelets Increased bleeding time Increased PT and PTT Schistocytes Increased fibrin split products (D-dimers) Decreased fibrinogen (consuming) Decreased factors V and VIII (consuming) |
DIC (Disseminated Intravascular Coagulation) causes | STOP Making New Thrombi Sepsis (Gm -) Trauma Obstetric complications acute Pancreatitis Malignancy Nephrotic syndrome Transfusion |
What else can increase d-dimers (fibrin split products) | Snake Venom |
What cytokines are increased in DIC | TNF and IL-1 |
Platelet Stimulation | Activation - Secretion of ADP, PDGF, serotonin, fibrinogen, lysosomal enzymes, thromboxane A2, calcium, thrombin - Thrombin: fibrinogen → fibrin - Thromboxane A2 → vasoconstriction and platelet aggregation |
What is the underlying pathophysiology in thrombotic throbocytopenic purpura (TTP) | Deficiency of the metalloprotease ADAMTS 13 |
What is the cause of ITP | Anti-GpIIb/IIIa antibodies binds to receptors on platelets and then immune systems removes platelets by splenic macrophages |
What is the classic triad of HUS? | Hemolysis Uremia Thrombocytopenia |
What is the classic pentad of TTP | Nasty Fever Torched His Kidneys |
What is the defect in Bernard-Soulier disease | Decreased GpIb = defect in platelet to vWF adhesion |
What is the life span of a platelet (which is also the maximum life of platelets after a transfusion) | 8-10 days |
What molecule is expressed on the surface of a platelet after it becomes activated | GpIIb/IIIa |
NSAIDs inhibit the production of which substance important in platelet aggregation? | Thromboxane A2 |
After a normal spontaneous vaginal delivery,the new mom bleeds profusely from her vagina and later from her gums. What abnormal lab values would you suspect | DIC: Decreased platelets Increased bleeding time Increased PT and PTT Increased D-Dimer |
What is the mechanism of action of each of the following drugs? Streptokinase | Directly or Indirectly aid conversion of plasminogen to plasmin |
What is the mechanism of action of each of the following drugs? Aspirin | Inhibits COX 1 and 2 |
What is the mechanism of action of each of the following drugs? Clopidogrel | ADP receptor inhibitor |
What is the mechanism of action of each of the following drugs? Abciximab | Glycoprotein IIb/IIIa Inhibitor |
What is the mechanism of action of each of the following drugs? Tirofiban | Glycoprotein IIb/IIIa Inhibitors |
What is the mechanism of action of each of the following drugs? Ticlopedine | ADP receptor inhibitor |
What is the mechanism of action of each of the following drugs? Enoxaparin | Low-molecular weight heparin Activatior of antithrombin (decreases thrombin and factor Xa) |
What is the mechanism of action of each of the following drugs? Eptifibatide | Glycoprotein IIb/IIIa Inhibitors |
Antiplatelet antibodies | ITP |
Bleeding disorder with gpIb deficiency | Bernard-Soulier syndrome |
Most common inherited bleeding disorder | vWF disease |
What are the 3 different mechanisms cells employ to break down proteins | Ubiquitin protein ligase Lysosomes Calcium-dependent enzymes |
Which medication used in the treatment of HIV is known for causing bone marrow suppression | NRTIs (Zidovudine) |
What is the treatment for lead poisoning | Adult vs Children Adult: EDTA and Succimer Children: Succimer, EDTA, Dimercaprol |
Intravascular hemolysis is going show | A decrease in serum haptoglobin Increase LDH (lactate dehydrogenase) Schistocytes Increased reticulocytes Hemoglobinuria/Hemosiderinuria Urobilinogen in urine |
Intravascular hemolysis is going show | a decrease in serum haptoglobin Increase LDH (lactate dehydrogenase) Schistocytes Increased reticulocytes Hemoglobinuria/Hemosiderinuria Urobilinogen in urine |
Extravascular hemolysis is going show | Macrophages in spleen clear RBCs Spherocytes in peripheral smear Increase LDH (lactate dehydrogenase) No hemoglobinuria/hemosiderinuria Increased unconjugated bilirubin (jaundice) |
Intravascular hemolysis is caused by | Mechanical hemolysis (Prosthetic valve) Paroxysmal nocturnal hemoglobinuria Microangiopathic hemolytic anemia |
Extravascular hemolysis diseases (5) | Hereditary spherocytosis G6PD deficiency Pyruvate Kinase Def HbC defect Sickle cell anemia |
Intrinsic hemolytic normocytic anemia diseases (Problem within RBCS) | Hereditary spherocytosis G6PD deficiency Pyruvate Kinase Def HbC defect Sickle cell anemia Paroxysmal nocturnal hemoglobinuria |
Hereditary spherocytosis | Defect in protein interacting with RBC membrane skeleton and plasma mem. |
What is MCHC | Mean corpuscular hemoglobin concentration |
Hereditary spherocytosis clinical findings | Splenomegaly Aplastic crisis (Parvovirus B19 |
Hereditary spherocytosis diagnostic test? | Osmotic fragility test- Increase lysis |
Hereditary spherocytosis treatment? | Splenectomy (HOWELL-JOLLY bodies) |
G6PD deficiency | Most common enzymatic disorder of RBCs X-linked recessive Defect in G6PD > Decreased glutathione > increased RBC susceptibility to oxidant stress |
Oxidative stress factors are | Sulfa drugs, antimalarials, infections, fava beans |
G6PD deficiency labs | Blood smears shows RBCs with Heinz bodies (clumps of hemoglobin) and bite cells (macrophages cause this) |
G6PD deficiency symptoms | Back pain, hemoglobinuria a few days after oxidant stress |
Pyruvate Kinase Deficiency | AR Decreased ATP > rigid RBCs (swelling & hemolysis) Swelling due to defect in Na+/K+ ATPase Common in newborns |
Paroxysmal nocturnal hemoglobinuria Characteristics What test would you use to test for it? | Increased complement-mediated RBC lysis (impaired synthesis of GPI anchor for decay-accelerating factor that protects RBC membrane from complement) Missing CD55 and CD59 = flow cytometry (+) Ham's test=RBC lysis at low pH |
Paroxysmal nocturnal hemoglobinuria triad | Coomb (-) hemolytic anemia Pancytopenia Venous thrombosis |
Sickle cell disease | HbS point mutation causes a single amino acid replacement in Beta chain (Sub. glutamic acid with valine) |
What factors cause sickling of RBCs | Low O2 tension Dehydration High altitude Acidosis |
What infectious disease are people with sickle cell anemia resistant to? What type of people are more prone to this trait? | Heterozygous (sickle cell trait) = resistance to malaria 8% of African americans |
Why are newborns at first asymptomatic to sickle cell disease | Because of the increased HbF in and decreased HbS |
Sickle cell complications | Aplastic crisis (parvovirus B19) Autosplenectomy (howell-jolly bodies) and Infections from SHiNE SKiS Salmonella osteomyelitis Painful crises (vaso-oclusive) Renal papillary necrosis due to hypoxemia and microhematuria |
What can be seen on skull x-ray in sickle cell patients | "Crew cut" on skull due to marrow expansion from increased erythropoiesis |
Sickle cell disease treatment | Hydroxyurea (increased HbF) Hydration |
HbC defect Common in newborn | Heterozygous for hemoglobin C: Asymptomatic Homozygous for hemoglobin C: -Mild hemolysis and splenomegaly HbC point mutation causes a single amino acid replacement in Beta chain (Sub. glutamic acid with lysine) |
Extrinsic hemolytic normocytic anemia diseases (Problem outside RBCS) | Autoimmune hemolytic anemia Microangiopathic anemia Macroangiopathic anemia Infections |
Autoimmune hemolytic anemia | Warm agglutinin (IgG) Cold agglutinin (IgM) Coombs (+) |
Microangiopathic anemia | RBCs are damaged when passing through obstructed or narrow vessel lumina |
Microangiopathic anemia can be caused by | DIC, TTP/HUS, SLE, and Malignant HTN |
Schistocytes | "Helmet cells" Seen on blood smear due to mechanical destruction of RBCs |
Macroangiopathic anemia | Prosthetic heart valves and aortic stenosis may cause hemolytic anemia, secondary to mechanical destruction Present with schistocytes |
Infections causing hemolytic anemia | Malaria (Plasmodium spp.) Babesia |
A child anemic since birth has now been cured with splenectomy. What is the disease | Hereditary spherocytosis |
What is the difference between the hemoglobin S defect and the hemoglobin C defect? | HbS is a point mutation of B-globin (by sub. of glutamic acid for valine) HbC is a point mutation of B-globin (by sub. of glutamic acid for lysine) |
Coombs' (+) | RBC agglutination with the addition of antihuman antibody because RBCs are coated with immunoglobulin or complement proteins |
Direct Coombs' (DAT) | "Type and screen" |
What is the difference between a warm agglutinin and a cold agglutinin | Cold: Antibodies against RBCs that interact more strongly at low temps (4°C) than at body temp Warm: Antibodies that react against RBC protein antigens at body temperature |
What are schistocytes | Mechanically destructed RBCs |
What are two protozoal diseases that can cause hemolytic anemia | Plasmodium spp. Babesia |
What findings are associated with hereditary spherocytosis | Anemia Jaundice Pigmented gallstones Splenomegaly (+) osmotic fragility test Coomb (-) Spherocytes in peripheral smear |
Cold Agglutinins | Antibodies against RBCs that interact more strongly at low temps (4°C) than at body temp |
Warm Agglutinins | Antibodies that react against RBC protein antigens at body temperature = chronic anemia |
HALLMARK Ham's test | Paroxysmal nocturnal hemoglobinuria |
HALLMARK Heinz bodies | G6PD deifcency |
HALLMARK DEB test | Fanconi anemia |
HALLMARK Osmotic fragility test | Hereditary spherocytosis |
HALLMARK Basophilic stippling | Lead poisoning Thalassemia |
Sx Painful cyanosis of the fingers and toes, with hemolytic anemia | Cold agglutinins |
Autosplenectomy | Sickle cell disease |
Drug used to treat sickle cell disease | Hydroxurea |
What is the target HgbA1C for every diabetic patient | <7.0 |
What are the fiver hereditary thrombosis syndrome | Antithrombin deficiency Factor V Leiden Protein S deficiency Protein C deficiency Prothrombin gene mutation |
Which Mycobacterium spp. fits each of the following description? Causes leprosy | Mycobacterium leprae |
Which Mycobacterium spp. fits each of the following description? Causes pulmonary TB-like symptoms in COPD patients | M. tuberculosis |
Which Mycobacterium spp. fits each of the following description? Causes cervical lymphadenitis in children | M. Scrofulaceum |
Which Mycobacterium spp. fits each of the following description? Causes a disseminated disease in AIDS patients | M. avium-intraceullulare |
Which Mycobacterium spp. fits each of the following description? Hand infection in aquarium | M. marinum |
Atrial fibrillation What are some charcteristics? | Irregularly spaced ORS complexes Irregular baseline No steady rhythm (SA node dysfunction) (Multiple SA nodes) No coordinated atrial contraction = no discrete P waves |
Atrial fibrillation can cause | Atrial stasis (pooling) and lead to thrombosis>emboli |
Atrial fibrillation can also cause SVTs? What is an SVT? How is it caused? | SVT= Supraventricular tachycardia It is caused by the increased signals by the multiple SA nodes to the AV node can cause frequent ventricular depolarization |
Atrial fibrillation treatment? New vs Chronic | New: <48 hrs = synchronized cardioversion |
Rate control drugs? Rhythm control drugs? | Chronic: Heparin, enoxaparin Coumadin Rate control (digoxin, Beta-blockers, CCBs) Rhythm control (sotalol, amiodarone)= Potassium channel blockers |
Why do you not cardioverse a chronic atrial fib | This can cause a thrombus to dislodge and form emboli, due to the heart resetting the contraction |
HALLMARK Atrial flutter characteristics | Sawtooth appearance on EKG |
First Degree Heart Block | PR interval prolonged (>200 msec or 5 little boxes) Asymptomatic |
First Degree Heart Block can be caused by what drug | CCBs (Toxicity= Increased PR interval) |
What bacterial infection can cause an AV node block | Borrelia burgdorgeri (Lyme disease) |
Second degree (Mobitz 1) Heart Block (Wenckebach) What is the major fact about it? | Unsteady rhythm PR intervals increase, until beat drop Every QRS complex is followed by a P-wave, but every P-wave doesn't followed by a QRS complex Benign |
What type of pattern is seen in Second degree (Mobitz 1) Heart Block | Regularly Irregular |
Second degree (Mobitz 2) Heart Block Characteristics? What can is progress to? What is it treated with? | Unsteady rhythm No increae in PR intervals except for DROP (abrupt) Can progress to 3rd degree block Treated with pacemaker |
Third degree heart block Characteristics? What bacterial disease assc. with? Treatment? | Steady rhythm Decreased HR Atria and Ventricular beat independently (SA no communicating with AV) Narrow QRS complex Ventricular rate is slower, atrial faster Treated with pacemaker Lyme disease |
Wolf-Parkinson-White Syndrome | Ventricular pre-excitation syndrome Accessory conduction pathway from atria to ventricle Bypass AV node (shortened PR interval) Goes through bundle of Kent Ventricles partially depolarize earlier |
Ventricles partially depolarizing earlier brings about what characteristics | Delta waves with widened QRS and shortened PR intervals |
Wolf-Parkinson-White Syndrome can result it in | Reentry ciruit > SVTs |
Wolf-Parkinson-White Syndrome treat with | Class IA (procainmide) and III (amiodarone) |
Paroxysmal Supraventricular Tachycardia Caused by | Originates above the AV node Can be caused by AV nodal reentrant tachycardia Narrow QRS |
Junctional Escape Rhythm | Delayed heartbeat no originating from SA node, signal is coming from AV junction Protective mechanism is SA node is dysfunctional, then another area of heart fires off heart beats P-wave location can change depending on where signal originated from |
Ventricular premature beats or PVC (Premature ventricular contraction) | Early occurring wide QRS complexes Microentery at Purkinje fibers No P waves, widened QRS |
Ventricular bigeminy | Occurs when a ventricular premature beat follows each sinus beat |
Ventricular trigeminy | 2 sinus beats followed by a ventricular premature beat |
Ventricular Escape Rhythm | Failure for SA and AV nodes So no P waves and get widened QRS complexes Longer than avg R to R intervals persistent rhythm leads to ventricular complexes Persistence can lower rate < sinus rhythm |
Ventricular Tachycardia (VT) | 3 or more successive ventricular complexes |
Nonsustained vs Sustained | Nonsustained VT: -Series of repetitive ventricular beats -Duration <30secs Sustained VT: Duration >30secs |
Monomorphic VT | Rhythm is normal Slight irregularity of R-R intervals QRS axis shifted to left Width of QRS >0.6 secs |
What happens if Monomorphic VT is sustained? treat with? | hemodynamic collapse; Treat: async defibrillation and CPR |
Torsades de Pointes | Polymorphic VT Shifting sinusoidal waveforms Progress to V. Fib Wide QRS and tachycardia Long QT interval predisposes |
What causes Torsades de Pointes? Treatment? | Hypokalemia Hypomagnesemia Drugs Treat: Magnesium sulfate |
Ventricular Fibrillation | A completely erratic rhythm with NO identifiable waves. Fatal w/o CPR or defib |
What is the initial treatment for ventricular fibrillation | Monomorphic: defibrillation and CPR |
What is the hallmark of a third degree heart bloc | independent beating of atrium and ventricle |
What drugs are know to prolong QT interval, increasing the likelihood of torsades in those at risk | Antibiotics (macrolide) Antiemetics (ondansetron) Antidepressants (TCA) Antiarrhythmics (Class 1A and 3) Antipsychotics (haloperidol) |
What are the two different types of second degree AV block? How do they differ? | Mobitz type 1: Increasing PR interval then drop Mobitz type 2: No warning, normal PR interval + drop |
Why is warfarin anticoagulation important in patients with chronic atrial fibrillation? | Prevents any thrombosis to form due to atrial stasis (pooling) |
What is the fibrous band attached to the testis and scrotum that aids in normal testicular descent? What is this structure called in females? | Gubernaculum Female Remnant: Ovarian ligament + Round ligament of uterus |
What is Osler-Weber-Rendu syndrome? | Hereditary hemorrhagic telangiectasia AD disorder of blood vessels. Findings: telangiectasia, recurrent epistaxis, skin discolorations, arteriovenous malformations (AVMs), GI bleeding, hematuria. |
Which studies use odds ratios, and which use relative risks? | Case-control uses Odds ratio Cohort uses Relative Risk |
What are the effects of NSAIDs' inhibition of PGE2? | Increased vascular tone-vasoconstriction Increased bronchial tone- bronchoconstriction |
What role do prostaglandins and angiotensin II play on the renal arterioles? | prostaglandins: Dilate afferent arteriole (Increases RPF, GFR, so FF remains constant) Angiotensin II: Constricts efferent arteriole Decreases RPF, Increases GFR, so FF increases) |
What is the mechanism of action of the following antiemetics? Scopolamine | Antimuscuranic (M1) |
What is the mechanism of action of the following antiemetics? Promethazine | Histamine antagonist D2 dopamine antagonist |
What is the mechanism of action of the following antiemetics? Prochlorperazine | D2 dopamine antagonist |
What is the mechanism of action of the following antiemetics? Metoclopramide | D2 receptor antagonist |
What is the mechanism of action of the following antiemetics? Ondansetron | 5-HT3 antagonist |
What are the 3 endogenous androgens, in order from the most potent to the least potent | DHT > testosterone > androstenedione |
How does a decrease in renal artery pressure cause an increase in blood pressure | JG cells secrete renin in response to decreased renal blood pressure, decreases NaCl delivery to distal tubule, and Increases sympathetic tone (β1). Angiotesinogen > Angiotensin I > Angiotensin II |
What physiologically is taking place in decompression sickness | Gases (especially nitrogen) that had dissolved in the blood at high pressures form gas bubbles that can occlude blood vessels |
Is a 34-year-old schizophrenic patient having active hallucinations, who is not oriented to time, place, or person, able to legally agree to a plan of care? | No |
What factors must be in place in order for a patient to have the capacity to make a decision? | Patient is ≥ 18 years old or otherwise legally emancipated Patient makes and communicates a choice Patient is informed (knows and understands) Decision remains stable over time Decision is consistent with patient's values and goals, not clouded by a m |
The thalamus is a relay station that conducts signals to and from the brain. Which thalamic nuclei are the relay stations for each of the following body sensations/activities? Auditory sensation | MGN (Medial Geniculate) |
The thalamus is a relay station that conducts signals to and from the brain. Which thalamic nuclei are the relay stations for each of the following body sensations/activities? Visual sensation | LGN (Lateral Geniculate) |
The thalamus is a relay station that conducts signals to and from the brain. Which thalamic nuclei are the relay stations for each of the following body sensations/activities? Motor to the body | VL (Ventral Lateral) |
The thalamus is a relay station that conducts signals to and from the brain. Which thalamic nuclei are the relay stations for each of the following body sensations/activities? Facial sensation | VPM (Ventral Posteromedial) |
The thalamus is a relay station that conducts signals to and from the brain. Which thalamic nuclei are the relay stations for each of the following body sensations/activities? Body sensation | VPL (Ventral Posterolateral) |
What is PPAR-γ | Peroxisome Proliferator-activated Receptor Gamma |
How is it relevant to the treatment of diabetes mellitus? | Plays a role in adipocyte differentiation Nuclear transcription regulator Through Glitazones/thiazolidinediones It increases the insulin sensitivity in peripheral tissue |
A 23-year-old man recently finished treatment to eradicate the lice infestation that he acquired during a recent trip to Mexico. What organism might be responsible for this man's recurrent fever? | Borrelia recurrentis |
What germ layer gives rise to each of the following adult structures | Retina Neuroectoderm |
What germ layer gives rise to each of the following adult structures | Salivary glands Surface ectoderm |
What germ layer gives rise to each of the following adult structures | Pancreas Endoderm |
What germ layer gives rise to each of the following adult structures | Muscle of Abdominal wall Mesoderm |
What germ layer gives rise to each of the following adult structures | Thymus Endoderm |
What germ layer gives rise to each of the following adult structures | Spleen Mesoderm |
What germ layer gives rise to each of the following adult structures | Aorticopulmonary septum Neural Crest |
What germ layer gives rise to each of the following adult structures | Anterior pituitary Surface ectoderm |
What germ layer gives rise to each of the following adult structures | Posterior pitutary Neuroectoderm |
What germ layer gives rise to each of the following adult structures | Bones of skull Neural Crest |
What germ layer gives rise to each of the following adult structures | Cranial nerves Neural Crest |
Left to Right Shunts; what are they? (3) | ASD, VSD, PDA |
What are the clinical uses for 1st (PEcK), 2nd (HEN PEcKS), 3rd, and 4th generation cephalosporins | 1st gen- Gm (+), Proteus m., E.coli, Klebsiella |
What are the clinical uses for 2nd gen-cephalosporins | Haemophilus influenzae, Enterobacter a., Neisseria spp., Gm (+), Proteus m., E.coli, Klebsiella, Serratia m. |
What are the clinical uses for 3rd gen-cephalosporins | Serious gram-negative infections resistant to other Beta-lactams, Pseudomonas, Neisseria gonorrhea |
What are the clinical uses for 4th gen cephalosporins | (Cefepime)- Gm (-), increase activity against Pseudomonas and Gm (+) organisms |
What are the classic symptoms of carcinoid syndrome | (BFDR) Brochospasm and wheezing cutaneous flushing Diarrhea Right-sided valvular disease |
Atrial Septal Defect (ASD) is caused by | Hole in the interatrial septum |
What type of murmur do you hear in an ASD? | Loud S1; wide fixed spilt S2 |
What can be seen on X-ray in ASD? | RVH |
Ventricular septal defect | Most common congenital heart defect |
Defect in intraventricular septum | 40% close in first 6 months of life |
Ventricular septal defect can cause what | If lesion is too large it can cause LV overload or HF |
Patent Ductus Arteriosus (PDA) is caused by | Prostaglandin synthesis and low O2 tension f infant is born through C-section |
What type of murmur do you hear in a Patent Ductus Arteriosus (PDA) | Continuous, "machine-like" murmur Left upper sternal border of the newborn |
Patent Ductus Arteriosus (PDA) treatment | Indomethacin (blocks PGE synthesis) |
When is PDA to remain open | When a patient has Transposition of the great vessels, to allow some oxygenated blood to mix with the deoxygenated blood (systemic) |
Eisenmenger Syndrome Severe | long-standing L to R shunt Increased pulmonary flow> pathologic remodeling of vasculature > pulmonary arterial htn RVH occurs to compensate > shunt becomes right to left |
Eisenmenger Syndrome symptoms | Later cyanosis, clubbing, polycythemia, and SOB |
Coarctation of the Aorta (infantile) | Infantile: Aortic narrowing near intersection of ductus arteriosus ("juxtaductal") - only blood flowing to distal aorta is from PDA. (Req open PDA)...Assc. with Turner Syndrome |
Coarctation of the Aorta (adult) | Distal to ductus arteriosus, Once PDA is closed remnant = Ligamentum aretiosum |
Once PDA is closed the remnant is | Ligamentum aretiosum |
Coarctation of the Aorta symptoms | Hypertension in upper extremities and weak delayed in lower extremities (brachial-femoral delay) Rib notching on chest X-ray (Dilated intercostal arteries) Can also lead to aortic regurg.> HF |
Right to Left shunts | Early cyanosis- "blue babies" (bypass pulmonary circuit) |
Requires surgery or an open PDA | right to left shunts |
Right to Left Shunt Diseases (5 Ts) (hint: TAPVR (5 letters in name) | Truncus arteriosus (1 vessel) Transposition (2 switched vessels) Tricuspid atresia (3= Tri) Tetralogy of Fallot (4 =Tetra) TAPVR (5 letters in name) |
Persistent truncus arteriosus | Failure of truncus arteriosus to divide into pulmonary trunk and aorta Deoxy blood of RV mixes with Oxy blood of LV Cyanosis, Respiratory distress, HF May have VSD |
D-transposition of Great Vessels (TGA) What does it req. to be compatible with life? What medication should be give to help shunts? | Aorta and Pul. artery are switched so Aorta leaves RV and Pulmonary a. leaves LV. Separation of pulmonary and systemic circulation Cyanosis is significant Prostaglandins |
Who are more prone to TGA? | Infant Diabetic Mothers |
LGA (Large for gestational age) | Infants suffer from shoulder dystocia, clavicular fractures, brachial plexus injury (Erb or Duchenne's palsy) Increased C-section |
Tricuspid atresia | Absent tricuspid valve and hypoplastic RV Require ASD and VSD |
Tetralogy of Fallot | Caused by anterosuperior displacement of the infundibular septum (MC cause of early cyanosis) |
4 components of Tetraology of Fallot | Pulmonary infundibular stenosis RVH Overriding aorta VSD |
What helps with the spells of cyanosis in Tetralogy of Fallot | Squatting=Increase SVR, Decreases R>L shunt, improves cyanosis |
Tetralogy of Fallot classic x-ray finding is | RVH or Boot-shaped heart in infant |
Total anomalous pulmonary venous return (TAPVR) Open or closed loop? What is needed to keep patient alive? | Pulmonary veins drain into right atria circulation (SVC, Coronary sinus) Closed loop Without shunt patient will die |
What is the most common congenital cardiac anomaly | VSD |
Ebstein anomaly | Associated with Maternal Lithium use Physical exam: widely split S2, tricuspid regurgitation |
Explain how the great vessels are attached in a transposition of the great vessels | Aorta and Pul. artery are switched so Aorta leaves RV and Pulmonary trunk leaves LV. Separation of pulmonary and systemic circulation |
What heart defect is associated with deletion of Chromosome 22q11 deletions | Tetralogy of Fallot Persistent Truncus arteriosus |
Describe blood flow through a PDA | L > R shunt Aorta > Pulmonary artery |
What heart defect is associated with Down syndrome | Endocardial cushion defect; Can cause abnormal connection of all 4 chambers of the heart |
A 45-year-old man presents with a BP of 160/90 on the right arm and 170/92 on the left arm.There are no palpable pulses in the feet/ankle. What problem does this patient most likely have? | Coarctation of the Aorta |
What heart defect is associated with each of the following disorders: Congenital rubella | PDA Pulmonary artery stenosis |
What heart defect is associated with each of the following disorders? Turner syndrome Infantile coarctation | Bicuspid aortic valves Infective endocarditis |
What heart defect is associated with each of the following disorders? Marfan syndrome | Aortic insufficiency Continuous machinery-like heart murmur PDA |
HALLMARK "Boot-shaped heart" | RVH or Tetralogy of Fallot |
HALLMARK Rib notching | Coarctation of aorta |
Most common congenital cause of early cyanosis | Tetralogy of Fallot |
Where does erythropoiesis take place in the fetus (4)? | Fetus: Yolk Sac. Liver, spleen, bone marrow |
Which bones in adults synthesize RBCs (7)? | Adults:Vertebrae, Sternum, Pelvis, Ribs, cranial bones, and tibia & femur |
Rx Next step in treatment of otitis media if resistant to amoxicillin | Amoxicillin + Clavulanic acid (beta-lactamase inhibitor) |
Rx Prophylaxis against bacterial endocarditis | 1st gen cephalosporin, Aminopenicillins, and Penicillin V |
Rx Increases the nephrotoxicity of aminoglycosides | Cephalosporins |
Rx Sufficient for the treatment of syphilis | Penicillin G |
Rx Single dose treatment for gonorrhea | Ceftriaxone |
What divides the right and left atria | Septum primum and secundum |
How is blood shunted from the right atrium to the left atrium in an embryo | Foramen ovale and ostium secundum |
What structure grows to close the opening/canal between the atrial chamber and ventricular chamber into two smaller openings | Superior and Inferior endocardial cushion |
What genetic abnormality is commonly associated with endocardial cushion defects | Trisomy 21 |
Truncus arteriosus and Aorticopulmonary septum arises from what type of cells | Neural crest cells |
Which embryologic structure of the heart gives rise to each of the following adult structures? Ascending aorta and pulmonary trunk | Truncus arteriosus |
Which embryologic structure of the heart gives rise to each of the following adult structures? Smooth parts of the left and right ventricles | Bulbis Cordis |
Which embryologic structure of the heart gives rise to each of the following adult structures? Trabeculated parts of the left and right ventricles | Primitive Ventricle |
Which embryologic structure of the heart gives rise to each of the following adult structures? Trabeculated left and right atria | Primitive Atrium |
Which embryologic structure of the heart gives rise to each of the following adult structures? Coronary sinus | Left horn of sinus venosus |
Which embryologic structure of the heart gives rise to each of the following adult structures? Smooth part of the right atrium | Right horn of sinus venosus |
Which embryologic structure of the heart gives rise to each of the following adult structures? SVC | R. common cardinal vein and right anterior cardinal vein |
What vessel carries oxygenated blood from placenta to fetus | Umbilical vein |
Areas of shunting in fetal blood flow | Ductus venosus Foramen Ovale Ductus arteriosus |
What starts shutting down fetal circulation, and gives rise to adult circulation | Breathing (respiration) |
Common to hear what murmur in the first 24 hrs of life, and then it disappears | PDA |
Which fetal vessel has the highest oxygenation | Umbilical Vein |
What structure divides the truncus arteriosus into the aortic and pulmonary trunks? What is the cellular origin of this structure? | Aorticopulmonary septum (Spiral) Neural Crest cells |
What causes the ductus arteriosus to close | Breathing Increased O2 and decreased prostaglandins [Indomethicin] (Drug to help close PDA) |
What causes the foramen ovale to close | Breathing Decrease resistance in pulmonary vasculature > increase in left atrial pressure vs. right atrial pressure |
Order of oxygenation from highest to lowest (3) | Umbilical Vein> Ductus venosus> IVC> R. atrium |
What are some of the clinical uses for somatostatin | Pituitary excess: Acromegaly, TSH-secreting tumor, ACTH-secreting tumor GI endocrine excess: Carcinoid syndrome, ZES syndrome, VIPoma, glucagonoma, insulinoma Diarrhea Reduce splanchnic blood circulation: Cirrhosis with bleeding esophageal varices |
What is the name given to a thyroid hormone-secreting teratoma | Mature teratoma (struma ovarii) |
Hypothyroidism symptoms | Constipation Lowering of the voice Menorrhagia Slowed mental and physical function Dry skin with coarse, brittle hair Reflexes showing slow return phase Myxedema (facial/periorbital) Weight gain Cold intolerance |
Levothyroxine | Synthetic analog of thyroxine (T4) |
Why should you start Levothyroxine at a low does for elderly patients | Can cause tachycardia, heat intolerance, arrythmias |
Triiodothyronine | Synthetic analog of T3 |
Cretinism findings? (6 P's) | Poor brain development Pot-bellied Pale Puffy-faced Protuding umbilicus Protuberant tounge |
Hashimoto thyroiditis | Most common cause of hypothyroidism in the U.S. Autoimmune disorder Painless goiter (nontender) |
Hashimoto thyroiditis diagnosis | Antithyroglobulin and Antithyroid peroxidase antibodies Assc. with HLA-DR5 and HLA-B5 |
Hashimoto thyroiditis histology findings? What are these cells called? | Dense infiltrates (germinal centers) of lymphocytes into thyroid. Hurthle cells |
Hashimoto thyroiditis can cause an increased risk in what lymphoma? | Non-Hodgikin lymphoma (Marginal Zone) (B-cell) |
Hashimoto thyroiditis can start off as hyperthyroidism because? | Thyrotoxicosis during follicular rupture |
Hashimoto thyroiditis can also be assc. with what other diseases (3) | Diabetes, Sjrojen syndrome, and Pernicious anemia |
Subacute (de Quervain) Thyroiditis | Hypothyroidism with a PAINFUL (tender) goiter Self-limited disease |
Subacute (de Quervain) Thyroiditis can start off as what? | Hyperthyroidism |
Subacute (de Quervain) Thyroiditis causes what to the thyroid? | Focal destruction with Granulomatous inflammation |
Subacute (de Quervain) Thyroiditis has an increase or decrease of ESR? | Increase |
Subacute (de Quervain) Thyroiditis is associated with? | HLA-B35, Effects Females (3:1) |
Subacute (de Quervain) Thyroiditis often follows what? | Viral infection (Flu-like) or URI |
Riedel's thyroiditis caused by? | Riedel's thyroiditis caused by? Chronic inflammation of thyroid > fibrous tissue (Hypothyroid or euthyroid) |
Riedel's thyroiditis presents as? Histology? | Fixed, hard, rock-like thyroid Painless goiter Histology: Macrophages and Eosinophils |
Prolonged Riedel's thyroiditis in a younger patient can lead to? Mimics what cancer? | Extension of fibrosis which may extend to airway or other structures. Anaplastic carcinoma |
Radioiodine uptake scan increased | Grave's disease or multinodular goiter , or even a single toxic adenoma |
Radioiodine uptake scan decreased | Adenoma or Carcinoma or thyroid |
Papillary carcinoma | Most common thyroid cancer Great prognosis Female predominance MC during 30s-50s |
HALLMARK Papillary carcinoma | "Orphan Annie" eyes or "Ground Glass" appearance or Psammoma bodies (concentric calcfications) |
Papillary carcinoma risk factor | Exposure to childhood irradiation Increased RET and BRAF mutation |
Follicular carcinoma | 2nd most common thyroid cancer Worse prognosis |
Follicular carcinoma diagnosis | Uniform cubodial cells lining the follicles Invade thyroid capsule |
What differs follicular carcinoma and follicular adenoma | Invasion of capsule occurs in carcinoma |
How does Follicular carcinoma spread | Hematogenously |
Follicular adenoma of thyroid | Benign tumor of thyroid Proliferation of follicles surrounded by capsule NO invasion |
Medullary carcinoma of thyroid | Proliferation of parafollicular "C" cells |
Parafollicular "C" cells like to secrete | Calcitonin |
Medullary carcinoma of thyroid assc. with | MEN 2A and 2B (RET mutations) |
Anaplastic carcinoma of thyroid | Anaplastic undifferentiated neoplasm Older patients |
Thyroidectomy complications? (3) | Hoarseness (Recurrent laryngeal nerve) Hypocalcemia (removal of parathyroid glands) Transection of recurrent and superior laryngeal nerves (during ligation of inferior thyroid artery and superior laryngeal artery) |
What type of thyroid cancer matches each of the following statements? Most common type of thyroid cancer (70-75%) | Papillary carcinoma |
What type of thyroid cancer matches each of the following statements? Second most common type of thyroid cancer (10%) | Follicular carcinoma |
What type of thyroid cancer matches each of the following statements? Activation of receptor tyrosine kinases | Papillary and Follicular carcinoma |
What type of thyroid cancer matches each of the following statements? Hashimoto thyroiditis is a risk factor | B-cell Lymphoma |
What type of thyroid cancer matches each of the following statements? Cancer arising from parafollicular C cells | Medullary carcinoma |
What type of thyroid cancer matches each of the following statements? Commonly associated with either a RAS mutation or a PAX8-PPAR gamma 1 rearrangement (LY) | Follicular carcinoma |
What type of thyroid cancer matches each of the following statements? Commonly associated with rearrangements in RET oncogene or NTRK1 | Papillary carcinoma |
What type of thyroid cancer matches each of the following statements? Most common mutation in the BRAF GENE (serine/threonine kinase) | Papillary carcinoma |
Cold tolerance | Hypothyroidism |
Enlarged thyroid cells with ground-glass nuclei | Papillary carcinoma |
Hypothyroidism will cause elevation LDL and cholesterol. Why? | (Hypercholesterolemia) Due to decreased LDL receptor expressio |
What are the characteristics of narcolepsy | Excessive daytime sleepiness Rapid progression from walking state to REM sleep Cataplexy (sudden muscular weakness) Hallucinations |
What is the treatment for narcolepsy | Daytime stimulants:(Amphetamines/Modafinil) and Nighttime sodium oxybate (GHB) |
What is the first-line treatment for a patient with erectile dysfunction | Phosphodiesterase inhibitors Sildenafil Vardenafil Tadalafil If hypogonadal: Testosterone replacement |
What other drugs are used to treat Crohn disease that target TNF-α | Infliximab and adlimumab |
With what physical exam finding must you presume scaphoid fracture despite a normal initial x-ray | Anatomical snuffbox tenderness = scaphoid fracture |
What might result in a proximal fracture of the scaphoid if left untreated | Avascular necrosis in the proximal fragment |
During what week of fetal development will the fetus reach the following landmarks? Implantation | Week 1 |
During what week of fetal development will the fetus reach the following landmarks? Organogensis | Week 3-8 |
During what week of fetal development will the fetus reach the following landmarks? Heart begins to beat | Week 4 |
During what week of fetal development will the fetus reach the following landmarks? can distinguish male or female genetalia | Week 10 |
During what week of fetal development will the fetus reach the following landmarks? gastrulation | Week 3 |
During what week of fetal development will the fetus reach the following landmarks? Formation of primitive streak and neural plate | Week 3-4 |
What is a cholesteatoma, and how does it present | Overgrowth of desquamated keratin debris within middle ear space May erode ossicles, mastoid air cells > conductive hearing loss and vertigo Grayish-white pearly lesion behind TM |
What are the adverse reactions of exogenous testosterone in males | Premature puberty in children Premature closure of epiphyseal plate Erythrocytosis Worsening of sleep apnea Suppression of spermatogenesis Increased LDL and Decreased HDL |
What is the initial medical treatment for the arrhythmia known as torsades de pointes | IV Magnesium sulfate |
What structures give rise to the blood brain barrier | Tight junctions between nonfenestrated capillary endothelial cells Basement membrane Astrocyte foot processes |
Which diuretics are most appropriate for patients with hyperaldosteronism? What are their important side effects? | K+-sparing diuretics Spironolactone, Amiloride, Triamterene, Eplerenone Competitive aldosterone receptor antagonist Hyperkalemia:SE |
Diagram the pathway by which the cochlea communicates a signal to the primary auditory cortex. | Cochlea > Cochlear nuclei > Contralateral superior olivary nucleus > Lateral lemniscus > Inferior colliculus > Medial geniculate body > Primary auditory cortex |
What clinical presentation might lead you to suspect a patient may have lymphoma | Weight loss, night sweats, fever, and painless lympadenopathy |
Most common leukemia in children | ALL |
Most common leukemia in adults in US | CLL |
Characteristic Auer rods | AML |
Greater than 20% blasts in marrow | Acute Leukemia |
Leukemia with more mature cells and less than 5% blasts | Chronic leukemia |
PAS (+) acute leukemia | ALL |
Always positive for the Philadelphia chromosome (t 9;22) | CML |
Acute leukemia positive for PEROXIDASE | AML |
Solid sheets of lymphoblasts in marrow | ALL |
Always associated with the BCR-ABL genes | CML |
What histological change takes place in the trachea of a smoker | Meteplasia (columnar to squamous) |
Major Depressive Disorder | Self-limited episode that must be present for atleast 2 weeks Decreased NE and Serotonin |
Common Antidepressants | SSRI- inhibits reuptake of serotonin SNRI- inhibits reuptake of serotonin and NE TCA- inhibits reuptake of serotonin and NE MAOI- monoamine oxidase inhibitor |
Electroconvulsive therapy (ECT) | Used for depression that isn't responding to any other treatments |
Treatment for atypical depression | First-line: SSRI |
"Maternal (postpartum) blues" | 50-85% incidence rate Depressed affect, tearfulness, and fatigue (2-3 days after delivery) Resolves in a week Support + follow-up= treatment |
What is postpartum depression | Postpartum depressive symptoms that persist for more than or equal to 2 weeks 10-15% incidence rate Depressed affect Anxiety Poor conc. |
Postpartum psychosis | 0.1-0.2% incidence rate Mood-congruent delusions Hallucinations Thoughts of harming baby/self Few days to few weeks Rx. Antipsychotics/Antidepressants |
Dysthmia | Persistent depressive disorder Often milder Lasts at least 2 years |
Seasonal affective disorder | Mild depression during winter months (due to decreased sunlight) Rx. Go outside or go on vacation |
Risk factors for Suicide | SAD PERSONS scale: Sex (male) Age (under 19 or over 45) Depression Previous attempts Ethanol (alcohol/drug use) loss of Rational thinking Sickness Organized plan No social support Stated future intent |
Who tries to commit suicide more? Who succeeds? | Women: try Men: succeed |
What is atypical depression | Hypersomnia Hyperphagia Mood reactivity Hypersensitivity to rejection Leaden paralysis Common subtype of Depression |
A 28-year-old woman has symptoms of mild depression for 6 years. What's the diagnosis | Dysthymia |
A 55-year-old man who is a smoker and heavy drinker presents with a new cough and flu-like symptoms. Gram-stain shows no organisms; silver stain of sputum shows gram-negative rods. What is the diagnosis? | Legionairres disease |
What is the classic presentation of a patient with temporalis arteritis? What lab findings help diagnose temporalis arteritis | Migranes (unilateral), loss of vision, pain when chewing Increased ESR and Biopsy (definitive) |
SSRIs | Used for Depression, Bulimia, general anxiety, and OCD, Premenstrual dysmorphic disorder Inhibit re-uptake of serotonin Takes weeks for it have any effect Less toxic |
Side effects of SSRI | Sexual dysfunction and Serotonin syndrome |
Drugs associated with Serotonin Syndrome? (8) | SSRIs SNRIs MAOIs St. Johns Wort Kava Kava Tryptophan Cocaine Amphetamines |
Treatment for Serotonin Syndrome | Cool patient down and provide benzodiazepine (first line) Or Cyproheptadine (5-HT2 receptor blockers) |
Side effects of SNRIs | Increased BP, sedation, nausea |
TCAs (Tricyclic Antidepressants) | Inhibit serotonin and NE re-uptake (Amitrptyline and Clomipramine-OCD) Older drug Depression Imipramine= bedwetting (enuresis) |
Side effects of TCAs | Sedation Alpha blocking effects Anticholinergic effects Decrease seizure threshold (desipramine and nortripyline not as much) |
Rx Treatment for TCA overdose | NaHCO3 to prevent arrhythmia (to alkalinized the urine)- excretes TCAs |
Monoamine Oxidase Inhibitors (MAOI) | Nonselective MAO inhibition which increases levels of amine neurotransmitter (NE, 5-HT, and Dopamine) |
What is Selegiline used for | Parkinsonism-like symptoms, so it increases dopamine |
MAOI side effects? What is "wash-out" period | Tachycardia and arrhythmia Must wait 2 weeks after stopping MAOI before starting new drug |
Bupropion | Used for smoking cessation Increases NE and dopamine by decreasing reuptake NDRI (NE and Dopamine reuptake inhibitor) Lowers seizure threshold NO sexual dysfunction |
Mirtazapine | Alpha-2 antagonist (pre-synaptic nerve terminals of adernergic neurons) Increase NE and 5-HT Also potent 5-HT2 & 5-HT3 receptor blockers Sedation, Increased appetite, weight gain |
Trazodone | Blocks 5-HT2 and alpha-1 receptors Good for Insomnia Side effect: priapism, sedation, postural hypotension |
What is the MOA of each of the following medication classes? SSRI | 5-HT inhibition re-uptake |
What is the MOA of each of the following medication classes? SNRI | Inhibit serotonin and NE re-uptake |
What is the MOA of each of the following medication classes? TCA | Inhibit serotonin and NE re-uptake (older) |
What is the MOA of each of the following medication classes? MAOI | Nonselective MAO inhibition which increases levels of amine neurotransmitter (NE, 5-HT, and Dopamine) |
SSRI drugs (5) | Fluoxetine, Sertraline, Paroxetine, Citalopram, Fluvoxamine |
TCA drugs | Notriptyline, Imipramine, Amitriptyline, Desipramine, Clomipramine, Doxepin |
MAOI drugs (3) | Selegiline, Tranylcypromine, Phenelzine |
NDRI drugs | Bupropion |
SNRI drugs (3) | Venlafaxine and Duloxetine, Milnacipran |
Tetracyclic drugs (2) | Mirtazapine and Trazodone |
Which antidepressant matches each of the following statements? SE:priapism | Trazodone |
Which antidepressant matches each of the following statements? Lowers the seizure threshold | Burpropion and TCAs |
Which Rx matches each of the following statements? Works well with SSRIs and increases REM sleep | Trazadone |
What are the symptoms of serotonin syndrome? | Hyperthermia Myoclonus Hyperreflexia Flushing and Diarrhea Autonomic instability (HTN and Increased HR) Mental status change Neuromuscular abnormalities |
Which antidepressant matches each of the following statements? Appetite stimulant that is likely to result in weight gain | Mirtazapine |
Which antidepressant matches each of the following statements? Bedwetting in children | TCAs (imipramine) |
What are the symptoms of TCA overdose? Tri-Cs | Convulsions Coma Cardiotoxicity (arrhythmias) Respiratory depression Confusion and hallucination Hyperpyrexia |
Which antidepressant matches each of the following statements? Smoking cessation | Bupropion |
What happens if you ingest tyramine while on MAOIS? | Hypertensive crisis Hemorrhagic stroke Cardiac arrythmias |
What drug is associated with the following side effects? Dry mouth, tinnitus, blurred vision, mania | Amitriptyline (due to anticholinergic effects) |
Tendon xanthomas | Familial hypercholesteremia |
Café-au-lait spots | NF-1 and Mccune albright |
Tuft of hair on lower back | Spina bifida occulta |
What structures are at risk for injury with an anterior shoulder dislocation? | Axillary nerve Posterior circumflex a. Supraspinatus tendon Anterior glenohumoral ligament Glenoid labrum Posterolateral humoral head |
What are the layers of the epidermis | Corneum Lucidum Granulosum Spinosum Basalis |
Candida Albicans | Thrush found in top of mouth and tongue Can be scraped off (leave bleeding mucosa) Common in immunocompromised (AIDS, diabetes, steroids, neonates) |
Vulvovaginitis | Caused by Candida albicans Severe itching Edema and discomfort Vaginal discharge Acidic env |
Candida albicans deep infections? Cause what? | Common in Drug users Endocarditis Dissemiated candidiasis Chronic mucocutaneous candidasis (immundef) |
Candida albicans diagnosis | When placed in serum: Develops germ tubes at 37 degrees Pseudohypahe and budding yeasts at 20 degrees |
Candida albicans treatment | Topical azloe- vaginal Nystatin, Fluconazole, or Caspofungin - oral/esophageal Caspofungin, Fluconazole, and amphotericin B- systemic |
Cryptococcus neoformans How does it replicate? Where is it found? Cultured on? | Heavily encapsulated Narrow based-budding- Rep. Found in soil and pigeon dropping Cultured on Sabouraud agar |
Cryptococcus neoformans diagnosed using | India Ink (stains polysaccharide capsule) Mucicarmine (stains red) |
Cryptococcus neoformans antigen test used | Latex agglutination test detects polysaccharide capsular antigen and is more specific |
Cryptococcus neoformans lesions look like? What is the disease assc. | Soap Bubble in brain (Cryptococcal meningoencephalitis) |
Cryptococcal meningitis treatment | Combo of Amphotericin B + Flucytosine and then followed by single-therapy Fluconazole |
Pneumocystis jirovecii What disease does it cause? Who does it affect? | Pneumocystis pneumonia (PCP) Diffuse intersitisl pneumonia Aytpical pneumonia (walking pneumonia) Asymptomatic normally Symptomatic for Immunosuppressed |
Pneumocystis jirovecii diagnosis | Identified in tissue or bronchial lavage using silver stain Chest imaging shows: Ground-glass opacities of interstital infiltrates |
Pneumocystis jirovecii treatment | TMP/SMX (Bactrim) |
When do you start PCP prophylaxis on HIV patients? (TMP-SMX) | Start when CD4+ count drops <200 cells/mm3 |
Aspergillus fumigatus (MOLD) | Can cause: Allergic reactions (ABPA) Colonized = Aspergilloma (fungus ball) after Tb Invasive aspergillus (immunocompromised) |
Aspergillus fumigatus diagnosis | Narrow septate hyphae that branch at acute angles (less than 45 degrees) |
Aspergillus fumigatus treatment | Combo ("-Azole" + Amphotericin B) |
Mucor and Rhizopus | Cause mucormycosis Cause disease in severely immunocompromised |
Mucor and Rhizopus diagnosis | Broad Irregular shaped, nonseptate hyphae branching at WIDE angles (90 degrees) -Ribbon-like Penetrate cribiform plate to enter bloodstream |
Patients with diabetic ketoacidosis or leukemia are prone to develop | Rhinocerebral frontal lobe abcess: Facial pain and headache Black necrotic eschar on face Cranial nerve deficits and blindness |
Mucor and Rhizopus treatment | Surgical debriment and amphotericin B High mortality Progression rapid |
Dermatophytes (diagnosis) | Branching septate hyphae visible on KOH prep. with blue fungal stain |
What are 4 molds that are considered dermatophytes (fungal species commonly found invading superficial layers of skin) | Trichophytin Microsporum (most prevalent) Epidermophytin Malassezia furfur |
What infections are commonly caused by dermatophytes | Tina pedis- feet Tina cruis- groin Tinea corporis- body (ringworm) Tinea capitis- head (ringworm |
Simple cutaneous dermatophytosis treatment | Topical Terbinafine/Azoles Extensive infection or that involve scalp and hair require oral Terbinafine/Azoles |
Tinea unguium-(onchomycosis)-nails Treatment | Caused by Trichophyton rubrum Thickened discolored nails Terbinafine/Azoles (oral) |
Tinea Vesicolor Caused by? Type of yeast? Causes? Occurrence? | Caused by Malassezia furfur Lipophillic yeast Hypopigmented/hyperpigmented patches on body (back and shoulders) Effects trunk and proximal limbs Hot, humid weather |
Tinea Vesicolor diagnosis and treatment | KOH prep: spaghetti and meatballs look" Treat with topical azole (selenium sulfide) |
Systemic MYCOSES Causes? Type of fungi? | Pneumonia and disseminate Dimorphic fungi NO person to person transmission Granuloma formation (noncaseseating) |
Systemic mycoses treatment? Local: Systemic: | Local: Fluconazole/itraconazole Systemic: Amphotericin B |
Histoplama capsulatum "Histo hides" | Macrophages filled with Histoplasma (smaller than RBC) Hepatosplenomegaly |
Blastomyces dermatitidis | Inhaled Can disseminate to skin and bones Forms granulomatous nodules |
Coccidioides immitis | Exists as a spherule (filled with endospores) Arthroconidia (barrel shape) Bigger than RBC |
Coccidioides immitis can cause if severe | Desert bumps= erythema nodosum Desert rheumatism= arthralgias |
Paracoccidioides brasiliensis | Inhaled Disseminates widely Severe pneumonia, infection of mucus membranes and skin Captain Wheels formation |
Sporothrix schenckii Can cause | Sporotrichosis Dimorphic fungus Lives in soil and on vegetation Rose garderner's disease |
Rose garderner's disease How is it caused? What forms | Caused by pricks from thorns or sphagnum moss Small pustule/ulcers Nodules (painless) also form along draining lymphatics |
Sporotrichosis treatment | Itraconazole or Potassium iodide |
Found in SW U.S. including West Texas and California | Coccidiomyces |
Found in Mississippi and Ohio River basins | Histoplasma |
Found in rural Latin America | Paracoccidiomyces |
Associated with plant thorns and cutaneous injury | Sporothrix schenckii |
Causes San Joaquin Valley fever | Coccidiomyces |
Found in states east of the Mississippi River (Central America-Canada) | Blastomyces |
Found in bird and bat droppings | Histoplasma |
Causes thrush in immunocompromised patients and vulvovaginitis in women | Candida albicans |
Mold form contains barrel-shaped arthroconidia | Coccidiomyces |
Multiple budding of yeast form | Paracoccidiomyces b. |
Opportunistic mold with septate hyphae that branch at a 45° angle | Aspergillus fumigatus |
Yeast known for causing meningitis in AIDS patients | Cryptococcus meningitis |
Opportunistic mold with irregular nonseptate hyphae that branch at wide angles (>90 degrees) | Mucor and Rhizopus |
Associated with dust storms and earthquakes | Coccidiomyces |
An HIV (+) patient with CSF showing 75/mm3 lymphocytes suddenly dies. Yeast is identified in the CSF. What is the most likely diagnosis? | Cryptococcus neoformans |
Broad-based budding of yeast | Blastomyces |
What infections are associated with birds | Histoplasma capsulatum Cryptococcus neoformans Chlamydophila psittaci H5N1 Influenza (Bird Flu) West Nile Virus |
Causes diaper rash | Candida albicans Erythematous skin Satellite lesions |
Known for causing pneumonia in AIDS patients start Bactrim prophylaxis when CD4 <200 | Pneumocystis jirovecii |
Most common opportunistic infection in HIV patients | Pneumocystis jirovecii |
Prophylaxis for Cryptococcus in AIDS patients | Fluconazole |
Prophylaxis for PCP in AIDS patients | TMP-SMX |
Treatment for oral candidiasis | Nystatin/Fluconazole |
Treatment for systemic candidiasis | Amphotericin B |
Opportunistic fungal infections | Candida albicans Aspergillus Cryptococcus neoformans Mucor & Rhizopus spp. |
What is the classic clinical presentation of a thyroglossal duct cyst | Presents as anterior midline neck mass that moves with swallowing or protrusion of tongue (asymptomatic) <30 yrs Can become infected (abcess) |
What conditions are associated with an elevated ESR (erythrocyte sedimentation rate) | Most anemias Infections (osteomyletis) Polymyagia rheumatica Inflammation (Temporal arteritis) Cancer Pregnancy Autoimmune disorders (Rheumatoid) |
What is the clinical use for tiotropium | Anti-muscuranic Respiratory COPD, asthma Relax airway |
1st order velocity/kinetics | when reaction velocity is directly proportional to concentration of substrate |
Zero order velocity/kinetics | is when velocity is completely independent of concentration of substrate |
Vmax | maximum velocity the reaction can achieve Proportional to the amount of enzymes available |
Km | substrate concentration at 1/2 of Vmax Inversely related to the affinity of the enzyme for its substrate |
Lineweaver-Burk plot | x-axis: 1/S y-axis: 1/V Slope = Km/Vmax X-intercept: 1/-Km Y-intercept: 1/Vmax |
Increased y-intercept does what to Vmax | Decreases Vmax |
A shift to the right on the x-intercept (closer to zero) does what to Km and affinity | increases Km and Lowers affinity |
Competitive inhibitor does what to Vmax | it does nothing to Vmax and increases Km |
Noncompetitive inhibitor does what to Vmax | it decrease Vmax and does nothing to Km |
What 4 pharmacokinetics equations are most important to know for Step 1 | 1) Volume of Distribution= Amt of drug in body/plasma drug conc 2)Clearance= Rate of elimin/plasma drug conc= (0.7 x Vd/half-life) 3) Loading Dose= Conc at steady state X Vd 4)Maintenance dose= Conc at steady state x Clearance |
Half-life | the time required to change the amount of drug in the by body HALF during elimination or constant infusion |
94% steady state concentration can be achieved after how many half lives | 4 Half-lives |
Efficacy | the maximal effect a drug can produce |
Potency | amount of drug needed for a given effect |
Competitive anatagonist | Shifts curve to right and lowers potency no change in efficacy Increased Km & same Vmax |
Noncompetitive antagonist or Irreversible antagonist | Shifts curve down (decrease efficacy) Decrease Vmax and same Km |
Partial agonist | Shifts curve down and left (decrease efficacy but increased potency) Decrease Vmax and Km |
What variable can you not compare between Agonist and Partial agonist | potency, because it is an independent variable |
Therapeutic index (TI)= | LD50= lethal dose to about 50% of people _____________________________________________ ED50= effective dose to about 50% of people |
Safer drugs have a | Higher therapeutic index (Increased LD50 or Decreased ED50) |
Examples of drugs with low TI? (5) | Digioxin Lithium Theophylline Warfarin Seziure drugs |
Therapeutic window | Measure of clinical drug effectiveness for a patient Higher therapeutic window= safer the drug |
How do you decreased target concentration by 1/2 | you stop infusion of drug for 1 half-life |
If clearance is decreased, but Vd is unchanged, How does that effect loading dose and maintenance dose? | loading Dose would remain unchanged, but Maintenance dose would decrease. |
Where would you expect to find B cells in a lymph node | B-Cells located in Follicle of lymph node |
Where would you find T cell | T-Cell located in Paracortex of lymph node macrophages located in Medullary Sinus of lymph node Plasma cells located in Medullary Cords of lymph node |
Where would you find macrophages | macrophages located in Medullary Sinus of lymph node |
Where would you find plasma cells | plasma cells located in Medullary Cords of lymph node |
The protein derived from what gene serves as a transcription factor for the development and function of regulatory T cells | FOXP3- Found on X chromosome Codes for Forkhead Box Protein P3 Important for self-tolerance Absence= autoimmune diseases |
What is the female homologue to each of the following male structures? Corpus spongiosum | Vestibular bulbs |
What is the female homologue to each of the following male structures? Cowper's glands (bulbourethral glands) | Greater vestibular glands of Bartholin |
What is the female homologue to each of the following male structures? Prostate | Urethral and paraurethral glands of Skene |
What is the female homologue to each of the following male structures? Glans penis | Glans clitoris |
What is the female homologue to each of the following male structures? Ventral penile shaft | Labia minora |
What is the female homologue to each of the following male structures? Scrotum | Labia majora |
What important intracellular proteins are common to both the extrinsic and intrinsic apoptotic pathways? | Cytosolic caspases |
What ratio indicates fetal lung maturity? What is the proper name for the main component of pulmonary surfactant? | Lecithin:sphingomyelin ratio (>2:1) Surfactant main component = lecthins (dipalmitoylphosphatidylcholine) |
What amino acids are necessary for purine synthesis | Glycine Aspartate Glutamine (GAG) |
What nutrient deficiency is associated with spooning of the nails (koilonychia) | iron |
What is the equation for determining a drug's clearance | CL= (0.7) x Vd ----------------- Half-Life |
Which bacteria are well known for being obligate intracellular bacteria | Rickettsia Chlamydia Coxiella |
Why can't obligate intracellular bacteria replicate extracellularly | because they rely on host ATP, can't make it on their own |
What is compartment syndrome, and how is it treated | Trauma/Burn/intense excercise to leg: Increased pressure within the fascial compartment Treatment: Fasciotomy |
Separates the greater and lesser sacs | Gastrohepatic ligament (right) Gastrosplenic ligament (left) |
May be cut during surgery to access the lesser sac | Gastrohepatic ligament |
2 ligaments that connect the spleen to other structures | Gastrosplenic and Splenorenal ligaments |
Contains the portal triad | Heaptoduodenal ligament |
Connects liver to the anterior abdominal wall | Falciform ligament |
Where are neurotrasmitters made | RER |
infant with failure to thrive, hepatosplenomegally, neurodegeneration | Neimann-Pick disease (genetic sphingomyelinase deficiency) |
infant with hypoglycemia, failure to thrive, and hepatomegaly | Cori's disease (debranching enzyme deficiency) |
infant with microcephaly, rocker-bottom feet, clenched hands, and structural heart defect | Edward's syndrome (trisomy 18) |
infant with failure to thrive, hepatosplenomegally, neurodegeneration | Neimann-Pick disease (genetic sphingomyelinase deficiency) |
infant with hypoglycemia, failure to thrive, and hepatomegaly | Cori's disease (debranching enzyme deficiency) |
infant with microcephaly, rocker-bottom feet, clenched hands, and structural heart defect | Edward's syndrome (trisomy 18) |
Jaundice, RUQ pain, fever | Charcot's triad (ascending cholangitis) |
Keratin pearls on a skin biopsy | squamous cell carcinoma (lung and cervix) |
large rash with bull's eye appearance | erythema chronicum migrans from Ixodes tick bite (Lyme Disease: Borrelia) |
Lucid interval after traumatic brain injury | epidural hematoma (middle meningeal artery rupture) |
male child, recurrent infections, no mature B cells | Bruton's disease (X-linked agammaglobulinemia) |
Mucosal bleeding and prolonged bleeding time | Glanzmann's thrombasthenia (defect in platelet aggregation due to lack of GpIIb/IIIa) |
Multiple colon polyps, osteomas/soft tissue tumors, impacted/supernumerary teeth | Gardner's syndrome (subtype of FAP) |
Necrotizing vasculitis (lungs) and necrotizing glomerulonephritis | Wegener's (c-ANCA positive) and Goodpasture's syndromes (anti-basement membrane antibodies) |
Neonate with arm paralysis following difficult birth or following clavicle fracture | Erb-Duchenne palsy (superior trunk [C5-C6] brachial plexus injury: "waiter's tip") |
no lactation postpartum, absent menstruation, cold intolerance | Sheehan's syndrome (pituitary infarction) |
Nystagmus, intention tremor, scanning speech, bilateral internuclear ophthalmoplegia | Multiple Sclerosis (Charcot's triad) |
Oscillating slow/fast breathing | Cheyne-Stokes respirations (central apnea in CHF or increased intracranial pressure) |
painful blue fingers/toes, hemolytic anemia | cold agglutinin disease (autoimmune hemolytic anemia caused by Mycoplasma pneumoniae, infectious mononucleosis) |
painful, pale, cold fingers/toes | Raynaud's syndrome (vasospasm in extremities) |
painful, raised red lesions on palms and soles | Osler's node (infective endocarditis) |
painless erythematous lesions on palms and soles | Janeway lesions (infective endocarditis) |
painless jaundice | cancer of the head of the pancreas obstructing the bile duct |
palpable purpura, joint pain, abdominal pain (child) | Henoch-Schonlein purpura (IgA vasculitis affecting skin and kidneys) |
pancreatic, pituitary, parathyroid tumors | Wermer's syndrome (MEN I) |
precocious puberty, cafe-au-lait spots, multiple unilateral bone lesions | McCune-Albright syndrome (a form of Polyostotic fibrous dysplasia) |
pink complexion, dyspnea, hyperventilation | "pink puffer" (emphysema: centroacinar [smoking], panacinar [alpha1-antitrypsin deficiency]) |
polyuria, acidosis, growth failure, electrolyte imbalances | Fanconi's Syndrome (proximal tubular reabsorption defect) |
Positive anterior "drawer sign" | Anterior cruciate ligament (ACL) injury |
ptosis, miosis, anhidrosis | Horner's Syndrome (sympathetic chain lesion) |
pupil accommodates but doesn't react | Argyll Robertson pupil (neurosyphilis) |
Rapidly progressive leg weakness that ascends (following GI/upper respiratory infection) | Guillain-Barre syndrome (autoimmune acute inflammatory demyelinating polyneuropathy) |
Rash on palms and soles | Secondary Syphilis, Rocky Mountain Spotted Fever, Coxsackie Virus |
Recurrent colds, unusual eczema, high serum IgE | Job's syndrome (hyper-IgE syndrome: neutrophil chemotaxis abnormality) |
Red "currant jelly" sputum in alcoholic or diabetic patients | Klebsiella pneumoniae |
Red, itchy, swollen rash of nipple/areola | Paget's disease of the breast (represents underlying neoplasm) |
red urine in the morning, fragile RBCs | paroxysmal nocturnal hemoglobinuria |
renal cell carcinoma, hemangioblastomas, angiomatosis, pheochromocytoma | von Hippel-Lindau disease (dominant tumor suppressor gene mutation) |
resting tremor, rigidity, akinesia, postural instability | Parkinson's disease (nigrostriatal dopamine depletion) |
restrictive cardiomyopathy (juvenile form: cardiomegaly), exercise intolerance | Pompe's disease (lysosomal glucosidase deficiency) |
retinal hemorrhages with pale center | Roth's spots (bacterial endocarditis) |
severe jaundice in neonate | Crigler-Najjar syndrome (congenital unconjugated hyperbilirubinemia) and lots of other causes |
Severe RLQ pain with rebound tenderness | McBurney's sign (appendicitis) |
short stature, increased incidence of tumors/leukemia, aplastic anemia | Fanconi's anemia (genetically inherited; often progresses to AML) |
single palm crease | Simian Crease (Down Syndrome) |
Situs inversus, chronic sinusitis, bronchiectasis | Kartagener's syndrome (dynein defect affecting cilia) |
skin hyperpigmentation | Addison's disease (primary adrenocortical insufficiency of autoimmune or infectious etiology) |
Slow, progressive muscle weakness in boys | Becker's muscular dystrophy (X-linked, defective dystophin; less sever than Duchenne's) |
small, irregular red spots on buccal/lingual mucosa with blue-white centers | Koplik spots (measles) |
smooth, flat, moist white lesions on genitals | condylomata lata (secondary syphilis) |
splinter hemorrhages in fingernails | bacterial endocarditis |
"Strawberry tongue" | Scarlet fever, Kawasaki disease, toxic shock syndrome=streak ovaries, congenital heart disease, horseshoe kidney=turner syndrome (XO, short stature, webbed neck, lymphedema) |
sudden swollen/painful big toe joint, tophi | gout/podagra (hyperuricemia) |
swollen gums, mucous bleeding, poor wound healing, spots on skin | scurvy (vitamin C deficiency: can't hydroxylate proline/lysine for collagen synthesis) |
swollen, hard, painful finger joints | osteoarthritis (osteophytes on PIP [Bouchard's nodes], DIP [Heberden's nodes]) |
systolic ejection murmur (crescendo-decrescendo) | aortic valve stenosis |
Thyroid and Parathyroid tumors, pheochromocytoma | Sipple's Syndrome (MEN 2A) |
Toe extension/fanning upon plantar scrape | Babinski's sign (UMN lesion) |
Unilateral facial drooping involving forehead | Bell's palsy (LMN CN VII palsy) |
Urethritis, conjunctivitis, arthritis in a male | reiter's syndrome (reactive arthritis associated with HLA-B27) |
Vascular birthmark (port-wine stain) | Hemangioma (benign, but associated with Sturge-Weber syndrome) |
Vasculitis from exposure to endotoxin causing glomerular thrombosis | Shwartzman reaction (following second exposure to endotoxin) |
Vomiting blood following esophagogastric lacerations | Mallory-Weiss syndrome (alcoholic and bulimic patients) |
"Waxy" casts with very low urine flow | chronic end-stage renal disease |
WBC casts in urine | acute pyelonephritis |
weight loss, diarrhea, arthritis, fever, adenopathy | Whipple's disease (tropheryma whippelii) |
"worst headache of my life" | subarachnoid hemorrhage |
WBCs in urine | acute pyelonephritis and cystitis |
Autosomal Chromosome 17 mutation in tumor suppressor gene that regulates "ras" signaling | Neurofibromatosis type I |
vitamin K dependent clotting factors | II, VII, IX, and X as well as proteins C and S |
anticentromere antibodies | Scleroderma (CREST) |
Antidesmoglein (epithelial) antibodies | pemphigus vulgaris (blistering) |
anti-glomerular basement membrane antibodies | Goodpasture's syndrome (glomerulonephritis and hemoptysis) |
antihistone antibodies | Drug-induced SLE (hydralazine, isoniazid, phenytoin, procainamide) |
anti-IgG antibodies (IgM) | Rheumatoid arthritis (systemic inflammation, joint pannus, boutonniere deformity) |
antimitochondrial antibodies (AMAs) | Primary Biliary cirrhosis (female, cholestasis, portal hypertension) (also pANCA present) |
antineutrophil cytoplasmic antibodies (ANCAs) | vasculitis (c-ANCA: Wegener's; pANCA: microscopic polyangiitis, Churg-Strauss syndrome, Pauci-immune crescentic glomerulonephritis) |
Antinuclear antibodies (ANAs: anti-Smith and anti-dsDNA) | SLE (type III hypersensitivity) |
Antiplatelet antibodies | Idiopathic thrombocytopenic purpura (ITP) (bleeding diathesis) |
Anti-topoisomerase antibodies | diffuse systemic scleroderma (not CREST) |
anti-transglutaminase antibodies | Celiac disease (diarrhea, distention, weight loss) |
antigliadin antibodies | Celiac disease (diarrhea, distention, weight loss) |
anti-endomysial antibodies | celiac disease (diarrhea, distention, weight loss) |
azurophilic granular needles in leukemic blasts | auer rods (acute myelogenous leukemia: especially the promyelocytic type) |
"Bamboo spine" on x-ray | ankylosing spondylitis (chronic inflammatory arthritis: HLA-B27) |
Basophilic nuclear remnants in RBCs | Howell-Jolly bodies (due to splenectomy or nonfunctional spleen) |
Basophilic stippling of RBCs | Lead poisoning or siderblastic anemia |
Bloody tap on LP | Subarachnoid hemorrhage |
"Boot-shaped" heart on x-ray | Tetralogy of Fallot, RVH |
Branching gram-positive rods with sulfur granules | actinomyces israelii |
bronchogenic apical lung tumor | pancoast's tumor (can compress sympathetic ganglion and cause Horner's syndrome) |
"Brown" tumor of bone | Hemorrhage (hemosiderin) causes brown color of osteolytic cysts. Due to: 1. hyperparathydoidism 2. Osteitis fibrosa cystica |
Cardiomegaly with apical atrophy | Chagas' disease (typanosoma cruzi) |
cellular crescents in Bowman's capsule | rapidly progressive crescentic glomerulonephritis |
"chocolate cyst" of ovary | endometriosis (frequently involves both ovaries) |
circular groupoing of dark tumor cells surrounding pale neurofibrils | Homor Wright rosettes (neuroblastoma, medulloblastoma, Ewing Sarcoma) |
Colonies of mucoid Pseudomonas in lungs | Cystic fibrosis (CFTR mutation in Caucasians resulting in fat-soluble vitamin deficiency and mucous plugs) |
abdominal pain, ascites, hepatomegaly | Budd-Chiari syndrome (posthepatic venous thrombosis- no JVD) |
Achilles tendon xanthoma | familial hypercholesterolemia (increased LDL leads to deposits) |
Adrenal hemorrhage, hypotension, DIC | Waterhouse-Friderichsen syndrome (meningococcemia) |
arachnodactyly, lens discoloration, aortic dissection, hyperflexible joints | Marfan's syndrome (fibrillin defect) |
athlete with polycythemia | erythropoietin injection |
back pain, fever, night sweats, weight loss | Pott's disease (vertebral tuberculosis) with caseating granulomas |
Bilateral hilar adenopathy, uveitis | |
blue sclera | Osteogenesis imperfecta (type I collagen defect) |
bluish line on gingiva | Burton's line (lead poisoning) |
bone pain, bone enlargement, arthritis, increased alk phos | PAGET'S DISEASE of bone (increased osteoblastic and osteoclastic activity) |
Bounding pulses, diastolic heart murmur, head bobbing | aortic regurgitation |
Cafe-au-lait spots, Lisch nodules (iris hamartoma) | Neurofibromatosis type I (+pheochromocytoma, optic gliomas) Neurofibromatosis type II (+bilateral acousitc neuromas) |
Cafe-au-lait spots, polyostotic fibrous dysplasia, precocious puberty | McCune-Albright syndrome (mosaic G-protein signaling mutation) |
Calf pseudohypertrophy | muscular dystrophy (most commonly Duchenne's) |
"Cherry-red spot" on macula | Tay-Sachs (ganglioside accumulation) or Niemann-Pick (sphingomyelin accumulation), central retinal artery occlusion |
Chest pain, pericardial effusion/friction rub, persistent fever following MI | Dressler's syndrome (autoimmune-mediated post-MI fibrinous pericarditis, 1-12 weeks after acute episode) |
Child uses arms to stand up from squat | Gowers' sign (Duchenne muscular dystophy: X-linked recessive deleted dystrophin gene) |
Child with fever develops red rash on face that spreads to body | "Slapped cheeks" (erythema infectiosum/fifth disease: parvovirus B19) |
chorea, demetnia, caudate degeneration | Huntington's disease (autosomal-dominant CAG repeat expansion) |
chronic exercise intolerance with myalgia, fatigue, painful cramps | McArdle's disease (muscle phosphorylase deficiency) |
Cold intolerance | hypothyroidism |
conjugate lateral gaze palsy, horizontal diplopia | internuclear ophthalmoplegia (damage to MLF; bilateral [multiple sclerosis], unilateral [stroke]) |
continuous "machinery" heart murmur | PDA (close with indomethacin; open with misoprostol) |
cutaneous/dermal edema due to connective tissue deposition | myxedema (hypothyroidism, Graves' disease) |
Dark purple skin/mouth nodules | Kaposi's sarcoma (usually AIDS patients [gay men]: associated with HHV-8) |
Deep, labored breathing/hyperventilation | Kussmaul breathing (diabetic ketoacidosis) |
Dermatitis, dementia, diarrhea, death | pellagra (niacin [vitamin B3] deficiency) |
dilated cardiomyopathy, edema, polyneuropathy | wet beriberi (thiamine [vitamin B1] deficiency) -> dry is without dilate cardiomyopathy and edema |
dog or cat bite resulting in infection | Pasteurella multocida (cellulitis at inoculation site) |
dry eyes, dry mouth, arthritis | Sjogren's syndrome (autoimmune destruction of exocrine glands) |
Dysphagia (esophageal webs), glossitis, iron deficiency anemia | Plummer-Vinson syndrome (may progress to esophageal squamous cell carcinoma) |
elastic skin, hypermobility of joints | Ehlers-Danlos Syndrome (collagen defect, usually type III) |
enlarged, hard left supraclavicular node | virchow's node (abdominal metastasis) |
erythroderma, lymphadenopathy, hepatosplenomegaly, atypical T cells | Sezary syndrome (cutaneous T-cell lymphoma) or mycosis fungoides |
facial muscle spasm upon tapping | Chvostek's sign (hypocalcemia) |
fat, female, forty, and fertile | Chvostek's sign (hypocalcemia) |
fever, chills, headache, myalgia following antibiotic treatment for syphilis | Jarisch-Herxheimer reaction (rapid lysis of spirochetes results in toxin release) |
Fever, cough, conjunctivits, coryza, diffuse rash | measles (morbillivirus) |
fever, night sweats, weight loss | B symptoms (lymphoma) or TB |
Fibrous plaques in soft tissue of penis | Peyronie's disease (connective tissue disorder) |
gout, mental retardation, self-mutilating behavior in a boy | Lesch-Nyhan syndrome (HGPRT deficiency, X-linked recessive) |
Green-yellow rings around peripheral cornea | Kayser-Fleischer rings (copper accumulation from Wilson's disease) |
Hamartomatous GI polyps, hyperpigmentation of mouth/feet/hands) | Peutz-Jeghers syndrome (genetic benign polyposis can cause bowel obstruction; increase cancer risk |
Hepatosplenomegaly, osteopososis, neurologic symptoms | Gaucher's disease (glucocerebrosidase deficiency) |
Hereditary nephritis, sensorineural hearing loss, cataracts | Alport's syndrome (type IV collagen mutation) |
hypercoagulability (leading to migrating DVTs and vasculitis) | Trousseau's sign (adenocarcinoma of pancreas or lung) |
Hyperphagia, hypersexulaity, hyperorality, hyperdocility | Kluber-Bucy syndrome (bilateral amygdala lesion) |
Hypertension, hypokalemia, metabolic alkalosis | Conn's syndrome (primary hyperaldosteronism) |
hypoxemia, polycythemia, hypercapnia | "blue bloater" (chronic bronchitis: hyperplasia of mucous cells) |
indurated, ulcerated genital lesion | NONPAINFUL: chancre (primary syphilis, Treponema pallidum); PAINFUL: painful, with exudate: chancroid (Haemophilus ducreyi) |
Degeneration of dorsal column nerves | tabes dorsalis dorsalis (tertiary syphilis) |
Depigmentation of neurons in substantia nigra | Parkinson's disease (basal ganglia disorder: rigidity, resting tremor, bradykinesia) |
desquamated epithelium casts in sputum | curschmann's spirals (bronchial asthma; can result in whorled mucous plugs) |
disarrayed granulosa cells in eosinophilic fluid | Call-Exner bodies (granulosa-theca cell tumor of the ovary) |
back pain, fever, night sweats, weight loss | Pott's disease (vertebral TB) |
big toe extension/fanning upon plantar scrape | Babinski's sign (UMN lesion --> spastic paralysis) |
Bilateral hilar adenopathy, uveitis | sarcoidosis (noncaseating granulomas) |
elevated D-dimers can be due to... | thrombosis, DIC, PE, DVTs, Budd Chiari |
abdominal pain, ascites, hepatomegaly | Budd-Chiari syndrome (posthepatic venous thrombosis - looks like CHF but no JVD) |
Bilateral hilar adenopathy, uveitis | sarcoidosis (noncaseating granulomas) |
bluish line on gingiva | Burton's line (lead poisoning) |
What is deficient in I- cell disease? | Phosphitransferase - tags enzymes with mannose 6 phosphate |
what bacterial structure is the space between the inner and outer cell walls of gram negative bacteria | periplasmic space |
what bacterial form provides resistance to dehydration, heat, and chemicals | (endo)spore |
what bacterial structure forms attachment between two bacteria during conjugation (transfer of DNA material) | (F or sex) pilus |
what bacterial structure is genetic material within the bacteria that contains genes for antibiotic resistance | plasmid |
what stain is required to see chlamydia | giemsa stain |
which organisms are not well visualized with gram stain | treponema (too thin), legionella (fatty acids), mycoplasma (no cell wall), mycobacteria (high lipid content) [also chlamydia and rickettsia] |
By what method are plasmids exchanged between bacteria | conjugation |
which exotoxin inhibits acetylcholine release at the neuromuscular junction, resulting in flaccid paralysis | botulinum toxin |
which exotoxin is a phospholipase that causes gas gangrene | alpha toxin of clostridium perfringens |
which exotoxin inhibits the inhibitor of adenylyl cyclase, causing whooping cough | pertussis toxin |
which exotoxin stimulates adenylyl cyclase, resulting in chloride and water entering the intestinal lumen, causing diarrhea | cholera toxin, heat labile ETEC toxin |
Which exotoxin destroys leukocytes | PV leukocidin and gamma-hemolysin of staph aureus |
which exotoxin is composed of edema factor, lethal factor, and protective antigen | anthrax toxin of bacillus anthracis |
Myelodysplastic Syndrome | A group of disorders caused when something disrupts the production of blood cells |
70yo M dies in a motor vehicle collision. Was undergoing evaluation for occult blood in the stool. Dx? | tubular adenoma |
38yo M with 1-week hx of watery, itchy eyes and a runny nose. Physical shows inflamed nasal mucosa. No congestion in lower lung. Pharmacotherapy? | Loratidine |
24yo M with small tender blisters on his penis 3 days after unprotected sex. Photograph shown. Causal agent? | Herpes simplex virus type 2 |
42yo F with 3-year hx of an intermittent facial rash, including the forehead, eyelids, nose, and cheeks. Rash seems to be getting worse since she moved from New York to Florida last year. Spicy foods precipitate a flushing reaction that seems to exacerba | Rosacea |
53yo M returned from Africa, has fever, headache, and abdominal discomfort. Received appropriate vaccinations prior to the trip. T 39.4C. A wright-stained peripheral smear shown. Dx? | Malaria |
68yo F with T2DM and hypertension that has even poorly controlled despite hydrochlorothiazide treatment. BP 150/96, Labs show serum glucose concentration of 130 and proteinuria. In addition to current Rx, which is most appropriate pharmacotherapy? | Lisinopril |
66yo M with stage IV colon cancer with 3-day hx of severe diarrhea after receiving chemotherapy with flourouracil, leucovorin, and irinotecan. perscribed opioid antidiarrheal agent with no CNS effects. Which med? | Loperamide |
35yo M in ED with 2-hour hx of sever fatigue and dizziness. Had profuse, watery diarrhea for 8 hours despite a lack of oral intake. Recently returned from a medical relief trip to a remove village in Honduras. T 36.7 C, P 122/min, BP 90/50. PE shows dry s | Activation of adenylyl cyclase |
59yo F with gradual onset of lack of muscle control in her left arm and leg. Sx 1 mo ago after dx with metastatic breast cancer. PE shows ataxia of left upper and lower extremities. Muscle strength, DTR, sensation, proprioception normal. Metastatic tumor | Cerebellum |
Newborn delivered at 38 weeks' gestation weighs 1800 g. PE shows petechial rash, microcephaly, and hepatosplenomegaly. Serologic test for CMV: IgG + in mother, + in newborn; IGM - in mother, + in newborn. Explanation? | CMV |
Female newborn is delivered at 38 weeks' gestation. Apgar 8 and 8 at 1/5 min. PE shows a bulging, fluod0filled mass approximately 5 cm in diameter in the midline over the lumbosacral region. No spontaneous movements of the lower extremities. Abnormality m | 15 to 40 |
64yo M in ED 3 hours after SOB with exertion and extreme fatigue. Has ischemic heart disease. P 125/min, BP 105/60. ECG shows atrial fibrillation. Intravenous ibutilide is administered. Ten minutes later, ECG shows normal sinus rhythm. Risk for which drug | Torsades de Pointes |
65yo F with 20-year hx of osteoarthritis of the hands now has pain radiating down the distal anterior thigh, knee, medial leg, and food. Bony outgrowth of vertebrae compressing one of the spinal nerves is suspected. Nerve root in which intervertebral for | L-3 to 4 |
38yo M with 3-year hx of T2DM. Taking an oral antihyperglycemic agent, he has tried diet and exercise. BMI 32. PE normal. Hb A1c is 10%. Physician recommends initiation of insulin injections. Responds, "I know that insulin would help control my blood suga | Contemplation |
24yo M with 2-day history of an itchy rash on his arms and legs. Returned from a camping trip in the woods 5 days ago. PE shows edematous, erythematous rash with linear vesicles. Cause is activation of which cell types? | T lymphocytes |
70yo M from china with poorly differentiated monoclonal carcinoma of the nasopharynx. DNA probes of neoplastic cells are most likely to detect genome of which virus? | Epstein-Barr virus |
24yo M with hx of IVDA could not be aroused. Friend reports that the patient injected himself with a drug 6 hours ago. Labs show drug concentration of 0.3. Assuming first-order one-compartment kinetics, has a half-life of 2 hours, and a volume of distribu | 480 |
Compound is taken up by bacterial cells. No energy is necessary for uptake, and the compound is not concentrated in the cell. Which describes this mechanism of transport? | Carrier-mediated diffusion |
Newborn has male genital ducts but female external genitalia. Cytogenetic analysis shows a 46,XY karyotype, and genetic testing shows a mutation of the gene encoding 5alpha-reductase. In absence of this mutation, labia majora would have been? | Scrotum |
Study designed to evaluate the efficacy of coenzyme Q10 in improving cardiac output in patients with CHF. Sixty patients with CHF are recruited. Each assigned by coin toss to one of tw groups. Design? | Randomized clinical trial |
12yo boy immersed up to his neck in 60F water for 20 minutes. Physiological changes? | Central blood volume DECREASED, ADH (vasopressin) INCREASED, Atrial Natriuretic Peptide INCREASED?? |
Tells patient he has lung cancer. The patient reponds, "How can this be happening to m? I eat right and exercise." Appropriate response? | "It must be difficult for you to accept this diagnosis when you feel healthy." |
18mo girl. Separation of the umbilical cord was delayed after birth. Has had four severe skin infections Staphylococcus aureus; No pus formation at infection sites. Persristent leukocytosis in absence of infection. Mechanisms impaired? | Leukocyte adhesion and transmigration |
32yo M with 6-month hx of low back pain and stiffness, worse in morning and improve during the day; the pain radiates to his buttocks but not down his legs. Back stiffness if he sits for prolonged periods. Which to confirm diagnosis? | X-rays of the sacroiliac joints |
17yo boy with 8-kg weight gain during the past year. No medical illness. BMI is 32. He asks, "Do you think that my weight gain is inherited from my father?" Appropriate response? | "Yes, your weight gain can be caused by genes and environment combined." |
52yo M neighbors have reported that he has been confused and not taking care of himself. 4-mo Hx of diarrhea. PE shows extreme muscle wasting, stomatitis, and a diffuse rash that is worse in sun-exposed areas. Diagnosis? | Pellagra |
80yo F with suspected temporal arteritis (TA). ESR is 100, Pretest probability for TA is 50% in this patient. In the evaluation of TA, ESR has a sensitivity of 99% and specificity of 60%. Based on the restults of the ESR testing, most appropriate next ste | Additional testing to confirm the diagnosis of TA |
Sequence surrounding the first two exons of the human beta-globin gene shown, with exons in bold. Translation start codon is underlined. A mutation from G-->A at position 355 is most likely to lead to beta-thalassemmia by which mechanism? | Disruption of normal splicing by creation of a new 3' splice site?? |
42yo M in ED because of a 10-day history of progressive fever, SOB, and nonproductive cough. 20-kg weight loss. Immigrated to USA from the Ivory Coast 4 years ago. T 38C. Lungs clear, CXr shows diffuse interstitial infiltrate. Silver stain obtained via br | Infection with HIV-2 |
27yo F with vaginal bleeding for 3 weeks. First pregnancy ended with a spontaneous abortion 8 months ago. No Rx since dilatation and curettage at that time. PE shows enlarged uterus, beta-hCG markedly increased. Ultrasonagraphy of pelvis shows material w | Trophoblastic tissue |
65yo M emigrated from Brazil with 8-month history of shortness of breath and fatigue, edema of lower ext. CXR shows cardiomegaly. endomyocardial biopsy specimen shows myofiber necrosis with a mixed inflammatory infiltrate of PMNs, T lymphos, m.phages, and | Trypanosoma cruzi |
40yo F with mole on her back that has increased in size during past 4 months. PE shows raised irregular lesion with variegated black-tan pigmentation and ill-defiined margins. Pleomorphic, hyperchromatic cells within clear islands that tend to collapse. E | Melanoma |
42yo farmer has a 7-mm red scaly plaque on helical rim of left ear. Photomicrograph shown. Dx? | Actinic keratosis |
30yo F training for a marathon, running 20 mild/day. Fasting glucose is 60. After her glucose stores have been depleted, which organ, in addition to liver, releases glucose? | Kidney |
Male newborn at 28 weeks'. Given ventilatory support with up to 80% oxygen for the next 72 hours, but dies of resp. failure. Cause is inadequate secretion from which labeled cell types? | Not sure about this one. Looking for surfactant. |
9yo girl with poor growth during the past year. < 3rd %ile for height and at 10th percentile for weight. PE normal. Visual field testing shows bitemporal hemianoia. Labs show GH deficiency. MRI shows calcified cystic mass in suprasellar region. Tumor deri | Diverticulum of the roof of the embryonic oral cavity |
28yo F G1P1 with 2-day history of a painful mass in her right breast. Delivered healthy female newborn 3 weeks ago, and been breast0-feeding since. T 37C, PE shows 3-cm tender mass surrounded by an area of erythema beneath the right areola. Causal org? | Staphylococcus aureus |
3yo boy with bacterial colitis caused by Salmonella enterica serovar arizonae. Which factor accounts for recruitment of PMNS to inflammatory site by intestinal epithelial cells? | Interleukin-8 (IL-8) |
27yo F with fever, malaise, abdominal pain, and vaginal d/c for 4 days. LMP 5 days ago. Had ectopic 1 year ago. T 38.3 C, bilateral lower quadrant tenderness with rebound and guarding. Pelvic exam with cervical motion tenderness and bilateral adnexal tend | Gonorrhea |
20M with 3-month hx of progressive thirst (drinking a lot of fluids) and urinary frequency during past 3 days. U/A shows specific gravity less than 1.006. Most likely has dysfunction of which endocrine structure? | Posterior pituitary gland |
21yo M in ED with excruciating anal pain for 4 hours. Exam shows 15-mm, blue tinged rounded mass at anal margin. Represents thrombus in a tributary of which blood vessel? | Inferior rectal vein |
6-week-old girl with 6-day hx of vomiting small amount of milk 2 to 3 times daily. 50 %ile for length and weight. Cause? | Immature lower esophageal sphincter |
37yo M with 4-day hx of diarrhea and abdominal pain, worse in past 24h, with watery-brown stools. Completed a 10-day course of amoxicillin for a sinus infection 5 days ago. Stool shows: Fecal fat negative; ova and parasites negative; Cx for infx negative; | Pseudomemnbranes of fibrin and inflammatory debris |
28yo M in ED 30 minutes after SOB. 3-year hx of cocaine abuse. T 38.1, P 100/min, BP 150/45. PE: diminished pulses in left upper extremity. Crackles heard over all lung fields. 2/6 diastolic murmur at left sternal border. CXr shows a widened aortic arch. | Dissecting aneurysm |
1-week-old girl. screening showed a possible defect in fatty acid oxidation. PE normal. Next step? | Measurement of serum acylcarnitine concentrations |
79yoM 30 minutes after LOC for 30 seconds. Alert, but dizzy. No urinary or fecal incontinence. Pulse 40/min, BP 92/56. PE shows no tongue biting. Lungs clear, Variable intensity S1. Oriented X3. ECG shows a third-degree atrioventricular block. Next step? | Insertion of transvenous pacemaker |
55yo M with chronic bronchitis in ED after being unresponsive. Found bottles of albuterol, ampicillin, codeine, and theophylline bedside. T 37.2 C, p 112/min, respirations are 6/min, BP 95/60. Acute Rx should include? | Naloxone |
50yo man has persistent cough for 2 months. He has had a 5 kg (11 lb) weight loss. He is a farmer and on itraconazole 4 weeks for histoplasmosis from chicken coops. Medications: hydrochlorothiazide, enalapril, atenolol, omeprazole, and metoclopramide for | Omeprazole |
20yo man with 6-hour difficulty breathing and vomiting. 10-year history of type 1 diabetes on insulin. Pulse 90, respirations 30 and deep, bp 90/60. Physical shows dehydration. Labs: Na+ 130, K+ 6.5, HCO3 5, glucose 500, pH 7.2, pCO2 25 mm Hg. Which compo | Epinephrine |
Physician sad to inform patient of progression of carcinoma to the terminal phase. Physician's face makes patient cry and ask, "It's bad news, isn't it?" Which is best response? | "Yes, it is." |
75yo man 2-year history of decreased force of urinary stream, urinate several times throughout night. BUN 55, Cr 5.0. Ultrasound shows bilateral hydronephrosis and dilated ureter. Mechanism of renal failure? | Increased hydrostatic pressure in Bowman space |
4yo boy two bacterial urinary tract infections past year. Physical exam normal. Radiologic studies show dilation of left ureter and renal pelvis, minimal left-sided renal function. Left nephrectomy done. Photo: dilated renal pelvis and ureter. Which is ca | Congenital ureteral obstruction |
45yo man poorly controlled type 2 diabetes 1-month low-grade fever. Getting hemodialysis for end-stage renal disease. T 37 C (98.6 F), pulse 72, bp 144/92. Physical subclavian catheter below right clavicle. Lungs clear. Cardiac exam no murmurs. Blood cult | Enterococcus faecalis |
3yo boy sickle cell disease with fever and pain over left foot 3 weeks. Hematocrit stable. Leukocyte count 15,000 predominance of neutrophils. Which is most likely explanation for findings? | Osteomyelitis |
45yo woman follow-up exam after 8 weeks tamoxifen therapy for estrogen- and progesterone-positive invasive ductal carcinoma of breast. 50yo sister also hormone-sensitive breast cancer. Physical exam normal. Serum decreased concentrations of endoxifen, act | 25% |
60yo man progressive shortness of breath past 3 months. Worked in shipyard. Respirations 25. Bilateral basilar crepitant crackles. Xray chest reticulonodular pulmonary infiltrates consistent with interstitial fibrosis. Picture: sputum sample of elongate s | Alveolar macrophage |
30yo woman 20 weeks gestation, uncomplicated pregnancy. Fundal height is greater than expected for gestational age. Ultrasound shows increased amniotic fluid. Which abnormality is cause? | Tracheoesophageal atresia |
27yo woman 12-hour history of fever and abdominal pain. History of recurrent urinary tract infections. Temperature is 39 C (102.2 F). Physical exam tenderness of right flank. Abdominal xrays bilateral staghorn renal calculi. Urinalysis pH 8, many RBCs, WB | Struvite |
48yo nulligravid woman with excessive uterine bleeding for 3 months. Bleeding during menses and at irregular intervals. Menses were regular before. BMI 27. Pelvic exam: adnexae are nonpalpable. Endometrial curettage shows abundant tissue. Which is cause o | Endometrial hyperplasia |
55yo woman 6-week history low energy, irritability, crying spells, difficulty falling asleep, wakes up during night, cannot focus. Taking lorazepam for 15y for generalized anxiety disorder. Taking estrogen replacement therapy for postmenopausal symptoms. | Major depressive disorder |
Ten years after total gastrectomy, 60yo man difficulty walking. Diffuse spasticity in arms and legs, impaired proprioception in his feet, increased muscle stretch reflexes in arms and knees, absence of muscle stretch reflexes in ankles, bilateral extensor | B12 (cobalamin) |
80yo man type 2 diabetes 2-month history severe constipation. Laxatives haven't relieved symptoms. Abdominal exam shows distention. Colonoscopy shows no abnormalities. Patient has dysfunction of which nerve? | Pelvic splanchnic |
34yo man lightheaded after running 12 miles of marathon hot day. Pulse 130 bp 80/60. Which changes to autonomic nervous system occurred? | Sympathetic efferent activity increased, parasympathetic efferent activity decreased |
6-Mercaptopurine (6-MP) used to treat acute lymphoblastic leukemia (ALL). 6-MP acted on by enzymes to make 6-thioguanine nucleotides (6-TGN). Efficacy and toxicity of 6-MP correlated with 6-TGN. 6-MP acted on by xanthine oxidase (XO), thiopurine methyltra | They may be given normal doses of 6-MP (?) |
Ten human subjects given new oral drug to monitor drug effect and toxicity. Blood analyzed for human pharmacokinetics of drug for first time. Which trial type? | Phase I |
15yo girl health maintenance exam. Mother dx squamous cell carcinoma face, maternal grandfather died of metastatic melanoma. In patient this age, which factor most predicts compliance with photoprotection? | Macrophages |
44yo woman follow-up after two Pap smears showing atypical squamous cells of undetermined significant. Test shows viral E6 protein of human papillomavirus. This protein promotes cell growth and malignancy by causing cellular p53 protein degradation. This | Ubiquitin ligase |
40yo woman hx of 6 month episodic sinusitis with 2-week intermittent headaches, fatigue, and generalized joint pain, worsening cough productive of blood-tinged sputum. Failed antibiotics, decongestants and nasal corticosteroids. Physical exam: erythema na | Wegener granulomatosis |
25yo woman 6-month history of joint pain poorly responsive to aspirin. Physical exam: bilateral swelling of proximal interphalangeal joints, metacarpophalangeal joints, and wrists; weakness of grasp. Small nodules palpated beneath skin around joints of f | Rheumatoid Arthritis |
50yo woman 1-year hx of hot flashes and irregular menses. Decreased bone mineral density. Alendronate prescribed. Mechanism of drug? | Inhibition of osteoclast-mediated bone resorption |
Female newborn at 36 weeks gestation has respiratory distress. Apgar 3 and 5 at 1 and 5 minutes. Physical shows cyanosis. Endotracheal and NG tubes placed. Xray shows nasogastric tube in left hemithorax, mediastinum displacement to right, absence bowel ga | Incomplete formation of pleuroperitoneal membrane |
65yo women progressive vulvar itching past 2 months; miconazole for yeast infections ineffective. Exam: atrophy of labia minora and thin, parchment-like skin over vulva and anus. Dx? | Lichen sclerosus |
75yo man 2-day ear ringing, nausea, fatigue. Temp 37 C (98.6 F), pulse 100, respirations 24, bp 140/85. Physical: mild epigastic tenderness. ABG pH 7.42 pCO2 30 pO2 95 HCO3 19. Dx? | Salicylate poisoning |
15yo girl 1-day hx redness and painful skin following sunbathing. She used sunblock. No medications. Physical exam: severe erythema of back and extremities, no blisters. Dx? | First-degree burn |
58yo man supraventricular tachyarrhythmia refractory to pharmacotherapy gets ablation of accessory excitatory pathway in atrial endocardium. Which area should be avoided to leave sinoatrial (pacemaker) node intact? | The junction of the superior vena cava and the right atrium |
81yo woman massive pulmonary embolism from deep venous thrombosis. Platelet count 160,000. Appropriate pharmacotherapy is started. One week later, platelets 55,000. Thrombocytopenia most likely caused by a drug with which of the following mechanism of act | Potentiates the action of antithrombin III |
22yo woman, g1p1, 2-day hx of fever, severe vaginal bleeding. four days ago delivered healthy male newborn. Temp 38.1 C (100.6 F). Pelvic exam: open cervix, heavy vaginal bleeding. US shows uterus with no placental tissue or thrombi. If operation required | Internal iliac |
35yo woman abnormal Pap smear. Cervical biopsy shows microinvasive cervical carcinoma. Which microscopic features led to dx? | Neoplastic cells in sub-basement membrane connective tissue |
17yo girl 1-day shortness of breath, weakness and muscle tenderness. Did triathlon previous day. BMI 19. Temperature 38 C (100.4 F), respirations 20, bp 150/90. Physical bilateral crackles lower lobes, muscle tenderness. Creatinine 4. Urinalysis 3+ protei | Myoglobin |
16yo boy with no signs of puberty. Sex development Tanner stage 2. Physical exam: circumcised penis, soft small tests 5 mL, prostate firm, nontender, no discharge or lesions. Testosterone low. Which hormone is cause of decreased serum testosterone and lac | Luteinizing hormone |
28yo woman wants to lose weight. She binges on high-carbohydrate foods 2 to 3 times a week, forcing herself to vomit after. BMI 23. Which physical finding is likely? | Parotid gland enlargement |
15yo girl emigrated from India and with several lesions on neck for 2 weeks. Physical exam shows hypopigmented, hypoesthetic area on left side of forehead and 4-cm lesions on neck. Biopsy shows acid-fast bacilli. Best explanation why the organism results | Temperature senisitivity |
59yo man has total thyroidectomy for 4-cm follicular carcinoma of thyroid. Twelve hours after procedure, has paresthesias of hands and feet. Vitals stable, carpal spasm on inflammation of bp cuff. Lab findings? | Decreased parathyroid hormone, decreased calcium |
41yo man with asthma and allergy to grass pollen wheezes and difficulty breathing 10 min after mowing lawn. Drug for immediate relief of acute symptoms? | Albuterol |
14yo boy daily headaches for 2 months. Headaches are bilateral aching in temples. "Has not been himself" for months. Confused, forgetting names, dates, places, clumsy, frequent falls. School performance declined. Physical exam: broad-based ataxic gait. Sl | Inhaled glue |
40yo African American woman 2-week hx fever, malaise, dyspnea. Temperature 36.7 C (98 F), respirations 20. Physical exam: erythema nodosum, parotid enlargement, hepatosplenomegaly. Calcium 16. CT chest bilateral hilar adenopathy. Increased in which in ser | 1,25-Dihydroxycholecalciferol |
56yo woman with restrictive cardiomyopathy, proteinuria, renal failure. 35-year history of rheumatoid arthritis. Renal biopsy shows glomerular deposition of eosinophilic hyaline material. Congo red statin: birefringent pattern under polarized light. Struc | beta-pleated sheet structure |
63yo man 3-month hx difficulty sleeping. Sleeps better upright. HR 90, bp 110/60. Physical exam: increased jugular venous pressure, mild ankle edema. Cause of edema? | Increased capillary hydrostatic pressure |
14yo boy come to ER 1 hour after colliding with teammate playing soccer. Physical exam: edematous tissues of left eye, mild depression of left zygomatic bone. Skin between eye and upper lip numb. Double vision look upward. Nerve damaged causing sensory lo | Maxillary division of trigeminal nerve |
Maxillary division of trigeminal nerve | Family history of a similar illness |
18yo man Crohn disease 1-day hx severe abdominal pain and intermittent bloody diarrhea. Temperature 38 C (100.4 F), pulse 98, respirations 18. Physical exam: draining anal fisutla. Treatment with antibiotics and prednisone over next 3 weeks recovers. Mech | T-lymphocyte function |
34yo woman with pyelonephritis treated with bactericidal antibiotic 4 days no improvement. Antibiotic added that inhibits binding to 30S ribosome, blocking protein synthesis intracellularly. Antibiotic? | Gentamicin |
3yo boy and his 5yo brother with recurrent hemarthroses. Both parents healthy, but mother with two younger brothers with same sx and maternal uncle who died at 8 of mild head trauma. Partial thromboplastin time is prolonged. Defect? | Factor VIII (antihemophilic factor) |
6yo boy from Russia with unstable gait and incoordination for 2 weeks. Pale, bulky stools for 4 years and two episodes of bacterial pneumonia and chronic cough since age 1 year. 3%ile for height/weight. Neuro exam shows ataxia, no DTRs, loss of propriocep | Vitamin E |
54yo F 1 week after sudden loss of vision in left eye, returned within 1 day. 3-month hx of progressive SOB with exertion. Echocardiography shows mass in the left atrium of the heart. Lesion is resected, photomicrograph of it is shown. Which describes the | Myoxma |
27yo primigravid woman at 34 weeks' with nausea and vomiting, and abdominal pain for 12 hours. Everything's been normal. BP is now 164/102, and right upper quadrant tenderness. Labs show Hb 7.4, HCT 24%, Platelets 72k, Cr 1.2, total bili 2.3, AST 112, ALT | Schistocytes |
60yo F 3 hours after sudden onset ankle pain. 4-year Hx of increasing serum creatinine concentrations. Began furosemide 1 month ago, also takes glipizide. P 120/min, resp 25/min, BP 150/100. Joint fluid shows negatively birefringent crystals. Increased ri | Nephrolithiasis |
56yo F follow-up 8 weeks after recovering from pneumococcal pneumonia. Chest X-rays normal. Which allowed this resolution to occur? | Metaplasia of mesenchymal cells to pneumocytes |
29yo F with 5-week hx of fatigue and 4-day hx of heart palpitations and anxiety. Has primary hypothyroidism Rx with triiodothyronine, but she has doubled the dose because of fatigue. TFT will show which? | TSH DECREASED, Free thyroxine DECREASED, Free triiodothyronine INCREASED |
20yo F has multiple neurofibromas. Mom, uncle, and brothers with similar lesions. Mode of inheritance? | Autosomal Dominant |
45yo M with yellow skin. Drinks eight to ten 12-ounce cans of beers daily for 10 days. Liver is tender. Serum: total bili 5.9, Alk Phos 210, AST 110, ALT 69, gamma-glutamyltransferase 25 (n = 0-30). Liver biopsy will show? | Mallory Hyaline |
35yo M uses crack cocaine daily, with 2-hour Hx of substernal chest pain. T 37C, P 110/min, BP 160/100. Most appropriate next step? | Admit the patient to the hospital for possible myocardial ischemia |
54yo F with hypertension and bilateral renal artery stenosis starts taking NSAIDS for back pain. Her Cr concentration increases from 1.0 to 5.0. Cause is due to inhibiting which? | Vasodilating prostaglandins at the afferent arteriole |
83yo M brought to ED after being found at home bedridden and confused. No meds. P 100/min, BP 85/50. BP unchanged after 1L IV saline. Pulmonary artery catheter shows: Cardiac output high, PCWP low, systemic vascular resistance low. Cause of hypotension? | Early septic shock |
32yo M with 3-month hx of swelling and breast tenderness. Receiving thyroid hormone and steroid replacements since removal of pituitary adenoma 2 years ago. Began hCG injections 4 months ago. Most likely binding site of hCG causing gynecomastia? | Tissue: Testicle; Effect: estradiol production |
38yo F with 3-day hx of sore throat. Photo shown of throat. Which nerves is tested by saying "ah," elevating area at tip of the arrow? | Vagus |
8yo boy with disruptive behavior, interrupts, always moving, trouble completing tasks. Drug with which mechanism is appropriate? | Increased release of dopamine and norepinephrine |
62yo M with alcohol-induced liver disease develops ascites. Infection ruled out. Most appropriate diuretic, in addition to loops, is which? | Spironolactone |
17. 72yo M with weakness and fatigue. Hemogrlobin concentration is 9.2, WBC 5400, platelets 350k. Peripheral blood smear is shown. Cause? | GI blood loss |
65yo F with widely metastatic breast cancer unresponsive to chemo. No family. "close friend," at all her visits and now she is moved to inpatient hospice after she decides she wants no further curative therapy. Says, "We can't bear to be apart. It would | "The two of you seem to have a very important relationship. Of course you may stay together." |
46yo M treated with oral cyclosporine after cardiac transplant. Cyclosporine decreases likelihood of rejected by which actions? | Suppressing the early response of T lymphocytes to activation |
30yo M in ED 15 minutes after found unconscious. Comatose, pupils 4 mm in diameter, not reactive to light. CT head shown. Cause of coma is bleeding from which structures? | Middle meningeal artery |
68yo M with 6-month hx of erectile dysfunction. PE and labs normal. If pharmacotherapy is indicated, drug with which MOA? | Inhibition of phophodiesterase |
35yo M with recurrent sinusitis and bronchitis. Cardiac examination shows PMI at fourth intercostal space within the midclavicular line on the right. Hepatic margin is palpable on the left. Endoscopy shows nasal polyps. Biopsy shows thickened, ciliated, p | Dynein arms |
30yo primi at 22 weeks' gestation with 1-day hx of fever, chills, and muscle aches. T 39.4, P 114/min, resp 15/min, BP 104/72. PE shows uterus consistent with 22-week gestation. Fetahl heart sounds are heard. WBC 12K, Blood cultures grow gram-positive rod | Listeria monocytogenes |
42yo M with multiple lesions over his body. PE shows flaccid bullous erosions involving upper and lower extremities and torso. Biopsy shows extensive epidermal acantholysis resulting in the formation of intraepidermal blister. Intact basal layer of kerat | Development of autoantibodies against desmosomal proteins |
50yo M 3 days after his first generalized tonic-clonic seizure. 1-month hx of frequent episodes of pins-and-needles sensation around the mouth, hands, and feet, involuntary contraction of muscles. Neuro exam shows mild, diffuse hyperreflexia. Which serum | Calcium |
52yo F with hot flashes. Menses have been irregular for the past 6 months. Physiologic cause? | Failure of the ovaries to secrete 17beta-estradiol |
32yo F G2P1 at 7 weeks' gestation with vaginal bleeding for 3 days and increasingly severe left abdominal pain for 18 hours. Direct and rebound tenderness with guarding in left lower quadrant. Cervical os is closed. serum beta-hCG is 6000. U/S shows empty | Ectopic pregnancy |
26yo F 5 weeks after birth of first child. Worries constantly that the infant is ill and wakes up to make sure he is well. Washes her hands 30 times per day. Worried about people braking into her house, checks lock 3-4 times a night. Not breast feeding. R | Sertraline |
10yo boy has had anemia since birth. Spleen is five times normal. Splenectomy is indicated if anemia is caused by which? | Hereditary spherocytosis |
62yo F in ED for 2-day hx of fever, abdominal tenderness, and painful urination. Agitated. T 38.8C, Labs show WBC 14k. Admitted to hospital, nurses note she has torn up four breakfast menus because she is confused. Cause? | Delirium |
60yo F with 3-year Hx of hyperlipidemia. Low-cholesterol diet and exercise program ineffective after 1 year. Lovastatin initiated, but unable to tolerate greater than 20 mg daily. Additional drug is added that inhibits transport of cholesterol through int | Ezetimibe |
48yo M with bronzing of his skin, weakness, and fatigue during the past 3 months. PE shows hepatomegaly, and small testes. Serum: AST INCREASED, ALT INCREASED, iron INCREASED, transferrrin sat INCREASED, ferritin INCREASED, testosterone DECREASED, LH DECR | Increased intestinal iron absorption |
45yo M with intermittent bloody diarrhea and abd pain. Sigmoidoscopy and rectal biopsy show IBD. Monoclonal antibody is begun, which is directed against what components? | Tumor necrosis factor |
"string of beads" sign Dx? | Fibromuscular dysplasia |
55yo M with sepsis. Appears anxious and confused. Rx with vancomycin and ceftriaxone initiated in ED. T 39.8, P 132/min, BP 85/48. PE shows warm, flushed skin. No aedema. Administrer which solutions? | 0.9% Saline |
36yo M undergoes elective liposuction under general anesthesia. Operation is terminated when patient develops hyperthermia, tachycardia, and marked muscle rigidity. MOA of drug that should be administered? | Decreases release of Ca from the sarcoplasmic reticulum |
In a survey of 100 households (average three residents per household), 45 with asthma are detected. Prevalence? | 15% |
While lifting weights, 24yo M swelling in right inguinal region. Photograph shown of small intestine resected. Dx? | Strangulation |
24yo with second-degre burn. Two weeks after, tissue shows increased fibroblast migration and proliferatoin, increased collagen and fibronectin, and decreased metalloproteinases. Caused by production of which? | Transorming growth factor-beta |
Protein found in brown adipose tissue of mice causes leak of H ions inward across inner mitochondrial membrane. Effect of this protein on oxidative phosphorylation and energy metabolism? | Increased ratio of oxygen consumption to ATP generation |
57yo M with alcoholism has distended abdomen with shifting dullness, fluid wave, caput medusae, palmar erythema, spider angiomata. Additional finding? | Gynecomastia |
16yo girl with 2-year hx of fainting; increased in frequency during past 6 months. BP 110/80 supine and 60/40 standing. Neuro exam normal. Plasma shows undetectable noreipinephrine and marked increase in dopamine concentration when standing. Deficiency of | Dopamine beta-hydroxylase |
HALLMARK: Peanut Farmer from China | Aflatoxin |
42yo M in ED for 5-hour hx of fever, chills, and severe pain and swelling of his left arm. Scratched his arm on a nail yesterday. Appears confused, T 40C, BP 71/38. Labs show Hb 14, HCt 42%, WBC 15K (35% PMNs, 40% bands, 25% lymphos), Platelets 50K, Serum | IL-1 and tumor necrosis factor (TNF)-alpha |
62yo M dies suddenly while playing tennis. No cardiac risk factors, no hx of CAD. Autopsy, cardiac valve defect and concentric LVH. Which valve abrnomalities is most likely involved? | Aortic stenosis |
67yo M has urinary urgency after placement of urinary bladder catheter during transurethral resection of the prostate. Most appropriate Rx has which MOA? | Inhibition of muscarinic receptors |
Mouse embryos are produced with two pronuclei, both of same parental origin. When the pronuclei are maternal, produces have poorly developed extraembryonic structures. When both pronuclei are paternal, produces have poorly developed embryonic tissue. Whic | Imprinting |
18-yo F with sepsis after an abortion. Within 24 hours she becomes dyspneic, oliguric, and develops petechiae, ecchymoses, and bleeding from venipuncture sites. Which lab finding? | Decreased plasma fibrinogen concentration |
63yo M with 6-month hx of exertional chest pain relieved by rest. smoked for 45 years. Mild HTN, no meds. Which lesion in LAD is most likely cause? | Calcified 80% stenosis |
65yo F with ovarian cancer treated with cyclophosphamide and other chemotherapeutic agents. Cyclophosphamide affects which target? | DNA replication |
Pharm co trying to develop a long-acting weight-loss agent that mimics activity of a naturally occurring peptide originates in adipose tissue, signals brain about stored fat, and suppresses appetite by its action in the CNS. Which chemical mediator? | Leptin |
45yo F has thyroidectomy because of asymmetric enlargement of thyroid noticed 6 weeks ago. Underwent adrenalectomy for pheochromocytoma 3 years ago. Bilateral thyroid lesions with spindle cells arranged in small clusters. Amyloid deposits b/w neoplastic | Calcitonin |
45yo F intubated, mechanically ventilated with fungemia with Candida albicans. Rx with caspofungin is started. Feature of causal organism targeted by this drug? | Beta-Glucan carbohydrates in the cell wall |
56yo M 4 hours after sudden onset of uncontrollable irregular movements of the left side of the body. PE shows flailing movements of the proximal appendicular muscles on the left. Nuclei damaged? | Right subthalamic |
60yo M 1-month Hx of progressive SOB with exertion. Breath sounds: Dec on right lung base, normal on left lung base Percussion note dull on right lung base, nml on left lung base Tactile fremitus decreased on right lung base, nml on left lung base Adv | Pleural effusion |
68yo M with difficulty swallowing solids for 2 months. Hx of dilated cardiomyopathy. X-rays of esophagus w/ barium contrast show indentation and posterior displacement of the esophagus. Enlargement of what caused dysphagia? | Left atrium |
25yo F with 3-year hx of irregular menses. Menarche was at age of 14 years. BP 116/62. PE shows increased hair growth on the face and chest. Pelvic exam shows clitoromegaly and a normal-appearing uterus. Serum shows increased 17-hydroxyprogesterone and an | 21-hydroxylase |
60yo M in ED for sudden onset of acute abdominal pain and tenderness, nausea, vomiting, and bloody diarrhea 2 hours ago. He has a Hx of cirrhosis and hepatocellular carcinoma. BP 99/50. Loss of bowel sounds. Surgery shows small intestine with dark purple- | Mesenteric venous thrombosis |
38yo M in ED 30 min after unable to stand upright. Lethargic, pulse 110/min, BP 90/62. PE shows dry mucosa and poor skin turgor. Midepigastric tenderness. Labs show: Serum: Na 143, K 3.2, Cl 101, HCO3 11 ABG: pH 7.28, Pco2 23, Po2 98 Acid-base status | Metabolic acidosis |
2mo boy given vaccine to convert T-independent antigens to T-dependent forms to enhance protection in young children. Which vaccine given? | Haemophilus influenzae type b |
A study conducted to assess effectiveness of injections of lidocaine into "trigger points" of pain symptoms in patients with fibromyalgia. Fifty patients randomly assigned - 0.9% saline only or saline plus lidocaine. Graph shows self-reported pain scores. | Placebo effect |
In a clinical study, a polymorphic marker with three alleles, 1, 2, and 3, is found to be tightly linked to the gene for polycystic kidney disease. Pedigree shown. If III, 1 is unaffected by this disease, patient is most likely carrier of? | 2,3 |
6yo girl with 4-day hx of round shiny bumps in areas where she has eczema. Her mother saw similar bumps on a playmate at pool party 3 weeks ago. No other Sx. PE shows firm, smooth, umbilicated papules 2 to 4 mm diameter in clusters. Causal organism? | Poxvirus |
80yo F in ED for 2-day hx of "feeling funny." "Lost my pep." Hx of poorly controlled hypertension. Just started medication 2 weeks ago. BP 130/85. Pe normal. Serum potassium is 3. Which drug? | Hydrochlorothiazide |
68yo M in for a hemiorrhaphy. Surgeon gives info of risks and benefits. Patient says that he understands what he has been told, and his family will be able to discuss later. In this patient, which combination of components fulfill the criteria for fully i | Information, competence, voluntariness |
47yo F with psoriasis for follow-up. Was given several topical creams, to be used in specified sequence twice daily. No improvements apparent at this appointment. How to begin discussion of compliance? | "using something twice daily can be difficult. I assume you are like most patients who miss at least 10% of treatments." |
Male newborn has macrocephaly with poor skull mineralization, shortened extremities with misshaped long bones, and several fx. Defect in which? | Collagen |
27yo F in ED 30 min after ejected through windshield during MVC. Unrestrained front-seat passenger. PE shws marked edema and tenderness of the jaw. Panorex x-ray of mouth shown. Which is injured? | Levator labii superioris |
Girl for well-child exam. Normal development includes pincer grasp, finger feeding, standing while holding onto a table, and playing peekaboo. Age (in months)? | 9 |
18yo M with yellow nodules on achilles tendons of his feet and extensor tendons of his hands. Exam shows collection of foamy histiocytes within the dermis. Serum cholesterol is 980, and lipoprotein electrophoresis shows a selective increase in LDL. Underl | Absence of functional LDL receptors in hepatocytes |
35yo M with 4-day Hx of high-grade fever, sever muscle aches, malaise, loss of appetite, and a nonproductive cough. wife and kids had similar illness. Temp 39.2, PE normal. CBC and CXR normal. Causal virus replicates its genome within the cell's nucleus. | Influenza virus |
28yo F at 18 weeks' gestation has palpitations. Labs show increased serum total thyroxine (T4) concentration. Best test to confirm hyperthyroidism? | Thyroid antibodies |
21yo M in ED 45 minutes after sustaining multiple injuries in a MVC. His BP is 90/50, PE shows diffuse abdominal tenderness. Dx with laceration of the spleen and undergoes splenectomy. Predisposed to infection with? | Streptococcus pneumoniae |
70yo F in longitudinal study of effects of aging on pulmonary function tests. Which represents woman now compared with results at age of 20 years? | Residual volume UP, Arterial Po2 DOWN, Alveolar-arterial Po2 difference UP |
45yo F farmer in ED for 2-day hx of confusion, lethargy, fever, headache, muscle pain, vomiting, and a rash on her wrists and ankles. Bitten by a tick a few days ago. T 38.5 C, Red-purple papules on distal extremities progress to trunk. Rx? | Doxycycline |
Study of breast cancer in women. Hundred healthy women observed for 10 years. Goal is to determine if number of family members who previously received dx of breast cancer correlates with incidence of future development of cancer. Best design? | Cohort |
62yo F with recurrent pulmonary emboli comes for follow-up. PE normal. Labs show PT of 12 seconds. Warfarin begun. Which clotting factors is first to be decreased by 50% after initiation of Rx? | VII (proconvertin) |
48yo F with 2-month hx of fatigue and intermittent headaches. BP 180/110, PE normal. Serum show a decreased potassium concentration and increased aldosterone. CT abdomen shows tumor on adrenal gland. Which additional findings supports aldosterone-secretin | Decreased plasma renin activity |
20yo F with 1-day hx of increasing urinary frequency and a burning sensation with urination. One sexual partner, uses condoms. VSS. PE shows mild suprapubic tenderness to deep palpation. Urine shows rare epithelial cells and 10 WBC/hpf. Urine grows E. col | Mannose-binding (type 1) fibria |
6yo boy with 3 systemic infections with Neisseria meningitidis over the past 2 years. Healthy otherwise. Which lab test is most likely abnormal? | Total hemolytic complement concentration |
13yo girl 2/6, holosystolic murmur heard best over left fifth intercostal space adjacent to the sternum; it increases with inspiration. Abnormality of which valves? | Tricuspid |
55yo F with left flank pain and gross hematuria. Mass is palpable in LUQ of abdomen. Ultrasonography shows a 12-cm solid mass on lower pole of left kidney. Angiograms show hypervascular mass. Dx? | Renal cell adenocarcinoma |
19yo M in MVC. Penetrating wound to right cerebral cortex with paralysis of the left lower extremity, fracture of right mid humerus with severing of the radial nerve, and a fracture of right tibia. After 10 weeks, DTR strongest in which locations? | Left Achilles tendon |
40yo F with 1-year hx of episodes of crampy abdominal pain, intermittent diarrhea, and rectal bleeding with passage of mucus. BMI 18. Abdomal exam: diffuse tenderness with no rebound. Sigmoidoscopy shows diffuse ulcers. Initial Rx? | Sulfasalazine |
27yo M for employment exam. No Hx of major illness. Never been sexually active. Minimal contact with parents and siblings, no hobbies. Doesn't feel depressed. Shrugs in response to congratulations about his new job. Flat affect. Personality disorder? | Schizoid |
30yo F has ptosis, ophthalmoplegia, and diplopia. Serum shows autoantibody with affinity for acetylcholine receptor at the postsynaptic neuromuscular junction. Which neoplasm? | Thymoma |
Exam scaled so scores are normally distributed with mean of 500 and SD of 100. Which % are between 400 and 600? | 67% |
62yo M with pericardial friction rub 3 days after acute myocardial infarction. Cause of rub? | Fibrinous pericarditis |
42yo F with 1-mo Hx of abdominal pain, after eating fatty meals. BMI 31. PE shows jaundice and tenderness of RUQ. Increase of which liver function? | Cholesterol synthesis |
70yo M with recent loss of mental function. Hx of weight loss. No drugs. VSS, not dehydrates. Mild anemia. Labs show Na 110, Cl 85, K 4.4, BUN 15, Cr 15; Plasma osmolality 250; Urine osmolality 750. Dx? | Pulmonary neoplasm |
40yo M skin extremely sensitive to sunlight, which causes formation of vessicles and blisters on the skin which take weeks to heal. Diagnosed with disorder caused by increased synthesis of compounds in the skin that are subject to excitation by visible li | Heme Synthesis |
60yo M with 6-month hx of fatigue. Four years ago, had subtotal gastrectomy after gunshot wound. Drinks 6-8 beers daily. PE shows paresthesias of both hands. Labs show: Hb 8, HCT 24%, MCV 115, WBC 5k, Platelets 165k, RBC Folic acid 500 (N = 125-600), B12 | Pariteal Cells |
40yo M with interstitial pulmonary fibrosis has greater maximal expiratory flow rate than predicted. Which best explains this? | Increased radial traction on airways |
20-year-old F secretary with 8yr history of intermittent headaches. Flashing lights in her right visual field, followed 20 minutes later by a unilateral throbbing headache accompanied by nausea and vomiting. occur around time of menses. Dx? | Migraine |
70 yr old african american women come to physician after 1 day onset of back pain. She's a part time cashier, low income, and smoked 1/2 a pack for 50 years and drinks 3 caffeinated beverages a day. X-ray shows vertebral compression fracture of L3 and she | Gender |
5-year-old-boy with mental retardedation is grossly obese and has facial features of Prader-Willi syndrome. Karytoyping and flourescent in situ hybridization studies do not show deletion in the usual site. Which to confirm PWS? | Maternal Origin of Chromosome 15 |
83yo M from Puerto Rico with weight loss and abdominal pain and blood in his stool for 1 month. Possible colon cancer. Poor english. Needs colonoscopy but family doesn't want to hear bad news and wants to make decision for about his treatment. Next step? | Use a Spanish-speaking interpreter to determine how much the patient wishes to know about diagnosis and treatment |
38-year-old F undergoes laparoscopic cholecystecomy with general anesthesia. Afetr she awakens postoperatively, she is nauseated and vomits threee times in 20 minutes. Treatment for N/V? | Ondansetron |
During an experimental study of oxygen consumption in the kidney, experiemtnal animals are ventillated with 100% nitrogen. Cells from which of the following areas of the kidney first show signs of anoxic injury? | Distal convoluted tubule? |
7-year-old girl 30 minutes after being hit in the mouth with basketball. Something stuck in her throat. Part of one tooth is missing. Lateral x-ray of the neck and chest is shown; arrow shows part of the tooth. Greatest risk for aspiration into which lobe | Right Lower |
Poliovirus mRNA lacks a 5' m7G cap but is translated efficiently by cellular ribosomes. Which of the following additional structural features of poliovirus mRNA is the most likely cause of its ability to be translated in the absence of a cap? | Presence of an internal ribosome entry site |
67-year-old F brother and mother have history of colon cancer. Physician recommends colonscopy, but patent only wants her stool to be tested for blood. Most likely concerned about which of the following regarding this test? | Low specificity |
69-year-old African American woman has moderate hypertension and type 2 DM. BMI 31. On hydrohlorothiazide. Labs show microalbuminuria. Most appropriate to add which drugs? | Irbesartan |
36-year-old man with 2-hour history of pain and swelling of his right calf. No shortness of breath or chest pain. 18-hour airplane trip 4 hours ago. Noninvasive vascular studies show an occlusion of right femoral vein. Immediate therapy has which mechanis | Activation of antithrombin III |
64 yr old alcoholic man with 1 day of confusion. Disoriented, disheveled. Dehydrated, jaundiced. and has spider angiomata over face and chest. Has flapping up and down of the hands when his arms are outstretched. Abdominal distention and bulging flanks. H | Killing of bacteria in the gut that generate ammonia |
1 year old boy is brought in. Has white, pale hair that hasn't changed color since birth. His eyes are blue. During opthalmic examination, the patient turns away from the flashlight and starts crying. Which of the following is the most likely cause of the | Inability to make melanin |
2-year-old boy with developmental delay. Hx of hearing loss in mother and delayed speech in older sister. Maternal uncle had stroke-like episodes at the age of 25 years. Physical shows ophthalmoplegia and hypotonia. Lactic acid concentration increased. E | Heteroplasmy |
Male newborn at 28 weeks' is tachypneic and hypoxemic. Which altered structure changes in the type II pneumocytes is the most likely cause? | Decreased numbers of lamellar bodies |
16-year-old girl with cystic fibrosis with 3-week history of generalized weakness, numbness and tingling of her arms and legs, and difficulty walking. Not adhered to medications. Bilateral weakness and decreased deep tendon reflexes in the upper and lowe | Vitamin E |
48-year-old woman with 6-month hx of irregular menstrual periods and hot flashes. LMP 35 days ago, and had scant blood flow. Menses had previously occurred at regular 28-day cycles. Mild thinning of the vaginal tissue. Labs most likely to show which of th | Decreased follicle-stimulating hormone |
62-year-old M with unstable angina pectoris undergoes coronary catheterization. To visualize the anterior interventricular (left anterior descending) artery, the tip of the catheter would need to be placed into the orifice of which arteries? | Left coronary |
35-year-old F with intermittent sharp chest pain exacerbated by deep breathing, and can be decreased by leaning forward. 2-month hx of pain and swelling in her hands and knees accompanied by morning stiffness that lasts 1 hour. Pulsus paradoxus less than | Pericarditis |
60-year-old M with a systolic murmur is a heard, which is loudest at the point indicated by the X in the diagram. Which cardiac abnormality is the cause? | Aortic valve stenosis |
31-year-oldwoman with invasive squamous cell carcinoma of the cervix. Biopsy shows tumor cells express human papillomavirus, type 16 antigens. Which cell types plays a role in recognizing and killing these virus-infected tumor cells? | T lymphocytes |
An 85-year-old woman is diagnosed with a fracture of the right femur and begins treatment with morphine by patient-controlled analgesic pump. Three days later, her respirations are 6/min. Physical examination shows pinpoint pupils. Her serum creatinine co | Morphine is metabolized to active metabolites that accumulate |
35 y/o man with 3 yr history of enlarging nose, coarse facies, muscle weakness, increased hand/foot size. Large fleshy nose and prognathism on exam. High IGF-1 in serum. MRI shows pituitary adenoma. Morphologic analysis of the tumor shows a densely granul | Adenlyl cyclase |
30-year-old woman with Li-Fraumeni syndrome found to have adenocarcinoma of the breast. Family history includes osteosarcoma. Which mechanisms underlies this condition? | Impaired regulation of apoptosis |
56-year-old with palpable hard nodule on prstate has increased serum prostate-specific antigen concentration. Fine-needle biopsy specimen shows adenocarcinoma. Patient undergoes radical prostatectomy. Which structure is at greatest risk for injury during | Pelvic parasympathetic nerves |
56-year-old woman frequently burned herself while cooking. Loss of pain and temperature sensation in both upper extremities and portion of her trunk from clavicles to just below the nipples. Touch, vibratory sensation, and proprioception normal. Findings | Syrinx of the central region of the spinal cord from C-4 to T-5 |
25-year-old man comes to the physician 8 hours after the onset of severe pain of his low back that radiates down his left leg. He started a weight-lifting regimen earlier in the day during which he tried to lift a bar loaded with 91 kg (200 lb) from the g | Rupture of an intervertebral disc |
43-year-old man with 6-week hx of sharp, stabbing pain on the left side of his face that occurs when he touches it. Pain when shaving. Pain just lateral to the left nasal ala reproduces the pain. Nerve supplying this area exits the skull through which for | Rotundum |
33-year-old man with 3-month hx of muscle weakness and cramping, appears shortly after he begins exercising. Serum creatine kinase increased. Venous blood from antecubital vein show lactate concentrations do not increased compared with preexercise values. | Glycogen phosphorylase |
67-year-old man with 2-month hx of weight loss. Has type 1 DM, gallstone removal 12 years ago, smoked 1 pack daily for 45 years. BMI 34, Calcium concentration of 11 mg/dL. Abdominal CT shows a pancreatic mass, biopsy shows pancreatic adenocarcinoma. Stron | Cigarette smoking |
28-year-old woman with 1-week history of fever and chills. 4.5-kg weight loss, 5-year hx of chronic sinusitis. Mildly distressed. BMI 18. Temp is 39 C (102.2 F). Has markedly diminished nasal septa. Chest x-ray shows multiple pulmonary nodules. Serologic | Wegener granulomatosis |
52-year-old man with 3-week history of increased thirst and urinary frequency; 4.5-kg weight loss. Has hypertension and hyperlipidemia treated with pravastatin and metoprolol. BMI 34. Which is most likely to be increased? | Fasting serum glucose concentration |
48-year-old man with possible hypertension. On basis of ten measurements, the patient's average diastolic blood pressure is 113, and standard deviation is 8. If four rather than ten measurements are made, which is the expected impact on 95% confidence int | Increase in width |
66-year-old M with 6-month history of decreased exercise tolerance and shortness of breath with exertion. Vitals stable. Auscultation shows bilateral basilar crackles. Cardiac examination shows S3 gallop. Grade 2/6 holosystolic murmur heard best at the ap | Dilated cardiomyopathy |
29-year-old woman for advice on losing weight. Has been taking thyroxine for several months in attempt to lose weight. Her thyroid function is normal. Which findings is most likely on histopathologic eam of the thyroid gland? | Follicular atrophy |
44-year-old man with 2-month history of abdominal pain and diarrhea. Pain relieved after eating and antacids. EPigastric tenderness. Serum gastrin concentration of 500 pg/mL (N=50 - 100) and gastric acid secretion of 80 (N=6-40). Most definitive treatment | Surgical removal of the suspected tumor |
50-year-old man with progressive bulge in his abdomen during past 6 months. No changes in bowel habits. Midline hernia above umbilicus that cannot be reduced be gently pushing on it. Operative repair initiated. Which extracellular matrix components requir | Collagen |
50-year-old man with progressive bulge in his abdomen during past 6 months. No changes in bowel habits. Midline hernia above umbilicus that cannot be reduced be gently pushing on it. Operative repair initiated. Which extracellular matrix components requir | Lysosomes |
53-year-old man in ED 1 hour after right-sided weakness. Right perioral droop. Babinski sign present on the right. CT scan of the head shows no abnormalities. One week later, a repeat CT scan shows a small area of hypodensity involving the left internal c | Microglial cells |
38-year-old woman in for pre-employment exam. No illness. No meds. Labs show Hb 8.2, HCT 25%, MCV 69. WBC 5.9k, Retics 0.8%, platelets 350k. Dx? | Iron deficiency anemia |
27-year-old man with acute myelocytic leukemia receives high-dose cyclophosphamide in preparation for hematopoietic stem cell transplantation. Which will decrease toxicity from this chemotherapy regimen? | Mesna |
30-year-old man develops hemoptysis, dyspnea, weakness, and hematuria. Diffuse pulmonary hemorrhages bilaterally. Renal biopsy shows focal glomerular necrosis with crescent formation and linear deposition of IgG and C3 in glomerular capillary loops. Patho | Autoantibodies against host cell basement membranes |
21-year-old woman with 2-week hx of blood-tinged vomiting. 2-year hx of self-induced vomiting after gorging on food. BMI 24. Which drugs is most appropriate? | Fluoxetine |
56-year-old man scheduled for physical therapy 3 days following right shoulder operation. Therapy to strengthen the infraspinatus and teres minor muscles. Which should this patient perform against resistance? | Lateral (external) rotation |
cASPofungin | cell wall inhibitor used in invasive aspergillosis |
Rx to avoid in Pregnancy | SAFE Moms Take Really Good Care Sulfonamides, Aminoglycosides, Fluoroquinolones, Erythromycin, Metro, Tetra, Ribavirin, Griseofulvin, Chloramphenicol |
22q11 synd (DiGeorges) | Truncus Arteriosus, TOF |
Heart defects seen in Turner's Syndrome | Preductal Coarctation |
Heart defect seen in Congential Rubella | Septal defects, PDA, Pulmonary Artery Stenosis |
Problems seen in diabetic mother's children | Transposition of great vessels hypoglycemia after birth clavical fractures and erb's palsy |
Sx Hypochloremic metabolic alkalosis w hypoKalemia, nonbilious projectile vomiting | Congenital Plyloric Stenosis |
Thoracodorsal + pathology | unable to wipe bottom |
Suprascapular + pathology | trouble initiating arm abduction |
Nerve runs with lateral thoracic artery | long thoracic nerve |
Nerve runs with deep brachial artery | radial nerve |
Medial Nerve + pathology | loss of forearm protonation |
Hemochromatosis Gene | HLA-A3 |
PAIR Ankylosing spondylitis gene | HLA-B27 |
Graves' Dz Gene | HLA-B8 |
Axillary lymph node location/drain | upper limb lateral breast drains |
What drains to Celiac lymph nodes | stomach drains into |
Sigmoid colon drains to | colic --> inferior mesentary |
What drains to the internal iliac | rectum above pectinate |
What drains to the superficial inguinal | anal canal scrotu thigh |
What drains to superficial/deep plexus -> paraaortic lymph node | testes |
What drains to thoracic duct | drains to L subclavian & internal jugular |
What drains to right lymphatics | drains to brachiocephalic vein |
IL-12, IFN b, IFN a --> | enhances NK cells |
T cells receptors | TCR CD28 CD3 |
CD 21= | EBV receptor on B cellls |
Helper T cell receptors | CD4 CD40L |
B cell receptors | CD19,20,21 CD40 MHCII B7 |
Macrophage receptors | MHC II B7 CD40 CD14***(endotoxin receptor) receptor for Fc & C3b |
NK cell receptors | MHC I CD16 (binds IgG Fc) CD 56*** |
Protection from Complement | CD55 CD59 |
IL1--> | stimulates endothelium adhesion molecues |
IL6--> | fever acute phase proteins |
IL8--> | major neutrophil chemotaxis |
Neutrophil chemotaxis--> | IL8 Leukotriene B4 C5a |
IL12--> | T cells into Th1 cells activate NK cell secreted by B cells |
Secreted by Macrophage--> | IL 1,6,8,12 TNF a |
TNF-alpha causes | septic shock vascular leak, activate endothelium acute phase pro |
Secreted by Th1 cells | IL2 IFN y |
Secreted by Th2 cells | IL4 IL5 IL10 |
IL5--> | stimulates eosinophils |
Interferons a & b causes | induce ribonucleases block viral pro synthesis |
IFN y causes | increase in MHC I and II |
IL3 causes | all T cells to secrete |
Bacteria with Ag variation | Salmonella (2 flagellar) Borrelia N gonorrhea (pilus) |
IFN a & b does what? | released by virally infected cells |
IFN y does what? | inhibits production of Th2 cells |
Terminal deoxynucleotidyl transferase--> | adds DNA during recombination of Ab diversity (B cells) |
C3b & IgG | primary opsinins |
Preformed Ab= passive immunity conditions | To Be Healed Rapid Tetanus, Botulinum, HBV, Rabies |
Name all granulomatous conditions | fungal (histo, blasto) syphilis (gummas) leprosy cat scratch fever (Bartonella henseliae) sarcoid crohn's berylliosis listeria foreign bodies wegeners (necrotizing granulomas) Chronic Granulomatous Dz |
MOA of hypersensitivity type 1 | IgE and histamine 15 min |
MOA of hypersensitivity type 2 | Ab mediated hypersensitivity IgM, IgG direct and indirect Coombs test |
MOA of hypersensitivity type 3 | Arthus reaction Ag-complement-IgG complex 5-12 hr |
MOA of hypersensitivity type 4 | T cell mediated 24-48 hr |
B cell Conditions | Bruton's CVID Hyper IgM Ig deficencies |
T cell Conditions | DiGeorges Job's synd (FATED) IL-12 r def chronic mucocutaneous candidiasis |
B & T cell Conditions | SCIDS Ataxia-Telangiectasia Wiskott-Aldrich |
Phagocyte Conditions | Chediak & Job Chronicallly Lack phagocytes (CGD and Leukocyte adhesion def) |
DiGeorges Child HALLMARK | young child w tetany from hypoCa++ and candidiasis |
CGD Child HALLMARK | young child with recurrent lung infxn and granulomatous lesions |
Ectoderm missing in DiGeorges | 2yo child multi viral and fungal infxn, hypoPTH, what germ layer gives rise to missing organ? |
Jobs Syndrome Symptoms | repeated Staph abcesses, neutrophils don't respond to stimuli |
DiGeorge Symptoms | heart defects and repeat viral infxn, low T cells |
MOA in Bruton's | no tyrosine kinase gene, low Ig of all classes |
MOA in Hyper IgM | defective CD40L severe pyogenic infxn high IgM, very low IgG |
IgA Deficency Symptoms | milk allergy repeated sinus infxn |
CVID Symptoms | defect in B cell maturation lymphomas low plasma cells |
IL-12 receptor Deficency Symptoms | disseminated mycobacterial infxn low IFN y |
IL-2 r, ADA def, MHC II def= | 3 types of SCID |
SCID labs | high adenine low IL-2r |
Ataxia Telangiectasia Symptoms | DNA repair enzyme defect IgA def |
Wiskott-Aldrich Symptoms | X-recessive Thrombocytopenic purpura Infxn Eczema Recurrent pyogenic infxn |
Leukocyte Adhesion Deicency Symptoms | defect in LFA-1 integrin delayed umbilicus separation |
Chediak Higashi Symptoms | partial albinism pyogenic infxn neuropathy |
Deficency of MAC C5-9 Symptoms | recurrent Neisseria infxn |
EEG waveforms | BATS Drink Blood Beta= eye open Alpha= awake, eyes closed Theta= light sleep Sigma= sleep spindles Delta= low freq, high amplitude REM= Beta= highest freq, low amplitude |
SEM * 2 +_ mean | how do you determine 95% confidence interval? |
3rd gen cephalosporins uses | serious gram - (Ceftriaxone= meningitis & gonorrhea) |
4th gen cephalosporin uses | pseudomonas & gram + |
Aztrenonam USE | binds PBP3 no penicillin allergy |
How are aztreonam & aminoglycosides similar? | serious gram - infxn (aztreonam= aminoglycoside pretender) |
Imipenem & meropenem | enterococci gram +/- anareobes (very broad) |
Next step in TX of otitis if resistant to amoxicillin | Augmentin |
What increases nephrotoxicity of aminoglycosides? | Cephalosporins |
Symptoms of Haemophilus influenza | MOPE Meningitis, Otitis media, Pneumonia, Epiglossitis |
Symptoms of Pseudomonas | PSEUDO Pneumonia, Sepsis, External otitis, Uti, Drug use, diabetic Osteomyelitis |
Enterobacteriacae | gut KEES PSS Klebsiella, E coli, Serratia, Proteus, Salmonella, Shigella |
Most common cause of gram - sepsis | Ecoli + Klebsiella |
Symptoms of Protease | carries urease cause staghorn calculi in renal |
Symptoms of Klebsiella | 4 A's Aspiration pneumonia, Abscess in lungs, Alcholics, diAbetics nosocomial UTI's |
Urease + Bugs | H. pylori Proteus |
Cat scratch organism | Bartonella Henselae transmission |
Dog/cat bite organism | Pasturella Multocida transmission |
Cat feces organism | Toxoplasmosis transmission |
Puppy feces organism | Yersinia enterocolitica transmission |
Animal urine organism | Leptospira transmission |
Rat bites organism | Spirillum Minus transmission |
Spirochetes organism | BLT Borrelia, Leptospira, Treponema |
Reiter's Syndrome bug | shigella flexneri C trachomonas D-K |
Motile organism cause UTI | Proteus |
Uses for macrolides | PUS Pneumonia (atypical- mycoplasma, chlamydia, legionella), UTI, STDs |
Most common UTI bugs | PEcK+ S. Saphrophyticus #1= Ecoli |
Drugs for anaerobic infections | Metro, clindamycin, imipenem |
Conditions associated with Use of Metronidazole | GET GAP on the metro Giardia, Entamoeba, Trichomonas, Gardnerella, Anaerobes, h Pylori |
MOA of Metronidazole | from free rads in bacteria- damage DNA |
H pylori Triple Therapy | PPI, clathromycin, amoxicillin/metro |
Effective against Pseudomonas | TCP Cefapime Aztreonam Fluoroquinolones Aminoglycosides Polymixins |
What type of hallucinations causes what type of conditions | ViAO = De Sc Ep tion Visual= Delirium Auditory= Schizo Olfactory= Epilepsy |
Side Effects of TCA | Tri=C's Convulsions, Coma, Cardiotox |
Side Effects of High Potency Neuroleptics | haloperidol, trifluoperazine, fluphenazine NMS & tardive dyskinesia |
Side Effects of Low Potency Neuroleptics | thioridazine, chlorpromazine anticholinergic, antihistamine, a block |
Patient on diphenhydramine & dementia wants sleep meds, which Rx | trazadone or high potency antipsychotics (b/c less anti-Ach SE) |
MOA of benzos and barbituates | increase in GABA (cl- channel vs duration) |
SNRIs | venlafaxine, duloxetine, nefanizone |
MAOIs | the MAOI PITS Phenelzine, Isocarboxazid, Tranylcypromine, Selegiline |
NDRI | Buproprion |
Tetracyclics | Mirtazapine= use for Depression w insomnia Trazadone= use for insomnia |
Prevent relapse in alcoholics | 1 AA 2 disulfram 3 naltrexone 4 topiramate 5 acamprosate |
Unique properties of this Rickettsial organism | Coxiella Burnetti neg Weil Felix from tick feces and cattle placenta - spores aerosilized sx w no rash |
Zoonotic Bacteria | Big Bad Bed Bugs From Your Pet (Ella) Bartonella spp, Borrelia burgdorferi, Borrelia recurrentis, Brucella spp, Francisella tularensis, Yersinia pestis/enterolytica, Pasturella multocida |
Symptoms of Chlamydia Trachomatis | 50% subclinical type D-K: urethritis, PID, ectopic preg, neonatal pneumonia type L1,2,3: lymphogranuloma venereum, lympadenitis, ulcers |
Symptoms of C. Pneumoniae & Psittaci | atypical pneumonia (aerosol) |
Antibiotics contraindicated in hepatic insufficiency | MCMCRT Metro, Chloramphenicol, Macrolides, Clindamycin, Rifampine, Tetracycline |
Ghon focus | Calcified Scar |
Ghon complex | Ghon focus + hilar nodes |
Symptoms of Mycobacterium kensaii | cause pulm TB-like sx in COPD pt |
Symptoms of Mycobacterium kensaii in child | cause cervical lymphadenitis in children |
Impenem SE of Ethambutol | red-green color blindness |
Rifampin uses | TB & leprosy meningococcal prophylaxis Hib |
Macrolide drugs | azithromycin class |
Fetal Erythropoiesis | Young Liver Syn Blood Yolk Sac-> Liver->Spleen->BM |
Truncus Arteriosus | ascend. Aorta/Pulm trunk embryol structure |
Bulbis Cordis | R ventr and smooth parts of L&R ventri embrol structure |
Trabeculated parts of L&R ventricle | L ventricle embryo structure |
Pain radiating to back | acute pancreatitis dissecting aortic anuersym diaphragm pain cholecystitis |
Patent urachus | urine discharge from umbilicus vesicourachal diverticulum (=lesser) |
Vitelline duct fails to close | meconium discharge from umbilicus meckel's diverticulum (=partial closure) |
Primitive atria becomes | trabeculated L&R atria embrological structure |
A gardener presents with SOB, salivation, miosis, and diarrhea. What is the cause/ MOA? | Organophosphate poison, anticholinesterase |
Atropine is not effective in reversal of organophosphate poisoning. Why? What helps? | No effect on cholinesterase, use Pralidoxime |
What muscarnic agonist / antagonist is used in asthma / COPD? | Ipratropium (an antagonist) |
30 YO has urinary rentention due to neuroleptic, what do you treat with? | Cholinergic Agonist (problem is anti cholinergic s/e's |
In Dark both pupils dialate. In light one pupil is miotic while another, given drug X, is mydratic. What is X? | Anticholinergic (atropine) |
What drug is most apropriate in a pt with shock in order to maintain renal blood flow | Dopamine (although clinically doesn't really work) |
60 YO male. Has a hard time driving at night due to worsening vision and halos appearing around headlights. What is causing this? | Cataracts |
A gymnast sustains an anterior shoulder dislocation. What nerve is injured? | Axillary n. |
A kid falls while skateboarding and injures his elbow. He can't feel the medial part of his palm. What nerve is injured? What "sign"? | Ulnar N. Ulnar Claw (can't extend 4/5 digits) |
A highschool athlete falls on his arm. Radiograph shows midshaft break of humerous. Which nerve / artery are at risk? | Radial n. Deep Brachial Art. |
What patients are suseptible to Listeria? | Immunocompromised, Neonates, Pregnant Women |
What organisms are implicatd in subacute endocarditis? | S. Veridians, Staph Epi, Enterococci. Staph Aure = Acute not subacute |
A woman is breast feeding develops swelling and redness over her right breast. Exam reveals a warm, fluctuant mass. What is this? | Acute Mastitis -> Staph Aures |
Most common aerobic skin flora? | Staph Epidermis |
6 month old child is given HONEY for a cough and cold and becomes flaccid. What causes this? MOA? | C. Botulinum (Gm + Rod) inhibits Ach release |
One hour after eating a potato salad at a picnic. Whole family vomits. 10hrs later they are better. Whats the cause? | Staph Aureus. Preformed toxin ingested (no infection) |
Which complement is responsible for neutrophil chemotaxis? | C5a (also leukotriene B4, IL8) |
Child presents with TETANY from hypocalcemia and CANDIDASIS due to immune suppression. What is deficient? What is the condition? | T cells, No Thymus = DiGeorges (22q11, "CATCH-22") |
A young child has recurrent LUNG infection and granulomatous lesions. What defect in neutrophils causes this? | Lack of NADPH oxidase = no respiratory Burst |
Mother brings 2 y/o child w/ Hx of multiple viral, fungal infections and the child is HYPOCALCEMIC. Which Germ layer gives rise to the missing structure? (Endo, Ecto, Meso) | No Thymus, DiGeorge Syndrome, Endoderm (from tongue) |
A child with immune diorder w/ repeated Staph Abcesses. Neutrophils do not respond to chemotactic stimuli. What is the diag? | Hyper IgE aka Job Syndrome |
A patient suffers recurrent Neisseria Infections. What part of complement is defective? | C5-C9, (LatE) |
How does mechanism of Type 2 Hypersensitivity differ from Type 3? | Type 2 = Ab against SELF antigens. Type 3 = Ab's against REAL antigens. Complexs get stuck places and cause problems. |
45 y/o female, MALAR RASH and ARTHRITIS. Which Ab is specific for the condition? | Anti dsDNA, Anti Smith. ANA is nonspecfic |
After bone marrow transplant a patient suffers dermatitis, enteritis, and hepatitis? What is the condition? | Graft vs Host Dz. |
A physican is looking for a risk factor for Pancreatitis. He interviews 100 w/ and 100 w/o pancreatitis. What kind of study is this? | Case Control |
New glucose test arrives. You test it with a solution of 90mg of glucose. The test gives you the following readings: 54, 56, 55, 54, 53, 56, 55, 54. What is its presions and accuracy? | High Precision; low accuracy (value stays in the 50s while not accurate which 50s value) |
A group of ppl who smoke and do not smoke are followed over 10 years. Every two years they check who develops cancer. What kind of study is this? | Cohort |
A certain screening test has a 1% false negative rate. What is the sensitivity? | 99% |
Prevalence of Varicella in Pop A is 2x that of Pop B. It has the same incidence in both populations. Why is the prevalence different? | Dz in Pop A has longer duration |
State the diagnosis: Gm (-), OXIDASE (+), DIPLO COCCI | Neisseria |
22 y/o medical student. Burning feeling AFTER MEALS. EGD shows gm (-) RODS in gastric mucosa, what are they? | H. Pylori |
50 y/o male smoker with new cough and flu like symptoms. Gm stains shows nothing. SILVER STAINS shows rods. What is the diagnosis? | Legionella (atypical pneumonia) |
40 y/o female. Acute unilateral knee pain and bilateral BELLS PALSY. What organism? How is it transmitted? | Lyme Disease via Burreli Burgdorferi via Tick |
21 y/o male. 5 day hx for fever chills and enlarged painful knee. What organism? And what treatment? | Gonorrhea -Ceftriaxone or Azithyromyocin if allergy |
After taking a course of Amoxicillin, and adult pts develops toxic MEGACOLON and DIARRHEA. What caused this? | C. Diff |
25 y/o with mycoplasma atypical pneumonia, exhibits anemia due to cryoagglutinins. What type of Ig is responsible for anemia? | IgM |
Homeless alcoholic pt vomited while intoxicated. Develops FOUL smelling sputum. What organism? | Klebsiella or anerobe |
65 y/o asks husband to stay in hospital overnight b/c she is afraid of being alone. What defense mechanism is this? | Regression |
Which defense mechanism underlies all others? | Repression |
60 y/o man admitted for chest pain, jumps out of bed and does 50 push ups to show he has not had a heart attack. What is the defense mechanism? | Denial |
4 y/o girl complains of painful genitalia. On exam discharge with smear showing N. gonorrhoeae. What happened? | Sexual Abuse |
72 y/o patient is unable to recall 3 objects during mini mental status exam. When asked what he would do if he smelled smoke he says "yell fire". When asked what a table can chair have in common he says both are made of wood. Family reports he needs const | Dementia |
72 y/o brought to the clinic by family. Strange behaviors in last week. Very agitated, NAPS frequently during the day, URINATES on self, poor appetite. Unable to focus during exam. Diagnosis? | Delerium |
You are on call and receive a call from a nurse asking to give sleep medication (diphenhramine) to an ELDERLY pt. with DEMENTIA. What do you do? | No diphenhydramine, no Benzos, use Trazadone or Haliperidol |
A patient tries to commit suicide by cutting wrists. After beening cared for in the ER, what question would you ask to determine her level of commitment? | What did she do after cutting her wrists? Call someone? Lie in a bathtub? |
28 y/o female with mild depression for 2 yrs. What diagnosis? | Dysthamia |
2 months after losing her spouse a 42 YO female is having trouble eating, concentrating, and sleeping/ What do you do? | This is still with in normal. But you can tx the insomnia and help with trazadone or something else |
A patient on whom you want an MRI tells you they are claustophobic. What can you do? | Give two Benzos prior to MRI |
A young woman is anxious about her 1st pap smear and is told to realx and to imagine what the steps are. What is the process and example of? | Systamatic Desensitization |
A woman has flashbacks about her boy friends death one monnth ago in a hit and run accident. She often cries and wishes for justice. Diagnosis? | Normal Greif |
Nurse has hypoglycemia with no elevation of C-protein. Diagnosis? | Malingering or Facistious depenidng on 2nd gain |
40 YO female tells you she is in love with you. You refer her (which you should never do during USMLE) and she attempts suicide. What is this personality disorder? | Splitting (Borderline ) |
30 Yo woman tells you that you are the best doctor and the nurses are very bad. On subsequent visit she threatens to change doctors because you do not feel a specific lab test is justified. You also notice several "scartches" on her left arm. What persona | Splitting (Borderline ) |
55 YO female wearing all black with a black feather boa and excess lipstick. What type of personality disorder? | Histrionic |
A pt. demands only the best most famous doctor in town. What personality disorder? | Narcissistic |
A patient returns from a trip to New Mexico, now has pneumonitis. What is fungal cause? | Coccidioidomycocces |
A 30 Yo female has "cauliflower" skin lesion. Tissue biopsy shows broad based budding yeast. What is this organism? | Blastomycoisis |
An HIV (+) pt with CSF showing 75/mm3 lymphocytes suddenly dies. Yeast is identified in the CSF. What is the diag? | Cryptococcus |
A pt presents with a "rose garden scenario" (thorn prick with ulcers along lymphatic drainage). What is infection? | Sporothrix |
A Pt who visited Mexico presents with Bloody Diarrhea. What infection could be found in the stool? | Cryptosporidium (usually filtered from city water supply....) more severe in AIDS |
32 YO male went camping in N. California 2 wks ago. Pt had a 2 day stint of diarrhea and how has liver damage and Jaundice. What is the diagnosis? | Entamoeba Histolytica (not Giardia b/c Jaundice is present) |
Pt returns after 2 wk vacation in Africa. Typical malaria presentation and recurrent fever. What is the mechanism for the cyclic? | Malaria cycle in RBS's causing lysis every 48-72 hrs |
Which Fetal Vessel has the highest 02 concentration? | Umbilical Vein (1 verin, 2 arteries) |
45 YO male with BP 160/90 on right arm and 170/92 on the left arm. No pulse in feet or ankle. What is the diagnosis? | Coarctation of the Aorta (adult type) |
Describe blood flow through a PDA? Just for fun what keeps it open, what closes it? | Left to right shunt. (during pregnancy not called "patent" so incorrect to say R-> L then becomes L-> R). Hear a continuous machine murmer. Open = PGE, close = indomethacin |
Monozygotic twins are delivered. One is pale and has a hematocrit of 15% the other is flushed with hct of 55%. What caused this? who will do better? | This is twin transfusion, mostly like due to monochorionic, mono amniotic pregnancy. Twin with lower hct will do better due to "sludging" in the one with high hct. |
A child presents with Cleft lip. What process failed? | Fusion of the maxillry process with the Medial Nasal Process. Cleft lip is mostly a aesthetic defect were cleft pallate has functional defiect as well |
23 YO male presents with one testicle. what is he at risk for? | Germ Cell tumor of teste |
24 YO male develops testicular cancer. Mets spread where? | via Inguinal canal to para-aortic LN's |
16 YO female with amenorhea. Pt lacks uterus and uterine tubes. Has two round structures in midline just superior to labia majora. What is the diagnosis? | Androgen Insensitive (46XY) |
While on an ACE-i a pts develops a cough. Why? what is a replacement? | No Ang II = build up of bradykinn = cough. Use AngioTensin II Receptor Blocker (ARB) |
40 yo male on lipid lower meds. Develops rash, puritis and diarrhea. What drug? Is this a allergic rxn? what mediates this response? | Niacin, not allergic due to prostaglandins (take asprin b4 hand to decrease) also dec with long term therapy) |
What is mechanism of action of Cardiac Glycosides (Digoxin)? | Blocks the Na/K atpase pump. Na leaves cell via Na/Ca counter transporter and increases intracellular Ca lvls = better contractility |
An abdominal Aortic aneurysm is most likley due to? | Atheroscerois |
A pt with poorly controlled HTN has actue sharp substernal pain raidiating to the back. Death occurs within a few hours. Diagnosis? | Dissecting Aorta |
During a high school football game a young athlete collapses and dies immediately. What is the condition? | Hypertrophic cardiomyopathy |
What murmers are heard best in the Left Lateral Decubitous Position? | Mitral Stenosis/ Regurg. And Left sided S3 / S4 |
80 YO male. systolic cresendo decresendo murmur. What is the condition? | Aortic Stenosis (probabley due to calcified aorta) |
IV drug user presents with Chest pain, dyspnea, tachycardia, tachypnea. What is the condition? | Bacterial Endocardidits -> PE (remeber drug users get it on the right) |
Pt brought to ER after MVA presents with chest pain, dyspnea, tachycardia, tachypnea. What is the condition? | Tension Pneumothorax most likely, could be Cardiac Tamponade too |
Post-op pt presents with Chest pain, dyspnea, tachycardia, tachypnea. What is the condition? | PE |
A young girl with a congeital valve dz is given penicillin prophylactically. In the ER bacterial endocarditis is diag. What is next? | IV Vancomyocin and possible echo to check valves (might need replacement) |
An adult pt with hx of HTN presents with sudden sharp, tearing pain, radiating to his back. What do you see on CXR? | Widening of the mediastinum (Dissecting Aorta) |
On auscultation of a patient you hear a pansystolic murmur at the apex with radiation to the axilla. Cause? | Mitral Regurgitation (Aortic Stenosis is not PANsystolic) |
A 25 YO pregnant woman in her 3rd trimester has normal BP when standing and sitting but drops to 90/50 when she lies supine. What is the condition? | Compression of IVC, dont lie on your back. |
45 YO male with squamous cell carcinoma of the penis. He had exposure to what Virus? | HPV - 16 / 18 |
20 YO college student presents with LAD, fever, and hepatosplenmegaly. His serum agglutinates sheep RBC's. What cell is infected? | B Cells - EBV mononucleosis (+) monospot. Cause of (-) monospot mononucleosis? CMV |
How does rabies travel through the CNS to cause fatal encephalitis and seizures? | Retrograde along neurons |
What is the characteristic shape of rabies? | Bullet Shaped |
An adolescent presents with cough and rust colored sputum. What does gm stain show? | Strep. Pneumo -> Gm (+) diplococci |
HIV (+) pt with a CD4 count of 250 presents with signs of meningitis. CSF shows a heavily encapsulated organism. What is it? | Cryptococcus Neoformans |
An older patient has blood in his urine and renal stones. What organism? | Proteus |
A 50 YO pt is recovering from Abd surgery from 2 days ago. He has had an internal catheter in place since then. He now has a fever of 100F. Most likely organism? | E-Coli (UTI) |
Hemidesmisomes, cadherin, integrin, ICAM-1. Which joins only cells of teh same type and does not attach to the basement membrane? | Cadherin |
Where does new bone formation take place in growing long bones? | Epipseal Plate |
A football player was kick in the legs and suffered a damaged medial meniscus. What else is likely to have been damaged? | ACL, MCL |
A man presents with pain and swelling of the knees, subcutaneous nodules around the joints and achilles tendon, equisite pain in the metatarsophalangeal joint of his right big toe. Biopsy reveals needle like crystals. | Most likely Gout |
Treatment of Acute Gout exacerbation? | NSAID, Colchicine |
A patient has difficulty swallowing, distal cyanosis in cold temp, anti-centromere antibodies. What other S/S will you see? | CREST: Calcinosis, Sclerodactyly, Telangiectasia |
A patient presents with photosensitivity, arthritis, renal disease and recurrent oral ulcers. She is taking Primaquine and NSAIDS. What should be checked 2/yr? | Renal Fxn (SLE) |
30 yo woman presents with low grade fever, rash across her nose and gets worse in the sun and widespread edema. What blood test would you use to screen? | ANA - SLE |
A CT scan of the chest shows bilateral hilar LAD. What is the diagnosis? | Sarcoidosis |
A 75 yo male presents with acute knee pain and swelling. X ray reveals erosion of the joint space and calcium deposits in the menisci. What is Diag? What would be found on FNA of joint? | PsuedoGout -> Calcium pyrophosphate |
A 50 YO female complains of double vision, amenorrhea and headaches. What is likely diagnosis? | Prolactinoma |
A patients MRI shows replacment of tissue in the sella tursica with CSF. What is the presentation? | Most likley asymptomatic or defieciency in Pituitary Hormones |
What hormones come from the anterior pituitary? | FSH, LH, ACTH, GH, prolactin, TSH |
Which hormones share a common alpha unit? | FSH, TSH, LH, bHCG |
A young woman is found to have short stature and shortened 4th and 5th metacarpals. What endocrine disorder is this? | Albrights osteodystrophy (Pseudohypoparathydroiism) aka body not responsive to PTH |
35 yo female presents with diffuse goiter and hyperthyroidism. What is TSH / T3/ T4? | Low TSH High T3,T4 |
48 YO female presents with progressive lethargy, and extreme cold sensitivity. What is Diag? Lab values? | Hypothyroid / Hasimotos most likely (High TSH low T3/4) |
An adult male with elevated serum cortisol and signs of cushing syndrome undergoes dexamethasone suppresion. 1mg does not decrease cortisol, 8mg does. What is diag? | ACTH secreting pituitary adenoma |
A very tan child with pale mother comes in and is found to be hypotensive. What is the condition? | Addisons |
28 YO male with normal well managed IDDM comes in with DKA hae had recently been taking OTC cold medicine. What caused his DKA? | Infection |
How is hemoglobin glycosylated in DM to make HA1c? | Non enzymatically (slowly do to glucose build up) |
What are the sources of Carbon for Purine formation? For pyrimidine? | Co2, Glycine, tetrahydrofolate / CO2 and aspartate |
How does UV light damage DNA? | Causes Thymine Dimers to form |
What ammino acid frequently has more coding sequences in mRNAt the represented in the peptide? | AUG - methione |
What happens to mRNA before it leaves the nucleus? | Spliced (remove introns), Poly A tail, and 5' Cap |
Two pts have the same mutation on chromosome 15. but they have different phenotypic expressions. One has a mutation from the father the other from the mother. What is this an example of? | Genetic Imprinting |
An obese woman presents with amenorrhea and increased serum testosterone. Diag? | Polycystic Ovarian Syndrome (stein-leventhaul) |
What type of cancer are patients with polycystic ovarian syndrome at risk for? | Endometrial |
Why is progesterone used in combo with estrogen in hormone replacement therapy? | To protect Uterus / endometrium from unregulated hyperplasia / cancer |
What circumstance would cause an elevated LH? | Polycystic Ovarian Syndrome (stein-leventhaul), physiological LH surge, low estrogen lvls, Turner Syndrome |
A pregnant woman with previous C section is at increased risk for what pregnancy complications? | Placenta Accreta, Previa |
A pregnant women at 16 weeks gestation presents with large abd and HTN. Diag? Lab values? | Hydatifrom Mole, increased beta HCG |
What substance is elevated in hydatifrom moles? | beta HCG (Really high in complete, slightly elevated in partial) |
15 YO pt who normally comes in with her parents presents alone. She states she is sexually active but knows she is not pregnant because she has never menstrated. What should you tell her? | Check for delayed puberty, talk about how you can get pregant on your first time even before you menstrated, STDs and maybe contraceptives |
23 YO female is on rifampin for TB and OCP. She gets pregnant. Why? | Rifampin increase Cyp450 metabolism of OCP, decreasing their effectiveness |
What is the best option of birth control of mental retarted pts? | Medroxyprogesterone (injection q3 months) |
A 58 YO post menopausal women is on Tamoxifen. What is she at risk of acquiring? | Endometrial Carcinoma |
What cells are responsible for maintaing a high testosterone concentration in the seminiferous tubules? | Leydig - Secrete; Sertoli - release of ABG = holds testosterone in place |
A 55 y/o man undergoing tx for BPH has increased testosterone and decreased DHT as well as gynecomastia and edema. What medication is he on? | Finasteride - 5 alpha reductase inhibitor (also used to treat renal stones) |
Where does testicular cancer first metastasize? | Para Aorotic Lymph Nodes |
What protein is involved in transporting an endocytosed vesicle from the plasma membrane to the endosome? | Clatherin |
A patient with a corticol lesion is UNAWARE of his neurologic deficiency. Where is the lesion? | Located on non dominate parietal Lobe (usually right) |
What are the findings of Brown Sequard Syndrome? | Ipsilateral UMN lesion below the lesion | Ipsilateral loss of tactile, vibration and proprioception below lesion | Contralateral pain and temp loss below lesion | ipsilateral all sension for a few levels above lesion | LMN at level of lesion |
A man in his 40s begins to develop early dementia and uncontrolable movements of his upper extremities. where in the brain do you expect to see atrophy? | Caudate -> Huntingtons |
A male presents with involuntary flailing of one arm. Where is the lesion? | This is hemibalismus. Contral lateral thalamus |
28 y/o chemist presents with MPTP exposure. What neurotransmitter is depleted? | Dopamine |
A patient cannot abduct her left eye on lateral gaze but convergence is normal. She also has difficulty smiling. Where in the CNS is the lesion? | CN IV (MLF tract) + CN VII. Both are at level of Pontine |
28 y/o woman in a MVA. Initally feels fine then loses consiousness. CT shows intracranial hemorrhage that does not cross suture lines. What bone and vessel were damaged? | MMA, and temporal bone |
85 y/o man with alzheimers falls at home and presents 3 days later with severe headache and vomiting. What is the diagnosis? What is damaged? | Subdural hematoma bridging veins |
A woman involved in a accident cannot turn her head to the left and has a right shoulder droop. What is damaged? | CN XI - accessory |
A 19 y/o pt presents with a furuncle on his philtrum and the cavernous sinous becomes infected. What might you see? | defects in CN 3, 4, 6 |
A pt has a leftward deviation of the tongue on protusion and has a right sided spastic paralysis. Where is the lesion? | Left Medulla + corticospinal tract (happens before the tract decusates) |